Você está na página 1de 216

INTRODUÇÃO AO

CÁLCULO

PROFESSORES
Me. Issao Massago
Me. Tiago Peres da Silva Suguiura
ACESSE AQUI
O SEU LIVRO
NA VERSÃO
DIGITAL!
EXPEDIENTE

DIREÇÃO UNICESUMAR
Reitor Wilson de Matos Silva Vice-Reitor Wilson de Matos Silva Filho Pró-Reitor de Administração
Wilson de Matos Silva Filho Pró-Reitor Executivo de EAD William Victor Kendrick de Matos Silva
Pró-Reitor de Ensino de EAD Janes Fidélis Tomelin Presidente da Mantenedora Cláudio Ferdinandi

NEAD - NÚCLEO DE EDUCAÇÃO A DISTÂNCIA


Diretoria Executiva Chrystiano Mincoff, James Prestes, Tiago Stachon Diretoria de Design Educacio-
nal Débora Leite Diretoria de Graduação Kátia Coelho Diretoria de Pós-Graduação Bruno do Val
Jorge Diretoria de Permanência Leonardo Spaine Head de Curadoria e Inovação Tania Cristiane
Yoshie Fukushima Gerência de Processos Acadêmicos Taessa Penha Shiraishi Vieira Gerência de
Curadoria Carolina Abdalla Normann de Freitas Gerência de Contratos e Operações Jislaine Cristina
da Silva Gerência de Produção de Conteúdo Diogo Ribeiro Garcia Gerência de Projetos Especiais
Daniel Fuverki Hey Supervisora de Projetos Especiais Yasminn Talyta Tavares Zagonel

FICHA CATALOGRÁFICA
Coordenador(a) de Conteúdo
Antoneli da Silva Ramos
Projeto Gráfico e Capa C397 CENTRO UNIVERSITÁRIO DE MARINGÁ.
Arthur Cantareli, Jhonny Coelho Núcleo de Educação a Distância. MASSAGO, Issao; SUGUIURA,
e Thayla Guimarães Tiago Peres da Silva.

Editoração Introdução ao Cálculo.


Lucas Pinna Silveira Lima Issao Massago; Tiago Peres da Silva Suguiura.

Design Educacional Maringá - PR.: UniCesumar, 2020.


Amanda Peçanha
216 p.
Revisão Textual “Graduação - EaD”.
Cindy Mayumi Okamoto Luca
1. Cálculo 2. Números 3. Equações. EaD. I. Título.
Ilustração
Natália Scalassara
Welington Vainer
Bruno Cesar Pardinho CDD - 22 ed. 515.5
CIP - NBR 12899 - AACR/2
Fotos Impresso por:
ISBN 978-65-5615-033-8
Shutterstock

Bibliotecário: João Vivaldo de Souza CRB- 9-1679

NEAD - Núcleo de Educação a Distância


Av. Guedner, 1610, Bloco 4Jd. Aclimação - Cep 87050-900 | Maringá - Paraná
www.unicesumar.edu.br | 0800 600 6360
BOAS-VINDAS

Neste mundo globalizado e dinâmico, nós tra-


balhamos com princípios éticos e profissiona-
lismo, não somente para oferecer educação de Tudo isso para honrarmos a nossa mis-

qualidade, como, acima de tudo, gerar a con- são, que é promover a educação de qua-

versão integral das pessoas ao conhecimento. lidade nas diferentes áreas do conheci-

Baseamo-nos em 4 pilares: intelectual, profis- mento, formando profissionais cidadãos

sional, emocional e espiritual. que contribuam para o desenvolvimento


de uma sociedade justa e solidária.
Assim, iniciamos a Unicesumar em 1990, com
dois cursos de graduação e 180 alunos. Hoje,
temos mais de 100 mil estudantes espalhados
em todo o Brasil, nos quatro campi presenciais
(Maringá, Londrina, Curitiba e Ponta Grossa) e
em mais de 500 polos de educação a distância
espalhados por todos os estados do Brasil e,
também, no exterior, com dezenas de cursos
de graduação e pós-graduação. Por ano, pro-
duzimos e revisamos 500 livros e distribuímos
mais de 500 mil exemplares. Somos reconhe-
cidos pelo MEC como uma instituição de exce-
lência, com IGC 4 por sete anos consecutivos
e estamos entre os 10 maiores grupos educa-
cionais do Brasil.

A rapidez do mundo moderno exige dos edu-


cadores soluções inteligentes para as neces-
sidades de todos. Para continuar relevante, a
instituição de educação precisa ter, pelo menos,
três virtudes: inovação, coragem e compromis-
so com a qualidade. Por isso, desenvolvemos,
para os cursos de Engenharia, metodologias ati-
vas, as quais visam reunir o melhor do ensino
presencial e a distância.

Reitor
Wilson de Matos Silva
TRAJETÓRIA PROFISSIONAL

Me. Issao Massago


Possui graduação em Pedagogia pela Faculdade de Filosofia, Ciências e Letras de
Jandaia do Sul (1990) e em Ciências - Habilitação Plena em Matemática pela Fun-
dação Faculdade de Filosofia Ciências e Letras de Mandaguari (1992). É mestre em
Matemática pela Universidade Estadual de Maringá (2015) e atua como professor
da disciplina de Matemática na rede pública estadual do Paraná. Sempre se preo-
cupou com o processo de construção de conhecimentos matemáticos e procura
atualizações em tecnologia educacional.

http://lattes.cnpq.br/5103036905790051

Me. Tiago Peres da Silva Suguiura


Possui graduação em Matemática pela Universidade Estadual de Maringá (2015) e
mestrado em Bioestatística pela mesma instituição (2017). Atualmente, é professor
mediador do curso de Licenciatura em Matemática da Unicesumar.

http://lattes.cnpq.br/4322415939992048
A P R E S E N TA Ç Ã O DA DISCIPLINA

INTRODUÇÃO AO CÁLCULO

Seja bem-vindo(a)! Este material foi elaborado especialmente para que você inicia o estudo
de alguns conteúdos relacionados à disciplina de Cálculo. Embora a relação dos conteúdos
abordados possa parecer um pouco estranha para aqueles que já tiveram algum contato com
essa disciplina, este material pode auxiliar no estudo das disciplinas de Cálculo. Pensando nis-
so, a ideia é introduzi-lo de forma simples, mantendo o foco e sem perder o rigor matemático.

Vale lembrar, ainda, que os conteúdos devem ser compreendidos, e não decorados. A aqui-
sição plena de conhecimentos ocorre somente quando conseguimos incorporá-los e, para
isso, precisamos acompanhar e reconstituir os raciocínios apresentados, de maneira que
possamos compreendê-los. Dessa forma, você deve focar na análise e na compreensão das
regras, das propriedades e, até mesmo, de algumas fórmulas, ao contrário de simplesmente
decorá-las, assim como muitos costumam fazer.

Temos plena consciência das possíveis dificuldades que você pode encontrar, ao mudar a forma
como encara a matemática, já que demoramos alguns anos de atuação, como professores dessa
disciplina, até começarmos a abandonar a forma tradicional de estudá-la e passarmos a vê-la
como algo que reserva muitas surpresas e margens para discussão. Além disso, precisamos ter
domínio pleno dos conteúdos abordados e da capacidade de analisar cada situação apresen-
tada, para que possamos transmitir o conteúdo de forma segura e que facilite a vida daqueles
que dependem de nós para aprender a matemática. Assim, além da aquisição dos conteúdos
matemáticos, começaremos a mudar um pouco a visão que temos acerca dessa disciplina.

Para facilitar o estudo, este material foi dividido em cinco unidades: na primeira, estudare-
mos os polinômios, as equações e as inequações; na segunda, as frações algébricas, pois,
para resolvermos muitas situações-problema, precisaremos de um novo tipo de equação;
já na terceira, a progressão aritmética e a progressão geométrica, dois tipos de sequências;
na quarta, a trigonometria, a qual, às vezes, é vista como algo de difícil compreensão ou até
um pouco confuso, mesmo que, comumente, esse fato se deve à falha na forma como é
apresentada para aqueles que precisam estudá-la; e, finalmente, na quinta, estudaremos os
números complexos, a unidade imaginária, suas formas de representação e propriedades.

Esperamos que este material auxilie sua formação acadêmica e contribua, também, para
sua prática docente, ao assumir uma sala de aula (se você ainda não é, será futuro(a) colega
de trabalho), para que, juntos, possamos sonhar com a melhoria da qualidade do ensino da
matemática, nesse país. Além disso, não podemos deixar de lhe desejar um bom estudo,
pois este material só terá valor na mão de alguém como você, que procura novos horizontes.
ÍCONES
pensando juntos

Ao longo do livro, você será convidado(a) a refletir, questionar e


transformar. Aproveite este momento!

explorando ideias

Neste elemento, você fará uma pausa para conhecer um pouco


mais sobre o assunto em estudo e aprenderá novos conceitos.

quadro-resumo

No fim da unidade, o tema em estudo aparecerá de forma resumida


para ajudar você a fixar e a memorizar melhor os conceitos aprendidos.

conceituando

Sabe aquela palavra ou aquele termo que você não conhece? Este ele-
mento ajudará você a conceituá-la(o) melhor da maneira mais simples.

conecte-se

Enquanto estuda, você encontrará conteúdos relevantes


online e aprenderá de maneira interativa usando a tecno-
logia a seu favor.

Quando identificar o ícone de QR-CODE, utilize o aplicativo Unicesumar


Experience para ter acesso aos conteúdos online. O download do aplicativo
está disponível nas plataformas: Google Play App Store
CONTEÚDO

PROGRAMÁTICO
UNIDADE 01
8 UNIDADE 02
45
POLINÔMIOS, FRAÇÕES
EQUAÇÕES E ALGÉBRICAS
INEQUAÇÕES

UNIDADE 03
67 UNIDADE 04
122
TRIGONOMETRIA PROGRESSÃO
ARITMÉTICA
E PROGRESSÃO
GEOMÉTRICA

UNIDADE 05
150 FECHAMENTO
182
NÚMEROS CONCLUSÃO GERAL
COMPLEXOS
1
POLINÔMIOS, EQUAÇÕES E
INEQUAÇÕES

PROFESSORES
Me. Issao Massago
Me. Tiago Peres da Silva Suguiura

PLANO DE ESTUDO
A seguir, apresentam-se as aulas que você estudará nesta unidade: • Polinômios • Equações polinomiais
• Sistemas de equações • Inequações

OBJETIVOS DE APRENDIZAGEM
• Identificar e efetuar operações que envolvem polinômios • Definir equações e resolvê-las • Construir
sistemas de equações e resolvê-los • Identificar as inequações e resolvê-las.
INTRODUÇÃO

Caro(a) aluno(a), seja bem-vindo(a) à primeira unidade do nosso livro!


Iniciaremos o nosso estudo, tratando de polinômios, equações e inequa-
ções. Apesar de existirem relações entre os assuntos, cada um apresenta
suas particularidades e, portanto, necessitam de alguns cuidados.
As equações, os sistemas de equações e as inequações estão presentes
no nosso cotidiano, mesmo que, às vezes, não atribuímos muita atenção.
Simples compras envolvem equações, pois certa quantia em dinheiro, ge-
ralmente, cédulas de alto valor, são trocadas por mercadorias, as quais re-
tornam um pequeno valor em espécie, nesse caso, o troco. As inequações,
que alguns acreditam que servem somente como desafios matemáticos,
também estão diretamente ligadas à nossa vida. Utilizamos as inequações
com frequência quando precisamos analisar se o dinheiro que possuímos
seria o suficiente para comprar certa quantia de mercadorias, por exemplo.
Assim, mesmo que alguns alunos, principalmente os da Educação
Básica, tenham certa rejeição a esses conteúdos, eles não devem ficar em
segundo plano. Talvez, a rejeição apresentada por alguns se deve ao fato
de não haver uma relação desses conteúdos ao cotidiano e de não existir
a realização da comprovação da validade de fórmulas ou de raciocínios,
devido à falha ou à ausência de contextualização e/ou demonstração.
Verificamos que os conteúdos matemáticos vieram para solucionar os
problemas do cotidiano, mesmo que tenham se tornado “pedras no sapa-
to” para alguns alunos. Para minimizar esse problema, estudaremos tais
assuntos a partir de uma ótica diferente, para que você incorpore mais um
pouco de conhecimento. Bom estudo!
1
UNIDADE 1

POLINÔMIOS

Iniciaremos o nosso estudo sobre polinômios a partir de sua definição. Assim,


como a raiz – poli – do próprio nome diz, o polinômio é composto por vários
termos, isto é, uma soma de monômios de graus diferentes, não admitindo ex-
poentes fracionários, nem negativos. Portanto, podemos defini-lo como:
n
ai xi  an xn  an1 xn1  an2 xn2    a2 x2  a1 x1  a0 , i
i 0
n
Observação: ai xi lê-se somatória de ai xi , com i variando de 0 a n .
i 0

O que é, no entanto, somatória? A somatória nada mais é do que a soma de vários


termos que seguem características definidas, variando a partir de um valor até
i
outro. No caso apresentado, a condição é ai x e i varia a partir de um determi-
nado valor (neste caso, 0 ) até outro (neste caso, n ). Por exemplo, se n = 5 , temos:
5
ai xi  a5 x5  a4 x 4  a3 x3  a2 x2  a1 x1  a0 x0 .
i 0

Nesse caso, o índice i de ai indica a ordem do coeficiente, isto é, indica a ordem


i
do termo, enquanto o índice i de x indica o valor do expoente.
10
O grau de um polinômio é determinado pelo maior dos expoentes da variável.

UNICESUMAR
Dessa forma, o polinômio apresentado
5
ai xi  a5 x5  a4 x 4  a3 x3  a2 x2  a1 x1  a0 x0 é de grau 5 (ou de 5º grau).
i 0

Observação: caso algum(ns) ai for(em) 0 , geralmente, não escrevemos esse(s)


termo(s).

Agora, estudaremos um pouco as operações que envolvem os polinômios.

Adição e subtração de polinômios

Considerando os polinômios
p1  an x n  an1 x n1  an2 x n2    a2 x2  a1 x1  a0 , n  

e
p2  bm x m  bm1 x m1  bm2 x m2    b2 x2  b1 x1  b0 , m  

tal que n > m , temos:


p1  p2  (an x n  an1 x n1  an2 x n2    a2 x2  a1 x1  a0 )
(bm x m  bm1 x m1  bm2 x m2    b2 x2  b1 x1  b0 )
 an x n  an1 x n1  an2 x n2    am1 x m1  (am  bm ) x m
(am1  bm1 ) x m1  (am2  bm2 ) x m2    (a2  b2 ) x2
(a1  b1 ) x1  (a0  b0 )

Exemplo 1: Considerando os polinômios


p1  4 x7  9 x6  4 x5  x 4  2 x3  6 x2  8 e p2  2 x 4  4 x3  x2  7 x  5 ,
procedemos da seguinte maneira para determinar a sua soma:

p1  p2  (4 x7  9 x6  4 x5  x 4  2 x3  6 x2  8)  (2 x 4  4 x3  x2  7 x  5)
 4 x7  9 x6  4 x5  (1  2) x 4  (2  4) x3  (6  1) x2  7 x  (8  5)
 4 x7  9 x6  4 x5  3 x 4  6 x3  7 x2  7 x  3
11
Observação: é usual omitirmos o coeficiente 1 dos termos de polinômios.
UNIDADE 1

A diferença entre polinômios é determinada de forma análoga à da soma dos polinô-


mios, tendo em vista que toda subtração pode ser transformada na adição do minuen-
do com o polinômio oposto do subtraendo. Entretanto o que é polinômio oposto?
O polinômio oposto é aquele cujo sinal de cada termo é oposto do polinômio dado.

6 5 4 3 2
Exemplo2:Considerandoospolinômios p1  x  7 x  2 x  5 x  6 x  15 x  3
7 6 5 3 2
e p2  4 x  x  4 x  4 x  6 x  x  10 , procederemos da seguinte maneira
para determinarmos a diferença entre o primeiro e o segundo polinômio:

p1  p2  ( x6  7 x5  2 x 4  5 x3  6 x2  15 x  3)  (4 x7  x6  4 x5  4 x3  6 x2  x  10)
 ( x6  7 x5  2 x 4  5 x3  6 x2  15 x  3)  (4 x7  x6  4 x5  4 x3  6 x2  x  10)
 4xx7  (1  1) x6  (7  4) x5  2 x 4  (5  4) x3  (6  6) x2  (15  1) x  (3  10)
 4 x7  3 x5  2 x 4  9 x3  12 x2  14 x  13

Para determinarmos o grau da soma dos polinômios, temos duas situações:


a) Se o grau dos polinômios envolvidos for diferente, o grau da soma dos
polinômios será o maior grau verificado.
b) Se o grau dos polinômios envolvidos for igual, o grau da soma dos poli-
nômios será o maior grau de termos não opostos.

3 2 3 2
Exemplo 3: Considerando p1  5 x  6 x  15 x  3 e p2  4 x  6 x  x  10 ,

o grau da soma p1 + p2 é 3 , pois o grau de p1 e p2 são iguais e os primeiros termos

3 3
de p1 e p2 não são opostos, isto é, 5 x  4 x .

4 3 2
Exemplo 4: Considerando os polinômios p1  6 x  5 x  6 x  15 x  3 e

p2  6 x 4  5 x3  6 x2  x  10 , o grau da soma p1 + p2 é 2, pois os termos de

2 2
maior grau que não são opostos são 6 x  6 x , tendo em vista que os termos

de grau 4 e 3 desses polinômios são opostos.


12
Multiplicação envolvendo polinômios

UNICESUMAR
Recorremos à propriedade distributiva da multiplicação em relação à adição para
determinarmos o produto, assim como podemos observar a seguir:

a) Multiplicação de polinômio por monômio: nesse caso, multiplicamos


cada termo do polinômio pelo monômio.

3 2
Exemplo 5: Considerando p1  7 x  5 x  3 x  1 e p2 = 2 x , procedemos da
seguinte maneira:

 
p1  p2  7 x3  5 x2  3 x  1  2 x

 
 7 x3  2 x  5 x2  2 x  3 x  2 x  1  2 x

  7  2  x31   5  2  x21   3  2  x11  1  2  x


 14 x 4  10 x3  6 x2  2 x

Observação: na multiplicação da potência de mesma base, conservamos a base


a b a b
e adicionamos os expoentes, isto é, x  x  x .

b) Multiplicação de dois polinômios: para a multiplicação de dois poli-


nômios, aplicamos a propriedade distributiva em todos os termos. Por
exemplo: considere os polinômios
p1  an x n  an1 x n1  an2 x n2    a2 x2  a1 x1  a0 , n  
e
p2  bm x m  bm1 x m1  bm2 x m2    b2 x2  b1 x1  b0 , m   .

Assim, teremos:

13

p1  p2  anxn +an-1xn-1 +an-2xn-2 + +a2x2 +a1x1 +a0 
UNIDADE 1


 bm x m  bm1 x m1  bm2 x m2    b2 x2  b1 x1  b0 
 anxn   bm x m  bm1 x m1  bm2 x m2    b2 x2  b1 x1  b0 

 an-1xn-1   bm x m  bm1 x m1  bm2 x m2    b2 x2  b1 x1  b0 

 an-2xn-2   bm x m  bm1 x m1  bm2 x m2    b2 x2  b1 x1  b0 

   a1x1   bm x m  bm1 x m1  bm2 x m2    b2 x2  b1 x1  b0 

 a0   bm x m  bm1 x m1  bm2 x m2    b2 x2  b1 x1  b0 

Ou seja, multiplicamos cada um dos termos do primeiro polinômio com todos


os termos do segundo e, em seguida, adicionamos os termos de mesmo grau.

Exemplo 6: Considerando os polinômios p1  7 x3  5 x2  3 x  1 e


p2  x2  4 x  7 , procedemos da seguinte forma:

 
p1  p2  7x3 -5x2 +3x +1  x2  4 x  7 
 7x3   x2  4 x  7   -5x2    x2  4 x  7   3x   x2  4 x  7 

 1   x2  4 x  7 

 7 x3   x2   7 x3   4 x   7 x3   7    5 x2    x2    5 x2    4 x 

  5 x2    7   3 x   x2   3 x   4 x   3 x   7 

 1   x2   1   4 x   1   7 

 7 x5  28 x 4  49 x3  5 x 4  20 x3  35 x2  3 x3  12 x3
 12 x2  21x  x2  4 x  7
 7 x5   28  5  x 4   49  20  3  x3   35  12  1 x2
  21  4  x  7
 7 x5  33 x 4  72 x3  46 x2  17 x  7

14
Observação: o grau do produto de dois polinômios ou de polinômio por mo-

UNICESUMAR
nômio é igual à soma de seus graus.

Divisão de polinômios

Para a divisão de dois polinômios, temos duas situações:

a) Divisão de polinômio por monômio: dividimos cada termo do poli-


nômio pelo monômio.

Observação: na divisão de potências da mesma base, subtraímos o expoente do


a b xa
divisor pelo expoente do dividendo, isto é, x  x  b
 x a b .
x
6 4 3 2
Exemplo 7: Para determinarmos o quociente do polinômio 9 x  5 x  6 x  8 x
2
pelo monômio 3 x , procedemos da seguinte maneira:

9 x6  5 x 4  6 x3  8 x2
   
9 x6  5 x 4  6 x3  8 x2  3 x2 
3 x2
9 x6 5x4 6 x3 8 x2
   
3 x2 3 x2 3 x2 3 x2
5 8
 3 x 6 2  x 4 2  2 x 3  2  x 2 2
3 3
5 8
 3 x 4  x2  2 x 
3 3

Observação: quando o expoente for igual a 0, sua potência é igual a 1 , isto é,


a  x0  a  1  a

b) Divisão de polinômio por polinômio: procedemos de acordo com o


exemplo a seguir:

3 2
Exemplo 8: Dividiremos o polinômio 10 x  11x  27 x  20 pelo binômio
2x + 5 :

15
10 x3 11x2 27 x 20 2x 5
UNIDADE 1

10 x3 25 x2 5 x2 7 x 4
14 x2 27 x
+14x 2 35 x
8 x 20
8 x 20
0

Observação: quando o resto da divisão é 0 , dizemos que é uma divisão exata.

5 4 2
Exemplo 9: Ao dividir o polinômio 20 x  58 x  97 x  25 x  17 pelo po-
2 3 2
linômio x  3 x  1 , obtemos o quociente 20 x  2 x  26 x  17 , assim como
podemos observar a seguir:

20 x5 58 x 4  0 x3 97 x2 25 x 17 x2 3 x 1


20 x5 60 x 4 20 x3 20 x3 2 x2 26 x 17
2 x 4 20 x3 97 x2
2 x4 6 x3 2 x2
26 x3 95 x2 25 x
26 x3 78 x2 26 x
17 x2 51x 17
17 x2 51x 17
0

conecte-se

Caro(a) aluno(a), para melhor compreensão, assista à resolução detalhada


desse problema no vídeo.

16
3
Observação: pelo fato de que não existia um termo com o expoente x no

UNICESUMAR
3
exemplo anterior, ele foi substituído por 0 ⋅ x .

Apresentamos um exemplo cuja divisão resultou em resto igual a 0 , mas isso não
ocorre sempre. Uma das maneiras para determinarmos o resto é efetuar a divisão,
mas existe uma maneira de verificarmos esse fato por meio de divisibilidade, quan-
b
do o divisor for um binômio do tipo a  x  b . Para isso, consideramos x   e
a
determinamos o valor numérico do dividendo, o qual será o resto da divisão.

Para finalizar esse conceito, surge o Teorema do Resto, que diz o seguinte:

Teorema (do resto): o resto da divisão de um polinômio P ( x) pelo binômio


b
a  x  b é igual ao valor numérico desse polinômio para x   , ou seja,
a
 b
P   r .
 a

3 2
Exemplo 10: O resto da divisão do polinômio 3 x  7 x  5 x  2 pelo binômio

x + 3 é igual a −35 . De fato, considerando x  3  x  3  0  x  3 , temos:


3 2
P(3)  3   3   7   3   5   3   2  35

Observação: o símbolo matemático “ ⇒ ” lê-se “implica” e significa que uma situação

resulta em outra verdadeira. Como anteriormente afirmado, x  3  0 resulta em

x  3 .

17
UNIDADE 1

3 2
Exemplo 11: O resto da divisão do polinômio 10 x  11x  70 x  20 pelo binô-
5
mio 2 x − 5 é igual a 70 . De fato, como 2 x  5  0  2 x  5  x ,
2
temos:
3 2
5 5 5 5
P    10     11     70     20
2 2 2 2
 125   25  5
 10     11     70    20
 8   4  2
625 275
   175  20
4 4
 70

pensando juntos

Você refletiu sobre o raciocínio que dá suporte à teoria de resto?

18
2
EQUAÇÕES

UNICESUMAR
POLINOMIAIS

As expressões matemáticas em que ocorrem igualdade entre polinômios são


chamadas de equações e algumas são bem conhecidas por todos. As equações
que trabalharemos nesta unidade são as de 1º grau e de 2º grau.
Quando dizemos “vamos resolver uma equação”, estamos a determinar o(s)
valor(es) da variável, tal que a expressão matemática se torne verdadeira.

Equação do 1º grau

O polinômio envolvido nessa igualdade possui grau 1 , ou seja, sua representação

é da forma a  x  b  0 . Esse tipo de equação é muito presente em nossa vida.

Vamos pensar em algumas situações:

Exemplo 12: Ao comprar uma roupa, você utiliza uma nota de R$100, 00 e

recebe uma nota de R$20, 00 de troco. Instantaneamente, você responde que

essa roupa custou R$80, 00 . No entanto, ao apresentar a equação x  20  100 ,


19
nem todos respondem que x = 80 , além de demorar mais para chegar ao resul-
UNIDADE 1

tado. Para resolvê-la, podemos proceder da seguinte forma:

x 20 100
x 20 20 100 20 Adicionamos ( 20) em ambos os lados da equação x
os lados da equação x 80

Exemplo 13: Se estivermos em uma mercearia e verificarmos que 4 pacotes de

doces custaram R$20, 00 , sem dúvida, a resposta é que cada pacote custou

R$5, 00 . Entretanto qual é o raciocínio utilizado?

Considerando o valor do pacote de doces como x , temos:

4  x  20
1 1 1
 4  x     20    Multiplicamos ambos os termos por  
4 4 4
 4  20
 x   
4 4
 x5

Exemplo 14: Para resolver a equação 4 x  8  2 x  2 , adicionaremos −8 e −2x

em ambos os lados da equação. Assim, temos:

4x  8  2x  2
 4 x  8   2 x    8   2 x  2   2 x    8 
 4x  2x  8  8  2x  2x  2  8
 2 x  6
1
Prosseguindo, multiplicaremos ambos os lados por . Assim, temos:
2
20
1 1

UNICESUMAR
2 x     6   
2 2
2 x 6
 
2 2
 x  3

Equação do 2º grau

O polinômio envolvido nesse tipo de equação é aquele cujo maior expoente é de


2
grau 2. Sua representação geral é ax  bx  c  0 , com a ≠ 0 . Uma das maneiras
mais conhecidas e utilizadas para resolver uma equação do 2º grau é por meio da
Equação de Bháskara (ou fórmula quadrática), dada da seguinte maneira:

b  b2  4 ac
x
2a

No entanto, às vezes, a origem dessa fórmula não é comentada com os alunos,


mesmo que a sua demonstração não seja difícil. Assim, vale a pena tentar com-
preendê-la. Vamos lá!
Primeiramente, vamos considerar uma equação geral do 2º grau dada por:
ax2  bx  c  0

tal que a, b, c   e a  0 .

Agora, adicionaremos −c em ambos os membros da equação, mantendo a


sua igualdade:
ax2  bx  c   c   0   c 
 ax2  bx  c

1
O próximo passo é multiplicar ambos os membros por . Assim, temos:
a

21
 ax2  bx    a1   c   a1 
UNIDADE 1

1 1 1


 ax2     bx     c   
a a a
a x2 bx c
  
a a a
bx c
 x2 +  (I)
a a
Agora, precisamos utilizar uma técnica chamada completar quadrado. Ela con-
siste, basicamente, em adicionar termos em sua equação para que ela se torne um
2
produto notável. Logo, como sabemos que  x  b   x  2bx  b , temos:
2 2

2
 b  2 bx b2
 x    x +  ,
 2a  a 4a2

Note que a parte em negrito é exatamente o que temos em nossa equação. Então,
resta-nos, apenas, acrescentar a última parte para que possamos transformar a
nossa equação em um produto notável.

2
Assim, ao adicionar b 4 a2 para ambos os membros da nossa equação I ,

é possível realizar a transformação do primeiro membro no produto notável


2
 b 
x  :
 2a 
2 bx  b2  c  b2 
x +    
a  4 a2  a  4 a2 
bx b2
2 4 ac  b2
x   2 
a 4a 4a2
2
 b  b2  4 ac
x  
 2a  4a2

22
Prosseguindo, extrairemos a raiz quadrada em ambos os termos:

UNICESUMAR
2
 b  b2  4 ac
 x     .
 2a  4a2
Todavia, como:
b2  4 ac b2  4 ac b2  4 ac
   ,
4a2 4a2 2a

Temos:
b b2  4 ac
 x  .
2a 2a

b
Agora, isolando a variável x , adicionaremos - em ambos os membros:
2a

b  b  b2  4 ac  b 
x  -     -  ,
2a  2a  2a  2a 

O que resulta em:

b  b2  4 ac
x
2a

conecte-se

Caro(a) aluno(a), para melhor compreensão, vejamos, no vídeo, alguns


exemplos de equações do segundo grau.

23
A equação do 2º grau também pode ser resolvida pelo método de fatoração,
UNIDADE 1

quando existir(em) raiz(es) real(is), tendo em vista que:


 x  x1    x  x2   x2   x1  x2   x  x1  x2
Assim, se x − x1 e/ou x − x2 for igual a 0 , a equação anterior, - dada por

x2   x1  x2   x  x1  x2  0 , torna-se verdadeira, isto é, x1 e x2 são os valores

procurados, ou melhor, são as raízes da equação do 2º grau.


A partir dessa equação, ainda verificamos que, se x1 e x2 são as raízes da
b
equação, então, constatamos que o termo é oposto da soma das raízes e que o
a
c
termo é o produto das raízes. Ou seja:
a

b
x1  x2  
a
e
c
x1  x2  .
a

2
Exemplo 15: A equação do 2º grau x  5 x  6  0 pode ser resolvida da se-
guinte maneira:

Note que, para essa equação, temos: a  1, b  1 e c = 6 . Logo, sabemos que a

soma das duas raízes da equação se dá por:


b
x1  x2  
a
5
 x1  x2  
1
 x1  x2  5

24
Além disso, sabemos que o produto das duas raízes se dá por:

UNICESUMAR
c
x1  x2 
a
6
 x1  x2 
1

Agora, precisamos saber quais são os dois valores cuja soma é 5 e o produto é
6 . Logo, constatamos que esses números são x1 = 2 e x2 = 3 , os quais são as
raízes da equação.

Observação: note que podemos reescrever a equação, em termos de suas raízes,


da seguinte maneira:
x2  5 x  6  0   x  2    x  3  0

pensando juntos

Você já percebeu que muitos recorrem à fórmula para determinar o conjunto solu-
ção da equação do 2º grau, mesmo que exista o processo de fatoração. O que leva
a essa escolha?

25
3
SISTEMAS DE
UNIDADE 1

EQUAÇÕES

Um sistema de equações é formado por um conjunto de equações que apresentam


mais de uma incógnita. Se possuirmos 2 incógnitas, precisamos de duas equações;
se possuirmos 3 incógnitas, precisamos de 3 equações e, assim, sucessivamente.
Para resolver um sistema, é necessário encontrar valores que satisfaçam si-
multaneamente todas as equações. Para tanto, existem algumas maneiras de se
resolver um sistema de equações, este assunto veremos adiante.
Dizemos que um sistema é do 1º grau, quando o maior expoente das in-
cógnitas é igual a 1 e não existe multiplicação entre essas variáveis. O conjunto
solução para um sistema de equações é dado por um par de valores que satisfaça
as equações simultaneamente.

Método da substituição

Para esse método, primeiramente, resolvemos uma das equações em termo de


uma das variáveis. Por exemplo, se uma das equações for 3 x  y  2 , fazemos
y  2  3 x. Então, substituímos essa expressão na segunda equação, obtendo uma
equação somente dependendo da variável x . Após resolvermos essa equação e
encontrarmos o valor da variável x , retornamos a uma das equações para en-
contrarmos o valor da variável y .
26
UNICESUMAR
3 x  2 y  16
Exemplo 16: Resolva o sistema  .
7 x  y  19

Pelo fato de que, na segunda equação, a variável y não é multiplicada por ne-
nhum valor, vamos isolá-la na equação, obtendo:

7 x  y  19  y  19  7 x

Substituiremos esse valor da variável y na primeira equação. Assim, obtemos:

3 x  2 y  16  3 x  2  19  7 x   16
 3 x  38  14 x  16
 11x  22
 x2

Agora que encontramos o valor de x , vamos substituí-lo em qualquer uma das


equações, a fim de encontrarmos o valor de y . Substituindo em y  19  7 x , temos:

y  19  7 x  y  19  7  2
 y  19  14
 y5

Logo, constatamos que a solução para esse sistema de equações é  2, 5  , isto


é, x = 2 e y = 5 .

Método da adição

Basicamente, o método da adição consiste em somarmos as variáveis semelhantes de


ambas as equações, com o intuito de obtermos um resultado igual a zero. Em outras
palavras, realizando uma adição (ou subtração) entre as equações, a ideia é que uma das
variáveis seja “cancelada”, pois, assim, podemos encontrar o valor da segunda variável.

27
UNIDADE 1

 x  2 y  17
Exemplo 17: Resolva o sistema dado por  .
 x  2 y  11

Note que, se somarmos as duas equações, o termo com a variável y se cancelará,


restando somente a variável x . Assim, temos:

 x  2 y  17
 
 x  2 y  11

2x  0 y  6

Agora, podemos encontrar o valor da variável x :

2x  0 y  6  2x  6
6
x
2
 x3

Nesse momento, realizamos o mesmo procedimento do exemplo anterior: subs-


tituímos o resultado obtido em uma das equações, a fim de obtermos o valor da
variável y :

x  2 y  17  3  2 y  17
 2 y  17  3
 2 y  14
14
y
2
 y 7

Logo, constatamos que a solução para esse sistema de equações é dada por
 3, 7  , isto é, x = 3 e y = 7 .

28
UNICESUMAR
4 x  3 y  2
Exemplo 18: Resolva o sistema  .
8 x  2 y  12
Analisando o sistema, não conseguimos realizar uma soma que, diretamente,
elimine uma das variáveis. Então, o que podemos fazer é multiplicar uma das
equações por um valor que resulte em algo desejável. Nesse caso, multiplicaremos
a primeira equação por −2 , obtendo:

4 x  3 y  2   2  8 x  6 y  4
 
8 x  2 y  12 8 x  2 y  12

Agora, podemos realizar a soma entre as duas equações. Assim, obtemos:


8 x  6 y  4

8 x  2 y  12

0  8 y  16
A partir disso, podemos encontrar o valor de y :
 8 y  16
16
y
8
 y  2

Substituindo esse valor em qualquer uma das equações originais, temos:


4 x  3 y  2  4 x  3   2   2
 4 x  6  2
 4x  4
4
x
4
 x 1
Portanto, o conjunto solução desse sistema de equações é 1, 2  ou x = 1
e y  2 .

29
Exemplo 19: A diferença de idade entre dois primos é de 4 anos e o produto de
UNIDADE 1

suas idades é 221 . Qual é a idade de cada um?

Vamos denotar a idade dos primos x e y . Pelo fato de que a diferença entre eles
é 4 e o produto é 221 , temos o seguinte sistema:

x  y  4

 x  y  221

Isolando a variável x na primeira equação, temos:

x y 4  x  4 y

Agora, substituindo-a na segunda equação, temos:

x  y  221   4  y   y  221
 4 y  y 2  221
 y 2  4 y  221  0
Resolvendo essa equação por meio da Equação de Bháskara, obtemos:

b  b2  4.a.c
y
2a
4  16  4  1  (221)
y
2 1
4  900
y
2
 4  30
 y1  2  13

 y  4  30  17
 2 2

Pelo fato de que estamos tratando de idades, consideraremos y = 13 . Agora, subs-


tituindo o valor na primeira equação, temos:
30
x y 4  x  13  4

UNICESUMAR
 x  17

Logo, constatamos que as idades dos primos são 13 e 17 anos.

explorando Ideias

O primeiro indício do uso de equações está relacionado, aproximadamente, ao ano de 1650


a.C., no documento denominado Papiro de Rhind, adquirido por Alexander Henry Rhind, na
cidade de Luxor - Egito, em 1858. O papiro de Rhind também recebe o nome de Ahmes, um
escriba que relata no papiro a solução de problemas relacionados à Matemática.
Fonte: Silva ([2020], on-line)¹.

anotações



















31
4
UNIDADE 1

INEQUAÇÕES

Às vezes, deparamo-nos com a situação em que precisamos pensar se o dinhei-


ro que temos seria, ou não, suficiente para comprar certa quantidade de pro-
dutos. Essa situação, sem dúvida, é um dos exemplos das inequações, pelo fato
de envolver polinômios e desigualdades. Assim, podemos definir as inequações
como expressões matemáticas que envolvem polinômios e desigualdades como
, ,  ou ≤ .
Caro(a) aluno(a), a seguir, estudaremos as inequações polinomiais do 1º e
do 2º grau.

Inequação do 1º grau

Para determinarmos o conjunto solução das inequações polinomiais do 1º grau,


em muitos casos, procedemos de forma análoga à das equações.

Exemplo 20: Para determinarmos o conjunto solução da inequação, definida


por 3 x  2  11 , procedemos da seguinte forma:

32
3 x  2  (2)  11  (2)

UNICESUMAR
 3x  9
1 1
 3x     9   
3 3
 x3

Assim, S   x   | x  3 .

Exemplo 21: para determinarmos o conjunto solução da inequação definida por


4 x  7  6 x  15 , procedemos da seguinte maneira:

4 x  7   6 x   6 x  15   6 x 
 2 x  7   7   15   7 
 2 x  8
 2 x  8   1
 2 x   1  8   1
 2 x  8
1 1
 2 x     8   
2 2
 x  4

Assim, S   x   | x  4

Observação: ao multiplicar ambos os membros de uma inequação por  1 ,


invertemos o sentido do sinal da desigualdade.

Inequação do 2º grau

Uma inequação do 2º grau na incógnita x é uma expressão que pode ser escrita
em uma das seguintes formas:

33
ax2  bx  c  0
UNIDADE 1

ax2  bx  c  0
ax2  bx  c  0
ax2  bx  c  0
Para resolvermos uma inequação do 2º grau, devemos, inicialmente, estudar o
sinal da expressão correspondente à equação obtida, trocando-se a desigualda-
de pela igualdade. Ao localizarmos as raízes da equação, estudamos o sinal da
equação correspondente.

Veremos dois casos de inequações do 2º grau.

Inequação-produto

Considere a seguinte inequação do segundo grau:


x2  2 x  8  0

Fatorando o primeiro termo da desigualdade, temos:


 x  2   x  4  0
Assim, transformamos um polinômio de grau 2 em um produto de polinô-
mios de grau 1 . Como sabemos que o produto de dois termos é positivo quando
ambos são positivos ou quando ambos são negativos, constatamos que a solução
dessa inequação-produto é o conjunto de todos os x reais, tais que:

 x  2  0 e  x  4  0
ou
 x  2  0 e  x  4  0 .

Para o primeiro caso, temos x  2  0  x  2 e x  4  0  x  4 . Anali-


sando esses intervalos, temos:

34
UNICESUMAR
x ≥ 2

x  4

----------------------------------------------------------------------------------------

x  2 e x  4

Portanto, para essa opção, temos x ≥ 2 .

Por outro lado, para a segunda opção, temos x  2  0  x  2 e


x  4  0  x  4 . Analisando esses intervalos, temos:

x≤2

x  4

----------------------------------------------------------------------------------------

x  2 e x  4

Portanto, para essa opção, temos x  4 . Logo, o conjunto solução para a inequa-
ção x  2 x  8  0 é S   x   | x  2 ou x  4 ou S  (, 4]  [2, ) .
2

Observação: note que realizamos, primeiramente, a interseção entre os inter-


valos de uma mesma condição   0 ou  0  , para, posteriormente, unirmos
os resultados.

35
Inequação-quociente
UNIDADE 1

Observe que as seguintes inequações apresentam um quociente de polinômios


de 1º grau:
x 1
 0, x 1  0
x 1
x 1
 0, 3 x  2  0
3x  2
2x 1
 0, x  4  0
x4
2x  3
 0, x  2  0
x2

A maneira de resolvermos esse tipo de inequação é semelhante ao visto


nas inequações-produto. Isto é, analisamos os sinais das equações polinomiais
de 1º grau envolvidas e, depois, analisamos o sinal do produto ou quociente,
lembrando as regras de sinais para números reais.

Exemplo 22: Determine os valores de x ∈ , tal que:

2
x 1  .
x2

Primeiramente, observamos que x ≠ 2 , para que a expressão faça sentido.

Temos duas opções para o termo x + 2 :

I - Se x  2  0 , segue que  x  1   x  2   2 .

II - Se x  2  0 , segue que  x  1   x  2   2 .

2
Para (i), temos x  3 x  2  2  0 , ou, ainda,
x2  3 x  0

Isto é:
x   x  3  0 .
36
Observe que as raízes da equação são: x = 0 e x  3 , e a inequação é satis-

UNICESUMAR
feita para x  3 e x > 0 . Analisando os intervalos, temos

x  3

x>0

---------------------------------------------------------------------------------------

x  3 e x  0

Além disso, a condição do item (I), de que x  2  0  x  2 , implica


que a solução para o item (I) é x > 0 ou  0,   .
Para o item (II), segue da mesma forma que:

x   x  3  0

A inequação, nesse caso, é satisfeita para 3  x  0 . Entretanto a condição


do item (ii) indica que devemos ter x  2 . Assim, constatamos que a solução
para esse item é:

3  x  2 ou  3, 2  .

Logo, a solução da inequação dada é a união entre as duas soluções:

 3, 2    0,   .

Observação: para o caso de uma inequação-produto ou inequação-quociente


na qual o resultado desejado é  ou  , devemos nos recordar que os sinais dos
termos devem ser opostos para que o produto entre dois termos seja negativo.
Em outras palavras, é necessário analisar os casos em que um termo é positivo
e o outro é negativo, e vice-versa, para, depois, realizar a união dos resultados.

37
CONSIDERAÇÕES FINAIS
UNIDADE 1

Caro(a) aluno(a), chegamos ao final de nossa primeira unidade. Nós acabamos


de estudar partes importantes e fundamentais para todo o decorrer do curso:
os polinômios, as equações (e seus sistemas) e as inequações. Algumas pessoas
não gostam dos polinômios por considerá-los abstratos. No entanto dão suporte
aos demais conteúdos, tendo em vista que os polinômios estão envolvidos nas
equações e nas inequações.
Além disso, alguns confundem e/ou não diferenciam a expressão “resolver
equações” de “efetuar operações matemáticas”. Entretanto ao resolvermos uma
equação, estamos determinando o(s) valor(es) da variável, que, na maioria
dos casos, é representada pela letra x , a fim de que a igualdade se torne ver-
dadeira. Para tanto, recorremos ao processo que consiste em isolar a variável
dos valores numéricos.
Para os primeiros exemplos que estiver realizando e ao passar para seus alu-
nos, é interessante que você indique quais operações são inseridas para que se
consiga isolar a variável, seja uma multiplicação, seja uma adição/subtração de
um termo. Depois de muita prática e de já compreender a importância desse pas-
so, as indicações não são mais necessárias e as resoluções se tornam mais diretas.
Os sistemas de equações apresentam uma variedade imensa de aplicações
no dia a dia. Muitos dos problemas enfrentados por todos utilizam duas ou mais
variáveis e, por isso, saber resolver sistemas de equações se torna fundamental.
Resolver inequações de 2º grau pode parecer trabalhoso, mas com bastante
prática, resolução de vários exemplos e dedicação, essa atividade se torna algo
tranquilo, pois somente é necessário analisar as possibilidades para o sinal de
cada uma das equações de 1º grau e, depois, unir os resultados.
Caro(a) aluno(a), você deve ter notado que várias demonstrações foram apre-
sentadas no decorrer desta unidade, tendo em vista que as demonstrações são
tão importantes quanto à contextualização. Esperamos que você tenha gostado
dessa forma de apresentação de conteúdo. Até a próxima unidade!

38
na prática

1. Analise as afirmativas, a seguir, e julgue com (V) para as Verdadeiras e (F) para as Falsas:

( ) O grau da soma de dois polinômios sempre será igual ao maior grau dos poli-
nômios envolvidos nessa adição.
( ) O grau da diferença entre dois polinômios sempre será igual ao menor grau
verificado entre os polinômios envolvidos nessa subtração.
( ) O grau do produto de dois polinômios sempre será igual à soma dos graus de
seus fatores.
( ) O grau do quociente de um polinômio por outro sempre será igual à diferença
entre o grau de dividendo e do divisor.

A sequência correta é:

a) V, V, V, V.
b) V, F, F, F.
c) F, V, V, V.
d) F, F, V, V.
e) F, F, F, F.

2. Analise as afirmativas, a seguir, e julgue com (V) para as Verdadeiras e (F) para as Falsas:

( ) O conjunto solução de uma equação polinomial de 2º grau apresenta até dois


valores reais distintos.
( ) O conjunto solução da equação polinomial de 2º grau sempre apresenta um ou
dois valores reais distintos.
( ) Para determinar as raízes de uma equação polinomial de 2º grau, deve-se igualar
a equação por 0.
( ) O conjunto solução de uma inequação polinomial de 2º grau sempre será um
intervalo ou conjunto vazio.

A sequência correta é:

a) V, V, V, V.
b) V, F, V, V.
c) F, F, F, V.
d) F, F, V, F.
e) F, F, F, F.

39
na prática

3. Quanto às equações, analise as afirmativas a seguir:

I - Uma equação do 1º grau é um polinômio de 1º grau cujo maior expoente da


variável é 1.
II - Uma equação do 2º grau do tipo não ad-
mite raízes opostas.
III - O método de fatoração permite resolver todos os tipos de equações do 2º grau,
mesmo aquelas que não admitem raízes reais.

É correto afirmar que:

a) Apenas, a afirmativa I está correta.


b) Apenas, a afirmativa II está correta.
c) Apenas, a afirmativa III está correta.
d) Apenas, as afirmativas I e II estão corretas.
e) Apenas, as afirmativas I e III estão corretas.

4. Uma pessoa aumentou igualmente os lados de um jardim retangular de 5 metros


por 3 metros e sua área passou para 35 m2. Cada lado desse jardim foi aumentado
em quantos metros?

5. Uma companhia de táxis cobra R$4, 00 de taxa fixa mais R$0, 75 para cada qui-
lômetro rodado. Um cliente fez uma corrida com essa companhia de táxis e pagou
R$50, 00 . Qual é a equação que representa esse valor?

6. Um cinema cobra R$18, 00 a entrada inteira e R$9, 00 a meia-entrada. Para uma


seção de lançamento de um novo filme, foram vendidos 380 ingressos, arrecadan-
do um total de R$5985, 00 . Determine quantos ingressos de cada tipo foi vendido.

40
na prática

7. João quer construir uma cerca retangular em sua fazenda. Ele deseja que a cerca
tenha, pelo menos, 180 m de perímetro. Se a cerca tem 20 m de largura, quantos
metros a cerca deve ter de comprimento?

8. Um objeto é lançado para cima com uma velocidade dada pela equação V  80  32t ,
em que t é dado em segundos e a velocidade em metros por segundo. Quando a velo-
cidade desse objeto estará entre 32 e 64 metros por segundo?

9. A Secretaria de Saúde de uma cidade realiza a vacinação contra Pólio e Sarampo.


Cada vacina de Pólio é constituída por 4 doses e cada vacina de Sarampo é cons-
tituída por 2 doses. Ano passado, a Secretaria de Saúde realizou 60 vacinações, o
que consistiu em um total de 184 doses. Quantas vacinas de Pólio e quantas vacinas
de Sarampo a Secretaria realizou no ano passado?

10. Encontre o conjunto solução da inequação  3 x  6    5 x  7   0 .

41
aprimore-se

De acordo com Miguel (1993), Clairaut, em 1741, demonstrava interesse em utilizar


a História da Matemática no ensino e na aprendizagem da matemática. Contudo,
existem poucos materiais instrucionais que podem auxiliar os professores a utiliza-
rem a História da Matemática como um recurso metodológico de ensino (BROLEZ-
ZI, 1991; MENDES, 2006; MIGUEL, 1993). Apesar da dificuldade de utilização desses
materiais, os resultados de estudos recentes mostram que a História da Matemática
pode ser utilizada de duas maneiras distintas no ensino e na aprendizagem da ma-
temática, ou seja, explícita e implícita (FERREIRA; RICH, 2001 apud DAMBROS, 2006).
Nesse sentido, a utilização da História da Matemática, de uma maneira implícita,
pode funcionar como um eixo orientador para auxiliar os professores a entenderem
algumas dificuldades dos alunos, que estão relacionadas com o ensino de um deter-
minado conteúdo matemático, por exemplo, funções. Entendemos que essa abor-
dagem pode resultar em uma antecipação dessas dificuldades pelos professores.
Em outra perspectiva, podemos empregar a História da Matemática, de uma ma-
neira explícita, por meio da utilização de situações-problema que ocorreram no de-
correr da história. Dessa maneira, podemos nos apropriar desses problemas para
oferecer situações históricas semelhantes àquelas ocorridas na história, porém,
adaptadas para outros contextos socioculturais. Então, a História da Matemática
funciona como um pano de fundo na preparação das aulas, pois não há uma preo-
cupação com o detalhamento da história do conteúdo matemático a ser estudado.

42
aprimore-se

Por exemplo, comparando os procedimentos utilizados na antiguidade para o


trabalho com os números negativos, percebe-se que esses números apresenta-
ram muitos obstáculos de entendimento pelas civilizações daquela época. Nesse
sentido, as civilizações europeias demoraram para aceitar os números negativos,
enquanto que a civilização chinesa utilizou barras pretas para representá-los e bar-
ras vermelhas para representar os números positivos, facilitando, dessa maneira, o
trabalho operacional com esses números. Assim, historicamente, os números nega-
tivos tiveram impactos diferentes em culturas distintas, pois o seu desenvolvimento
dependeu do contexto local e das ideias matemáticas que foram desenvolvidas em
grupos culturais específicos (RADFORD, 1997).
Então, é de suma importância que percebamos a influência dos aspectos cultu-
rais no desenvolvimento do conhecimento matemático, pois existe a necessidade
de que a cultura seja considerada como um fator relevante para o processo de ela-
boração de atividades matemáticas baseadas na perspectiva Sociocultural da Histó-
ria da Matemática.

Fonte: Oliveira, Viana e Rosa (2013).

43
eu recomendo!

livro

Fundamentos de matemática elementar: conjuntos, funções


Autor: Gelson Iezzi e Carlos Murakami
Editora: Atual
Sinopse: o livro é uma coleção consagrada, ao longo dos anos, por
oferecer ao estudante o mais completo conteúdo de matemática
elementar. Este volume apresenta os seguintes tópicos: conjun-
tos e funções. Além disso, é composto por teoria e exercícios de
aplicação, testes de vestibulares atualizados, selecionados criteriosamente e or-
denados por grau de dificuldade, acompanhados das respostas correspondentes.
Há, ainda, uma série de artigos sobre história da matemática, relacionados aos
temas abordados.

conecte-se

Como resolver inequação do 2º grau


É uma ótima videoaula que mostra a forma de resolver inequações do 2º grau.
https://www.youtube.com/watch?v=4nWOEQns2iw

44
2
FRAÇÕES
ALGÉBRICAS

PROFESSOR
Me. Tiago Peres da Silva Suguiura

PLANO DE ESTUDO
A seguir, apresentam-se as aulas que você estudará nesta unidade: • Expressões algébricas em forma
de fração • Expressões algébricas • Equações fracionárias • Sistemas com equações fracionárias

OBJETIVOS DE APRENDIZAGEM
• Conceituar variáveis e expressão algébrica • Definir, exemplificar e operar expressões algébricas •
Identificar o conceito de equação algébrica • Resolver sistemas e problemas com equações algébricas.
INTRODUÇÃO

Caro(a) aluno(a)! É notória a importância da matemática no dia a dia das


pessoas, pois ela está presente em muitos lugares, desde a contagem de
moedas para se comprar uma dúzia de pães franceses, até na escolha das
roupas que usaremos, já que devemos relacionar a roupa com o local, a
temperatura e as combinações que podemos fazer. Tudo isso é fazer ma-
temática no dia a dia.
Obviamente, nem toda a matemática pode ser descrita de forma algé-
brica. Algumas vezes, basta utilizarmos o raciocínio lógico para se resolver
um problema, como uma coleção de informações essenciais que servem de
base para um raciocínio (premissas) e as quais, se estudadas, levam-nos a
uma conclusão, por exemplo. Entretanto, é bom saber que nem toda cole-
ção de premissas nos leva a uma conclusão.
Em nossa primeira unidade, apresentamos o conceito de equações po-
linomiais, mas quem garante que essas expressões matemáticas garantem o
estudo de todas as situações possíveis? Por exemplo, quando uma incógnita
se encontra presente no denominador de uma fração, será que podemos
ter uma solução? A resposta é sim, pois, em nossa segunda unidade, apre-
sentaremos o conceito de equações fracionárias ou frações algébricas, ou
seja, aquelas que apresentam incógnita no denominador.
Trata-se de um tema importante e relevante, pois corresponde a um tó-
pico que fará parte de sua vida profissional, como professor de Matemática
no Ensino Fundamental e, algumas vezes, até no Ensino Médio ou, ainda,
no Ensino Superior, nas disciplinas de exatas.
Assim, cabe a você, enquanto futuro educador, dedicar-se à identifi-
cação de frações algébricas, assim como a sua simplificação, na adição, na
subtração e por que não na multiplicação dessas expressões, além estender
esse conceito para sistemas de equações fracionarias, pois também é uma
possibilidade que apresentaremos nesta unidade. Portanto, explore, inves-
tigue e dedique-se. Um forte abraço e mãos à obra!
1
EXPRESSÕES ALGÉBRICAS EM

UNICESUMAR
FORMA DE FRAÇÃO

Antes de começarmos a falar sobre o conteúdo de nossa unidade, precisamos


relembrar alguns conceitos importantes. Vamos lá?

Definição: denomina-se variável a letra que representará qualquer número ou


um conjunto de números, ou seja, é um símbolo representativo (incógnita) capaz
de representar o número de um conjunto.
Ao expandirmos esse conceito, misturamos as variáveis com outros números
e operações aritméticas, como 2 x  2  ( x  3) , por exemplo, e acabamos repre-
sentando uma expressão. Tais expressões são chamadas expressões algébricas.
Note que, nessa expressão, temos uma variável, x , números conhecidos, 2 e 3 ,
e operações aritméticas também conhecidas, a soma e a multiplicação.

47
2
EXPRESSÕES
UNIDADE 2

ALGÉBRICAS

Agora que já retomamos o conceito de expressão algébrica, analisaremos um caso


específico, quando as expressões envolvem frações. Vejamos o seguinte exemplo:
1 a −b x a⋅x
Exemplo 1: , , 2 , .
x b − 2 x  2  x  y  y2 2 ⋅ b ⋅ y
Perceba que, em todos os casos, temos números conhecidos, variáveis cujo valor
não conhecemos (uma ou mais variáveis) e operações aritméticas. É comum,
também, o fato de que todos os casos apresentam variáveis no denominador das
frações. Por isso, essas expressões recebem o nome de frações algébricas.
Algo muito importante e que deve ser destacado é: nunca podemos ter o
valor zero no denominador de uma fração. Pelo fato de que temos variáveis nos
denominadores de nossas frações, precisamos estar atentos aos valores que essas
variáveis podem assumir. Por exemplo, na fração algébrica 1 x, sabemos direta-
mente que x ≠ 0 , pois x é o único elemento do denominador e, portanto, deve
ser diferente de zero. Vejamos outros exemplos.

Exemplo 2:
x
I - . Nesse caso, a expressão no denominador é x −1 . Precisamos en-
x −1
48
contrar qual é a restrição que deve acontecer:

UNICESUMAR
x 1  0 + 1
x  1  1  0  1
x 1
Logo, a restrição é que x deve ser diferente de 1 .

n
II - . Aqui, a expressão no denominador é m − 3n . Isto é, possuímos
m − 3n
duas variáveis distintas. Isso demonstra que a restrição de uma das va-
riáveis está ligada à restrição da outra. Novamente, sabemos que o de-
nominador deve ser diferente de zero. Assim, temos:
m  3n  0 +  3n 
m  3n   3n   0   3n 
m  3n
Logo, a restrição é m ≠ 3n . Por exemplo, se n = 1 , sabemos que m não
pode assumir o valor 3 .
y 2
III - 2
. Nesse exemplo, a expressão no denominador é x − 1 , a qual é
x −1
uma expressão do segundo grau. Podemos reescrever essa expressão,

utilizando produto notável, como x  1   x  1  x  1 . Assim, a res-


2

trição de nosso denominador deve ser:


 x  1   x  1  0
  x  1  0 e  x  1  0
 x 1 e x  1
Ou seja, temos duas restrições para nossa variável: x não pode ser 1 e
não pode ser −1 .

49
UNIDADE 2

pensando juntos

Você sabia que as frações algébricas estão muito presentes em nosso dia a dia? Imagine
que você está abastecendo R$100, 00 de combustível e que o preço de um litro de com-
bustível é x reais: assim, seu carro receberá 100 x litro de combustível. Essa é uma conta
que fazemos constantemente e nem percebemos!

Simplificação de frações algébricas

Quando queremos realizar a simplificação de uma fração, dividimos tanto o nu-


merador quanto o denominador pelo mesmo número diferente de zero. Isso equi-
vale a “cancelar” os fatores em comum, obtendo uma fração mais simples, como:

15 3  5 3 42 2  3  7 1
   2 2 
20 2  2  5 4 252 2  3  7 6

Com o mesmo procedimento em mente, podemos realizar simplificações de


frações algébricas, quando há algum fator em comum, não nulo, no numerador
e no denominador.

Exemplo 3:

4  x  y2 2  2  x  y  y 2  y
I -  
6  x2  y 2  3  x  x  y 3  x

x2  y 2
II - . Note que o numerador é uma diferença entre dois quadrados,
x y
logo, podemos reescrever como x  y   x  y    x  y . Assim, temos:
2 2

50
 x  y x  y  x  y

UNICESUMAR
x y

8b − 4b2
III - . Constatamos que 4b é um fator comum das parcelas do nu-
2b
merador, portanto, podemos colocá-lo em evidência e simplificar:

8b  4b2
2
4 b  2  b 

2b 2 b
2 2  b 

1
 4  2b

Adição e subtração de frações algébricas

As operações de adição e subtração com frações algébricas são realizadas da


mesma maneira que adicionamos ou subtraímos números na forma fracionária:

■ Obtemos frações equivalentes e de mesmo denominador.


■ O denominador comum poderá ser o produto ou o MMC dos denominadores.

Exemplo 4:
1 3 2
i -   . Vamos realizar essa operação, usando o produto dos deno-
2x 4 y 3
minadores:
1 3 2 12  y  18 x  16  x  y
  
2x 4 y 3 24  x  y
2   6  y  9  x  8  x  y 

2  12  x  y
6 y  9 x  8 x y

12  x  y

51
UNIDADE 2

1 x
ii -  2 . Vamos realizar essa operação, utilizando o MMC:
x  y x  y2
1
 2
x

 x  y  x
x  y x  y2  x  y    x  y 
2x  y

 x  y x  y
2x  y

x2  y 2

Observação: note que x  y   x  y    x  y  , ou seja, os denominadores


2 2

x − y e  x  y    x  y  possuem um termo em comum. Isso quer dizer que


mmc  x  y,  x  y   x  y     x  y    x  y  . Em outras palavras, quando
queremos calcular o MMC entre duas expressões algébricas e ambas possuem
um termo em comum, não é necessário realizar o produto entre elas.

Multiplicação de frações algébricas

A operação de multiplicação, envolvendo frações algébricas, ocorre do


mesmo modo como multiplicamos números em forma de fração. Vejamos
alguns exemplos:

Exemplo 5:
3 x 8  y 2 3  x  2 8  y  y 6 y
I -   
4 y 7  x3 4  y  7  x  x  x 7  x2

6 x y x  y 6 x y x y
3
6  x  y   x  y  3y
II - 2     
x  y 2 2  x  x  y    x  y  2  x  x  y    x  y   2  x x  y

Observação: é necessário sempre estar atento às restrições. Para o exemplo I),


sabemos que x, y ≠ 0 . Para o exemplo II), sabemos que:

52
x2  y 2  0  x2  y 2

UNICESUMAR
 x2  y2
 x  y
Além disso, x ≠ 0 .

Divisão de frações algébricas

Da mesma maneira que ocorre com as outras operações, a divisão de frações


algébricas também se dá de modo similar: multiplica-se a primeira fração pelo
inverso da segunda fração.

Exemplo 6:
4 xy 4 x 4 xy  5 z y5
I - :   
z2 5z z 2 4 x z
x2  4 x2  2 x x2  4 2  y  1  x  2   x  2    2  y  1 x  2
II - :     2
x  2  x  y 2  y  1 x  2  x  y x2  2 x x 1  2  y   x  x  2  x

4 xy
4 xy 4 x 2
Observação: lembre-se que 2 : = z .
z 5z 4x
5z

Potenciação de frações algébricas

Assim como nos casos anteriores, o cálculo de potências de frações algébricas


ocorre da mesma maneira. Vejamos alguns exemplos:
2 2
 2x 

 2 x   4 x2
I -  
 3y   3 y 2 9 y2
2
 ab
2

1

 3x 
2

3x   9 x2
II -   2  
 3x   ab  ab  a  b 2 a2  2ab  b2
 
 3x 
53
3
EQUAÇÕES
UNIDADE 2

FRACIONÁRIAS

Denominamos equação fracionária toda fração que possui, ao menos, uma va-
riável no denominador:
45
I - =4
3x

23( x  3)
II - 0
4x

Resolução de equações fracionárias


3 2 1
I -   , x  *
4 x 3

54
3 2 1
Reduzimos ao mesmo denominador.

UNICESUMAR
4 x 3
9 x 24 4x
Realizamos a soma do lado esquerdo.
12 x 12 x 12 x
9 x 24 4 x
Eliminamos o denominador.
12 x 12 x
9 x 24 4 x
9x 4x 24
5x 24
24
x
5

x 1 x2  1
II -   2 , para x  3 e x  3
x 3 x 3 x 9

x 1 x2 1
Reduzir ao mesmo denominador (mmc).
x 3 x 3 x2 9
x x 3 x 3 x2 1
Eliminar o denominador.
x 3 x 3 x 3 x 3 x 3 x 3
x x 3 x 3 x2 1

x2 3 x x2 x 2
x2 x2 3 x x 2
2x 2
2
x
2
x 1

55
4
SISTEMA COM EQUAÇÕES
UNIDADE 2

FRACIONÁRIAS

Neste tópico, apresentaremos um caso particular dos sistemas de equações que


estudamos, na Unidade 1, deste livro. Aqui, as equações que aparecem no sistema
são equações fracionárias, isto é, possuem pelo menos uma variável no denomi-
nador. Vejamos, por meio de exemplos, como devemos solucionar um sistema
de equações fracionárias:

Exemplo 7:

2x 21
 y 3  5

I -  .
 1 2 5

 x y  1 6 x

O primeiro passo é impor as restrições: para a primeira equação, temos y ≠ 0 ,


enquanto, para a segunda equação, temos x ≠ 0, y ≠ 1 . Isto é, para o nosso siste-
ma, as restrições são x ≠ 0, y ≠ 0, y ≠ 1 .
Agora, é necessário reescrever o sistema, de forma que as equações estejam
na forma ax  by  c . Em outras palavras, devemos reduzir o denominador a
um termo comum e realizar a simplificação:
56
2 x 3 21 1 2 5
 

UNICESUMAR
 
y 1 5 x y 1 6x
10 x 15 y 21 y 1  6  y  1 2  6x 5   y  1
     
5y 5y 5y 6 x  y  1 6 x  y  1 6 x  y  1
 10 x  15 y  21 y
 6  y  1  12 x  5  y  1
 10 x  15 y  21 y  0
 6 y  6  12 x  5 y  5
 10 x  6 y  0
 12 x  6 y  5 y  5  6
 12 x  y  1

10 x  6 y  0
Agora, resolvemos o sistema resultante:  .
12 x  y  1

10 x  6 y  0 10 x  6 y  0

12 x  y  1  6  3
 10  6y  0
10 x  6 y  0 41
 30
 72 x  6 y  6   6y
41
82 x  6
30
6 y
x 246
82 5
3 y
x 41
41

3 5 5
Precisamos, então, verificar as restrições. Assim, como ≠ 0, ≠ 0 e ≠1,
41 41 41
3 5
a solução do sistema é o par ordenado  ,  .
 41 41 

 x  y  60

II -  5 9 8 , sabendo que x  y  675 .
 x  y  15


57
Note que as restrições são x ≠ 0 e y ≠ 0 . Assim, reduziremos a equação que
UNIDADE 2

possui uma fração algébrica a um denominador comum:


5 9 8
 
x y 15
75 y 135 x 8 xy
  
15 xy 15 xy 15 xy
 75 y  135 x  8 xy
xy 675
 135 x  75 y  8  675
 135 x  75 y  5400  15 
 9 x  5 y  360

 x  y  60
Isso resulta, então, no sistema dado por  . Vamos resolvê-lo pelo
9 x  5 y  360
método da substituição:

9 x  5 y  360
x  y  60 y  60  x
 9 x  5  60  x   360
 y  60  x  y  60  15
 9 x  300  5 x  360
 y  45
 4 x  60
 x  15

explorando Ideias

No século XVI, o astrônomo polonês Nicolaus Copernicus trocou a visão tradicional do


movimento planetário centrado na Terra por um em que o Sol estava no centro e os pla-
netas giravam em torno dele, em órbitas circulares. Embora o modelo de Copérnico esti-
vesse muito próximo de predizer o movimento planetário corretamente, existiam discre-
pâncias. Todavia o problema foi resolvido pelo matemático alemão Johannes Kepler, que
descobriu que as órbitas planetárias não eram círculos, mas elipses. Kepler descreveu o
movimento planetário por três leis, sendo que uma delas pode ser representada por uma
equação fracionária, evidenciando, assim, uma constante k , conhecida como a constante
de Kepler, a qual só depende da massa do Sol.
58
T2

UNICESUMAR
T 2  kr 3 ou k
r3
As leis de Kepler não se aplicam somente aos planetas orbitando o sol, mas a todos os
casos em que um corpo celestial orbita outro sob a influência da gravitação e, até mesmo,
estrelas orbitando outras estrelas.
Fonte: o autor.

CONSIDERAÇÕES FINAIS

Caro(a) aluno(a)! Chegamos ao fim de mais uma unidade, a segunda de um total


de cinco, em que conceitos importantes foram apresentados e, talvez, até relem-
brados. Contudo, isso não demonstra que eles são menos importantes, pois, para
um bom desenvolvimento da matemática, é preciso ter uma base teórica sólida. É
como um ditado: uma casa construída sobre a areia não é resistente como uma casa
construída sobre a rocha. Assim, a sua dedicação é essencial para que isso aconteça.
Esta unidade foi importante para que você descobrisse ou relembrasse que as
equações polinomiais não se limitam apenas as apresentadas na unidade anterior,
pois também podemos ter incógnitas no denominador de uma fração e, por que
não, no expoente de um número. Esse conceito será apresentado (ou relembrado),
futuramente, em outras disciplinas.
Aqui, também foi possível notar que, mesmo mais complexas, as equações fracio-
narias também fazem parte de nosso cotidiano, podendo aparecer no dia a dia e sendo
importantes na execução de uma tarefa ou apresentação de um resultado. Portanto,
saber como elas funcionam e se aplicam é essencial para a nossa vida docente.
Outro ponto importante foi perceber que os sistemas de equações podem utili-
zar as equações fracionárias, com algumas restrições, é claro. Não só, mas que pode-
mos utilizar alguns métodos já vistos para resolvê-los e que novos métodos surgem
a partir disso, assim como é o caso do artifício de mudança de variáveis. Dessa
forma, torna-se importante saber como utilizá-los e em que momento aplicá-los.
Portanto, cabe agora a você, aluno(a), revisar os temas apresentados, refazer
os exemplos citados e treinar os exercícios propostos a seguir, pois, como um
bom matemático, também devemos treinar os nossos conhecimentos, por meio
da realização de exercícios, para fixação do conteúdo. Não se esqueça de assistir
às aulas conceituais, uma vez que se tratam de um importante recurso de estudo
e fixação da matéria. Está preparado(a) para a nossa próxima unidade? Espero
que sim! Um forte abraço e até a próxima.
59
na prática

1. Escreva cada uma das frações algébricas com as respectivas restrições ao denominador:

a) Numerador: dez.
Denominador: o dobro de um número real qualquer.
b) Numerador: o sucessor de um número natural qualquer.
Denominador: o triplo desse número.
c) Numerador: um número real qualquer.
Denominador: esse número aumentado em um.

2. Apresente a restrição que devemos fazer ao denominador, para que cada fração
algébrica represente um número real:

x
a) .
x −1
y
b) .
2x + 6
x
c) 2
.
x −1
x
d) .
x− y

3. Simplifique as frações algébricas:

x + xy
a) .
x + xz

a3
b) .
a 2

2 y 2 − 10 y
c) .
y −5

60
na prática

a 3  b3
d) .
a3  a2b  ab2

4. Por meio de frações algébricas, apresente o perímetro e a área de uma região re-
x 2
tangular cujo comprimento corresponde a e a largura a .
y y

5. Resolva as seguintes equações fracionárias:

3 2 3
a)    x  0 .
x 5 4
2 1
b) 2
  x  2; x  2  .
x 4 x2
1 2 3
c)  2   x  3; x  3 .
x 3 x 9 x 3

6. Uma torneira enche um tanque em 9 horas e outra enche o mesmo tanque em x


horas. Juntas, elas enchem o tanque em 4 horas. Descubra o número x de horas
que a segunda torneira demora para encher o tanque.

61
aprimore-se

NEM SÓ ÁLGEBRA, NEM SÓ ARITMÉTICA

Este artigo se inspira na linha de que se pode ensinar matemática, no primeiro grau,
por meio de dados simples, tirados de fatos da vida cotidiana, evitando que um sim-
bolismo exagerado leve à fuga do concreto e ameace tornar as aulas enfadonhas.
Acreditamos que, ao partir de situações concretas, impedimos que o aluno se
escravize às operações e às regras, estimulando-o a refletir sobre um problema, e
não somente sobre quais operações executar para resolvê-lo.
Nessa direção, apresentamos sugestão de novo enfoque para 5 problemas que,
nessa ou noutra versão, são comumente estudados em sala de aula. Tentaremos,
ainda, mostrar, nos exemplos, como um desenho da situação descrita em um pro-
blema pode ajudar na busca da solução.

Exemplo 1: Calcular dois números, dadas sua soma e diferença.

Sabendo que, para determinar o menor deles, basta dividir por 2 a diferença dos
números dados, o estudante poderá sair-se bem em um exame. Mas o que restará
quando a regra tiver sido esquecida?
Nossa sugestão é apresentar o problema numa situação concreta: Mário e Ro-
berto têm, juntos, 45 bolinhas. Mário tem 7 bolinhas a mais do que Roberto. Quan-
tas bolinhas tem cada um?
Pode-se encenar o problema dando a dois alunos da classe 45 objetos (bolinhas,
feijões, ou o que estiver ao alcance) e pedir que eles os dividam entre si, nas condi-
ções do problema. A classe toda será convidada a participar e todas as sugestões
serão analisadas. Eventualmente, a classe perceberá que dando, inicialmente, ao
Mário, as 7 bolinhas que ele possui a mais do que Roberto e, em seguida, repartindo
em partes iguais as bolinhas restantes, o problema estará resolvido.
Posteriormente, pode-se dar ao problema um tratamento mais abstrato: Se x for
o número de bolinhas de Roberto,
45  7
x  x  7  45  x   19 .
2

62
aprimore-se

O desenho pode ser um grande auxiliar no ensino de matemática, mesmo fora da


geometria. Quem já não viu o problema folclórico:

Exemplo 2: Um tijolo pesa um quilo mais meio tijolo. Quanto pesa um tijolo inteiro?

O seguinte desenho fala por si:

um tijolo meio
tijolo
1 kg

Se um quilo está no lugar de meio tijolo, meio tijolo pesa um quilo. Logo, o tijolo
pesa 2 quilos.
 x 
"Algebrizando": x  1   x  2  .
 2 

Observação final:

Durante muitos anos, no primeiro grau, predominavam as seguintes atitudes:


■ Até a 5.ª série, problemas eram resolvidos com o uso, apenas, da aritmética.
■ Da 6.ª série em diante, com a introdução da álgebra, os problemas passavam
a ser resolvidos, exclusivamente, por processos algébricos.

A nossa opinião é que o raciocínio aritmético (nos exemplos, apoiado por figuras)
deva continuar sendo cultivado, mesmo após a introdução da Álgebra, ou seja, no
primeiro grau, nem só álgebra, nem só aritmética.

Fonte: Viotto (1990).

63
aprimore-se

livro

Tudo é matemática - 8º ano


Autor: Luiz Roberto Dante
Editora: Ática
Sinopse: coleção que ensina matemática por meio de exemplos e
situações-problema inteligentes e criativos. Inicia-se, no primeiro
capítulo, revendo o que aprendemos, para uma eficaz revisão de
conteúdos estudados. A teoria é apresentada com destaque, por
meio de explicações sempre ligadas a exemplos do cotidiano dos alunos. Além
disso, atividades e problemas seguem as explicações para trabalhar imediata-
mente os conceitos. Todos os exercícios estão em rigorosa ordem de dificuldade:
do mais fácil ao mais complexo.

filme

O homem que viu o infinito


Ano: 2016
Sinopse: o filme retrata a história de Srinivasa Ramanujan, um
matemático indiano que fez importantes contribuições para o
mundo da matemática, para a teoria dos números, a série e fra-
ções contínuas.

64
anotações



































anotações



































3
TRIGONOMETRIA

PROFESSORES
Me. Issao Massago
Me. Tiago Peres da Silva Suguiura

PLANO DE ESTUDO
A seguir, apresentam-se as aulas que você estudará nesta unidade: • Triângulo retângulo e Teorema de
Pitágoras • Razões trigonométricas no triângulo retângulo • Ciclos trigonométricos • Leis trigonométricas

OBJETIVOS DE APRENDIZAGEM
• Identificar os triângulos retângulos e aplicar o Teorema de Pitágoras para resolver situações-problema
• Definir as principais razões trigonométricas no triângulo retângulo e resolver situações-problema que
as envolvem • Resolver situações-problema que envolvem o ciclo trigonométrico • Conhecer as leis dos
senos e dos cossenos, envolvendo triângulos quaisquer, e resolver situações-problema que as envolvem.
INTRODUÇÃO

Caro(a) aluno(a), bem-vindo(a)! Nesta unidade, estudaremos trigonome-


tria. Para tanto, iniciaremos o nosso estudo, caracterizando o triângulo
retângulo. Aproveitando, avançaremos para o Teorema de Pitágoras, de-
monstrando-o a partir de duas óticas: a primeira é a de nosso costume e a
segunda é uma maneira mais elaborada.
Após a nossa familiarização com os triângulos retângulos, estudaremos
a trigonometria, o foco de estudo desta unidade. Como de costume, par-
timos da definição de cada uma das razões trigonométricas e, em seguida,
demonstraremos as relações entre elas. Para demonstrarmos as relações en-
tre várias razões trigonométricas, recorremos às semelhanças de triângulos.
Depois, avançaremos para a determinação das razões seno, cosseno e
tangente dos ângulos notáveis, que são de 30º, 45º e 60º, tendo em vista que
não devemos, apenas, decorar os valores, mas precisamos ter noção de como
esses valores foram determinados. Para determinarmos os valores dessas ra-
zões trigonométricas, recorremos novamente às semelhanças de triângulos.
Posteriormente, estudaremos os ciclos trigonométricos conhecidos
como trigonometria na circunferência. Nos ciclos trigonométricos, com-
preenderemos o sinal de cada uma das razões trigonométricas e a redução
ao 1º quadrante. Para isso, precisamos nos lembrar das simetrias e dos
ângulos complementares e suplementares.
Por fim, estudaremos duas leis trigonométricas, isto é, a lei de senos e
a lei de cossenos para triângulos quaisquer. Essas duas leis trigonométri-
cas permitem a resolução de algumas situações-problemas que envolvem
razões trigonométricas em triângulos quaisquer. Atente-se para o fato de
que a sua demonstração envolve semelhanças de triângulos, mais especi-
ficamente, de triângulos retângulos, tendo em vista que as razões trigono-
métricas são razões verificadas entre os lados de um triângulo retângulo.
Caro(a) aluno(a), desejamos-lhe um bom estudo!
1
TRIÂNGULO RETÂNGULO E

UNICESUMAR
TEOREMA DE
pitágoras

Antes de iniciarmos o estudo da trigonometria propriamente dita, compreende-


remos um pouco o triângulo retângulo.

Triângulo retângulo

Ao dividirmos um retângulo por uma das diagonais, obtemos dois triângulos


semelhantes, assim como podemos observar na figura a seguir:

ou

Figura 1 – Obtenção de dois triângulos a partir da divisão de um retângulo / Fonte: os autores.


69
Por ser exatamente a metade de um retângulo, podemos observar que o maior
UNIDADE 3

ângulo de cada um dos triângulos é de 90º graus. O lado oposto ao ângulo reto
(ângulo de 90º ) do triângulo retângulo, nesse caso, o que era diagonal, é o lado
de maior medida e é chamado hipotenusa. Os lados adjacentes ao ângulo reto,
que eram os lados do retângulo, são chamados catetos.

Teorema de Pitágoras

A partir da medida de dois lados de um triângulo retângulo, podemos determinar


a medida do terceiro lado por meio de Teorema de Pitágoras:
h2  c12  c22

Sendo:
h : hipotenusa,
c1 : cateto 1 , e
c2 : cateto 2 .

2 2 2
Observação: alguns escrevem o Teorema de Pitágoras como h  c  C . No
entanto, não podemos usar letra maiúscula para representar o segmento de reta,
como o cateto. O pior é associar o C ao cateto de medida maior, uma vez que nem
sempre conhecemos as medidas de ambos os catetos.
Alguns demonstram a validade do Teorema de Pitágoras, construindo um
triângulo retângulo cuja hipotenusa mede 5 cm e os catetos medem 4 cm e 3
cm, assim como é observável na figura a seguir:

5 cm
3 cm

4 cm

Figura 2 – Construção de um triângulo retângulo / Fonte: os autores.

70
Em seguida, constroem-se 3 quadrados cujas arestas são cada um dos lados deste

UNICESUMAR
triângulo retângulo, assim como é mostrado a seguir:

A = 25 cm2

5 cm A = 9 cm2
3 cm

4 cm

A = 16 cm2

Figura 3 – Três quadrados são formados a partir de um triângulo retângulo / Fonte: os autores.

Podemos observar que a área do quadrado maior, isto é, a área do quadrado cuja
aresta é a hipotenusa do triângulo, equivale à soma das áreas de quadrados cujas
arestas são iguais aos catetos do mesmo triângulo.
Essa relação também pode ser observada na figura a seguir:

A = 3.125π cm2

5 cm A = 1.125π cm2
3 cm

4 cm

A = 2π cm2

Figura 4 – Formam-se três metades de círculo a partir de um retângulo / Fonte: os autores.

71
Entretanto, no Ensino Superior, precisamos mostrar que o Teorema de Pitágoras
UNIDADE 3

é válido para todos os triângulos retângulos. Dessa forma, faremos assim como
é evidenciado no exemplo a seguir:
C

α
A B

Figura 5 – Triângulo retângulo / Fonte: os autores.

Marcamos o ponto D , de forma que DB seja a altura do triângulo em relação à


base AC , assim como é visível na figura a seguir:
C

D β

α
α β
A B

Figura 6 – Dividimos o triângulo retângulo em dois por meio de sua altura / Fonte: os autores .

Observação: denotaremos por AB , o segmento de reta entre os pontos A e B ,


e denotaremos por AB , a medida desse segmento de reta.

Verificamos que  ABC ,  ADB,  BDC são semelhantes pela relação AAA
(ângulo, ângulo, ângulo), isto é, todos os ângulos correspondentes são congruentes:

72
C

UNICESUMAR
B
β C
β
β

α α α
A B A D B D

Figura 7 – Triângulos semelhantes com ângulos congruentes / Fonte: os autores.

Observação: ângulos congruentes significam ângulos com a mesma medida.

Pelo fato de que as razões entre os lados correspondentes de dois triângulos se-
melhantes são proporcionais, ao comparar ABC e ADB , temos:

AB AD

AC AB

    AC    AD 
2
 AB

Comparando ABC e BDC , temos:

BC DC

AC BC

    AC    DC 
2
 BC

Assim:

 AB    BC    AC    AD    AC   CD 
2 2

  AC    AD  DC 

Como AD  CD  AC , substituindo AD + CD por AC , temos:

73
 AB    BC    AC    AC 
2 2
UNIDADE 3

  AB    BC    AC 
2 2 2

O que equivale a:

 AC    AB    BC 
2 2 2
.

Pelo fato de que no ABC , AC é hipotenusa  h  , AB é um dos catetos  c1 


e BC é outro cateto  c2  , fazendo as substituições, temos:

h2  c12  c22 .

pensando juntos

Alguns recorrem à soma das áreas de quadrados, enquanto outros empregam a soma de
semicírculos, para mostrar a validade do Teorema de Pitágoras. Será que podemos usar
outras áreas, como a de triângulos equiláteros ou de pentágonos?

74
2
RAZÕES TRIGONOMÉTRICAS

UNICESUMAR
NO TRIÂNGULO
retângulo

Primeiramente, estudaremos as razões seno, cosseno e tangente. Para isso, ana-


lisemos o triângulo retângulo a seguir:

hipotenusa
cateto oposto ao ângulo α
α
cateto adjacente ao ângulo α

Figura 8 – Triângulo retângulo inscrito em uma circunferência tal que a hipotenusa do triân-
gulo é igual ao raio da circunferência / Fonte: os autores.

75
Começaremos com a razão seno, que é a razão entre o cateto oposto ao ângulo
UNIDADE 3

dado e a hipotenusa.

Exemplo 1: na figura apresentada a seguir, temos:

3 cm
2 cm

Figura 9 – Triângulo retângulo com hipotenusa igual a 3 cm e cateto oposto igual a 2 cm


Fonte: os autores.

2 Cateto Oposto
sen a = , tendo em vista que sen a = .
3 Hipotenusa

Já a razão cosseno é a razão entre o cateto adjacente ao ângulo dado e a hipotenusa.

Exemplo 2: na figura:

5 cm

α
4 cm
Figura 10 – Triângulo retângulo com hipotenusa igual a 5 cm e cateto adjacente igual a 4 cm
Fonte: os autores.

4 Cateto Adjacente
temos cos a = , tendo em vista que cos a = .
5 Hipotenusa
76
UNICESUMAR
2
Observação: note que sen a   sen a  .
2

Utilizaremos a observação anterior para introduzirmos a principal relação exis-


tente entre seno e cosseno, a qual é conhecida como a Relação Fundamental
da Trigonometria. Ela é dada por:

sen 2 a  cos2 a  1
.

Demonstração: façamos a demonstração da relação fundamental, partindo do


seguinte triângulo retângulo, sabendo que 0º < a < 90º :

α
A B

Figura 11 – Triângulo retângulo / Fonte: os autores.

A partir disso, temos:

Cateto Oposto BC
sen a   sen a 
Hipotenusa AC

e
Cateto Adjacente AB
cos a   cos a  .
Hipotenusa AC

Portanto, temos:
77
BC  sen a  AC
UNIDADE 3

e
AB  cos a  AC .

Já sabemos que é válido o Teorema de Pitágoras. Então, vamos aplicá-lo no


triângulo ABC :

 AC    AB    BC 
2 2 2
.

Agora, substituiremos os valores encontrados para BC e AB :

 AC    cos a  AC   sen a  AC 
2 2 2

  AC   cos2 a   AC   sen 2 a   AC 
2 2 2

1
 
Multiplicando todos os termos por AC 2 , temos:

1  cos2 a  sen 2 a

A razão tangente é a razão entre o cateto oposto ao ângulo dado e o cateto adjacente
ao mesmo ângulo, que resulta em razão entre a razão seno e a razão cosseno, ou seja:

Cateto Oposto
sen a Hipotenusa Cateto Oposto
=
tg a = = .
cos a Cateto Adjacente Cateto Adjacente
Hipotenusa

Demonstração: observe a figura a seguir:

78
C

UNICESUMAR
α

A B

Figura 12 – Triângulo retângulo e circunferência tal que o raio da circunferência é igual ao


cateto adjacente ao ângulo α / Fonte: os autores.

BC sen a
=
Na figura apresentada, sabemos que tga = .
AC cos a

Para mostrar a validade da razão anterior, construiremos a altura CD em relação


à base AB . Em seguida, denominamos o ângulo de 90º − a como b :
C

β α

A D B

Figura 13 – Altura construída no triângulo retângulo e circunferência tal que o raio da circun-
ferência é igual ao cateto adjacente ao ângulo α / Fonte: os autores.
79
Verificamos que ABC e ACD são semelhantes por AAA (ângulo, ângulo,
UNIDADE 3

ângulo), pelo fato de todos os ângulos correspondentes serem congruentes:

α
β α β

A B A C

Figura 14 – Semelhança de triângulos por congruência de todos os ângulos / Fonte: os autores.

Como as razões entre os lados correspondentes de dois triângulos formam uma


proporção, temos:

BC CD
=
tga = .
AC AD

1
Multiplicando CD e AD por , temos:
AC

1 CD
CD 
CD AC  AC .
tga  
AD AD  1 AD
AC AC

CD AD
Como = sen a e = cos a , temos:
AD AC

sen a
tg a = .
cos a

80
Exemplo 3: na figura:

UNICESUMAR
3 cm

α
5 cm

Figura 15 – Triângulo retângulo com cateto adjacente igual a 5 cm e cateto oposto igual a 3 cm
Fonte: os autores.

3
Assim como já estudamos, temos tg a = .
5
Agora, estudaremos um pouco mais as três razões trigonométricas, que são: se-
cante, cossecante e cotangente.
A razão secante é a razão entre a hipotenusa e o cateto adjacente ao ângulo
a , isto é, a razão inversa da razão cosseno:

Hipotenusa
sec a = ,
Cateto Adjacente
Que equivale a:
1
sec a = .
cos a

Demonstração: considere o triângulo retângulo apresentado a seguir:


C

α
A B
Figura 16 – Triângulo retângulo / Fonte: os autores.
81
Construindo a altura DB em relação ao lado AC , temos:
UNIDADE 3

D
β

α
α β

A B

Figura 17 – Construção da altura DB do triângulo retângulo / Fonte: os autores.

C e BDC
Assim, verificamos congruências entre A B  , ACB
 e BCD
 , e B AC e
 . Portanto, ABC e BDC são semelhantes por AAA :
C BD
C

β
C

β
α α
A B B D

Figura 18 – Semelhança de triângulos com todos os ângulos iguais / Fonte: os autores.

Hipotenusa
Como sec a = , temos:
Cateto Adjacente
1
BC 
AC BC BC  1
sec a   
AB BD BD  1 BD
BC BC
1
 sec a 
BD
BC
82
UNICESUMAR
BD
Agora, como = cos a , temos:
BC

1 1
sec α   .
BD cos α
BC

A razão cossecante é a razão entre a hipotenusa e o cateto oposto ao ângulo a ,


isto é, a razão inversa da razão seno:
Hipotenusa
cossec a = ,
Cateto Oposto

Que equivale a:
1
cossec a = .
sen a

Demonstração: considere o triângulo retângulo exposto a seguir:

α
A B
Figura 19 – Triângulo retângulo / Fonte: os autores.

Construindo a altura DB em relação ao lado AC , temos:

83
C
UNIDADE 3

D
β

α
α β

A B

Figura 20 – Construção da altura DB no triângulo retângulo / Fonte: os autores.

 e ADB
Dessa forma, verificamos congruências entre ABC  , ACB e ABD
 ,e
B AC e B AD . Portanto, ABC e ADB são semelhantes por AAA :

C
B

β
β

α α
A B A D

Figura 21 – Semelhança de triângulos com todos os ângulos iguais / Fonte: os autores.

Hipotenusa
Como cossec a = , temos:
Cateto Oposto

84
1
AB 

UNICESUMAR
AC AB AB  1
cossec a   
BC BD BD  1 BD
AB AB
1
 cossec a 
BD
AB

BD
Agora, como = sen a , temos:
AB

1 1
cos sec α   .
BD sen α
AB

A razão cotangente é a razão entre o cateto adjacente ao ângulo a e o cateto


oposto ao mesmo ângulo, isto é, a razão inversa da razão tangente:

Cateto Adjacente
cotg a = ,
Cateto Oposto

Que equivale a:
1
cotg a = .
tg a

Demonstração: considere o triângulo retângulo apresentado a seguir:


C

α
A B
Figura 22 – Triângulo retângulo / Fonte: os autores.

85
Construindo a altura DB em relação ao lado AC , temos:
UNIDADE 3

D
β

α
α β

A B
Figura 23 – Construção da altura DB no triângulo retângulo / Fonte: os autores.

 e ADB
Dessa forma, verificamos congruências entre ABC  , ACB  e ABD
 ,e
B AC e B AD . Portanto, ABC e ADB são semelhantes por AAA :
C
B

β
β

α α
A B A D

Figura 24 – Semelhança de triângulo com todos os ângulos iguais / Fonte: os autores.

Cateto Adjacente
Como cotg a = , temos:
Cateto Oposto
1
AD 
AB AB AD  1
cotg a   
BC BD BD  1 BD
AD AD
1
 cotg a 
BD
AB
86
UNICESUMAR
BD
Agora, como = tg a , temos:
AD
1 1
 cotg a   .
BD tg a
AD

Tabelas de razões trigonométricas dos ângulos


notáveis

Determinaremos os valores de seno, cosseno e tangente de ângulos notáveis, isto


é, dos ângulos de 30º , 45º e 60º .
Para tanto, começaremos com o sen 30º :
C

30°
A B

Figura 25 – Triângulo retângulo com ângulo  igual a 30º / Fonte: os autores.

 = 60º , marcamos D sobre AC , de forma que BCD se


Pelo fato de que BCA
torne um triângulo equilátero, assim como podemos observar na figura a seguir:
C

60°

60°

60°
30° 30°
A B
Figura 26 – Construção do triângulo equilátero CDB no triângulo retângulo / Fonte: os autores.
87
É perceptível que ABD é um triângulo isósceles. Assim, AD e BD são con-
UNIDADE 3

gruentes e, consequentemente, BC , CD, BD e AD são congruentes. Portanto,


verificamos que AC  2  DC  2  BC .
Cateto Oposto
Como sen a = , temos:
Hipotenusa

BC BC 1
sen 30º   
AC 2  BC 2

Para determinarmos o valor de sen 45º , consideremos a figura a seguir:


C

45°

A B

Figura 27 – Triângulo retângulo com ângulo  igual a 45º / Fonte: os autores.

Pelo fato de que a soma dos ângulos internos de um triângulo é de 180º ,


  180º 90º 45º  45º . Assim, ABC é um triângulo isósceles de base
ACB
AC . Portanto, AB e BC são congruentes:
C

45°

45°

A B
Figura 28 – Verificação que se trata de um triângulo isósceles / Fonte: os autores.
88
Assim, temos:

UNICESUMAR
 AC    AB    BC 
2 2 2

  BC    BC 
2 2

 2   BC 
2

 
2
 AC  2  BC

 AC  2  BC

Cateto Oposto
Como sen a = , temos:
Hipotenusa

BC BC 1 2 2
sen 45º      .
AC 2  BC 2 2 2

Para determinarmos o valor de sen 60º , consideremos a figura a seguir:

Figura 29 – Triângulo retângulo com ângulo  igual a 60º / Fonte: os autores.

Construímos DBC simétrico ao ABC em relação ao lado BC , assim como


podemos observar na figura a seguir:

89
C
UNIDADE 3

30° 30°

60° 60°

A B D

Figura 30 – Construção de um triângulo simétrico ao apresentado na Figura 29 / Fonte: os autores.

Pelo fato de que ADC é um triângulo equilátero, temos AC , AD e CD con-

gruentes. Além disso, como AB e BD são congruentes e AB  BD  AD ,    


temos AB = AC .
2
AC
    AB    BC 
2 2 2
Agora, como AC , e AB = , temos:
2
2
 AC 
 AC   
2 2
   BC
 2 

 AC 
2

 
2
  BC
4
 AC 
2

    AC 
2 2
 BC 
4
    AC 
2 2
4  AC
 
2
 BC 
4
 
2
3  AC
 
2
 BC 
4

 
2
3  AC
C
 BC 
4
3  ( AC )2
 BC 
4
90 3  AC
 BC 
2
UNICESUMAR
Cateto Oposto
Como sen a = , temos:
Hipotenusa

3  AC 3  AC
BC 2 2 3  AC 1 3
sen 60º       .
AC AC AC 2 AC 2
1
Para determinarmos os valores de cossenos de ângulos de 30º , 45º e 60º , proce-
demos de forma análoga ao de seno desses ângulos ou, ainda, podemos recorrer
à relação entre o seno e o cosseno.
Para determinarmos os valores de tangente desses ângulos, determinamos a
razão entre seno e cosseno por tg a = sen a .
cos a

91
Assim:
UNIDADE 3

Ângulo ( a ) 30º 45º 60º

sen a 1 2 3
2 2 2

Razão cos a 3 2 1
2 2 2

tg a 3 1 3
3

Tabela 1 – Razões trigonométricas de ângulos notáveis / Fonte: os autores.

Finalizamos, assim, a apresentação e a construção da tabela das razões trigono-


métricas dos ângulos notáveis. Conhecer essa tabela e a sua construção ajuda e
facilita grande parte dos cálculos, envolvendo trigonometria.

pensando juntos

Constatamos que, ao dobrar a medida do ângulo de 30º , isto é, ao aumentar de 30º


para 60º , não foi dobrada a razão seno. Você já pensou no por quê?

92
3
CICLOS

UNICESUMAR
TRIGONOMÉTRICOS

Construiremos a circunferência cujo centro é o ponto de origem, assim como é


visualizável na figura a seguir:
3

O
-3 -2 -1 0 1 2 3

-1

-2

-3

Figura 31 – Circunferência de raio 2 e centro na origem do plano cartesiano / Fonte: os autores.

93
Verificamos que os eixos cartesianos, isto é, o eixo das abscissas e o eixo das or-
UNIDADE 3

denadas, dividem a circunferência em quatro partes denominadas quadrantes:

2º quandrante 1º quandrante

3º quandrante 4º quandrante

Figura 32 – Um círculo possui 4 quadrantes / Fonte: os autores.

Você se lembra das razões trigonométricas estudadas?

F E

2º quandrante 1º quandrante

C B D
O A

3º quandrante 4º quandrante
H

Figura 33 – Razões seno e cosseno apresentadas no ciclo trigonométrico / Fonte: os autores.


94
Cateto Oposto
Pelo fato de que sen a = , a hipotenusa é a medida do raio da

UNICESUMAR
Hipotenusa

circunferência, AE e BF são positivos e CG e DH são negativos, verificamos


que o valor de seno no 1º e 2º quadrantes é positivo, enquanto no 3º e 4º qua-
drantes é negativo.
Analogamente, o valor de cosseno no 1º e 4º quadrantes é positivo, enquanto,
no 2º e 3º quadrantes, negativo. Consequentemente, o valor da tangente no 1º e
3º quadrantes é positivo e no 2º e 4º quadrantes, negativo.
Assim:

1º qua- 2º qua- 3º qua- 4º quadran-


drante drante drante te

seno + + – –
Sinal
cosseno + – – +
de
tangente + – + –

Quadro 1 – Sinal das razões trigonométricas em cada quadrante / Fonte: os autores.

No próximo tópico, será apresentada a redução ao 1º quadrante. É uma ferramen-


ta que nos permite transpor uma razão trigonométrica para ângulos presentes no
primeiro quadrante, com o intuito de agilizar e facilitar seus cálculos.

Redução ao 1º quadrante

Quando o ângulo pertencer ao 2º, 3º ou 4º quadrante, podemos reduzi-lo ao 1º


quadrante:

a) Redução do 2º quadrante para o 1º:

Observe a figura a seguir:

95
UNIDADE 3

α
180° α
A O

Figura 34 – Triângulo inscrito no 2º quadrante da circunferência cujo raio é a hipotenusa do


triângulo / Fonte: os autores.

Vamos refletir os pontos A e B em relação ao eixo das ordenadas e, em seguida,


construiremos um triângulo simétrico ao OAB , assim como o apresentado na
figura a seguir:

B B1

180° α 180° α
A O A1

Figura 35 – Construção de um triângulo simétrico ao apresentado na Figura 34 / Fonte: os autores.

96
  180º  a .
Assim, verificamos que AB = A1 B1 e AO = A1O quando A1 OB

UNICESUMAR
1

No 2º quadrante, temos:

■ sen a é positivo: se 90º < a < 180º , temos sen a  sen 180º  a  .

■ cos a é negativo: se 90º < a < 180º , temos cos a   cos 180º  a  .

■ tg a é negativo: se 90º < a < 180º , temos tg a  tg 180º  a  .

b) Redução do 3º quadrante para o 1º:

Observe a figura a seguir:

A α 180° O

Figura 36 – Triângulo inscrito no 3º quadrante da circunferência cujo raio é a hipotenusa do


triângulo / Fonte: os autores.

Vamos refletir os pontos A e B em relação ao ponto de origem e, em segui-


da, construiremos um triângulo simétrico ao OAB , assim como é visível na
figura a seguir:

97
UNIDADE 3

B1

A α 180°
α 180° O A1

Figura 37 – Construção de um triângulo simétrico em relação à origem / Fonte: os autores.

  a  180º .
Assim, verificamos que AB = A1 B1 e AO = A1O quando A1 OB1

No 3º quadrante, temos:

■ sen a é negativo: se 180º < a < 270º , temos sen a   sen  a  180º  .

■ cos a é negativo: se 180º < a < 270º , temos cos a   cos  a  180”  .

■ tg a é positivo: se 180º < a < 270º , temos tg a  tg  a  180º  .

c) Redução do 4º quadrante para o 1º:

Observe a figura a seguir:

98
UNICESUMAR
α A
O
260° α

Figura 38 – Triângulo inscrito no 4º quadrante da circunferência cujo raio é a hipotenusa do


triângulo / Fonte: os autores.

Vamos refletir o ponto B em relação ao eixo das abscissas e, em seguida,


construiremos um triângulo simétrico ao OAB , assim como é visualizável
na figura a seguir:

B1

360° α A
O 360° α

Figura 39 – Construção de um triângulo retângulo simétrico / Fonte: os autores.

99
Assim, verificamos que AB = AB1 e possui lado comum  AO  , quando
UNIDADE 3

 = 360”- a .
AOB 1

No 4º quadrante, temos:

■ sen a é negativo: se 270º < a < 360º , temos sen a   sen  360º  a  .

■ cos a é positivo: se 270º < a < 360º , temos cos a  cos  360º  a  .

■ tg a é negativo: se 270º < a < 360º , temos tg a  tg  360º  a  .

Assim, para resumirmos, observe o quadro a seguir:

2º quadrante 3º quadrante 4º quadrante

sen a sen 180º  a   sen  a  180º   sen  360º  a 

cos a  cos 180º  a   cos  a  180º  cos  360º  a 

tga tg 180º  a  tg  a  180º  tg  360º  a 


Quadro 2 – Regra de redução ao 1º quadrante / Fonte: os autores.

Observação: outra unidade usada para medir os ângulos é o radiano, sendo:

1⋅ p rad = 180” .

p 3⋅p
Consequentemente, rad = 90º , rad = 270” , e 2  p rad  360º .
2 2
Exemplo 4:
2p
Determine a medida do cosseno, sabendo que o ângulo mede rad.
3

p 2p
< <p  2p 
Pelo fato de que sabemos que 2 3 e queremos calcular cos   ,
 3 
concluímos que esse ângulo está no 2º quadrante. Assim:
100
 2p   2p  p 1
cos     cos  p    cos    cos 60º  .

UNICESUMAR
 3   3  3 2

conecte-se

Caro(a) aluno(a), para melhor compreensão, assista ao vídeo no QRCode, o


qual apresenta a resolução desse problema.

pensando juntos

As razões trigonométricas nos triângulos retângulos sempre são positivas. No entanto os


valores nos ciclos trigonométricos podem ser negativos. Já pensou o que leva a esse fato?

anotações


















101
4
LEIS
UNIDADE 3

TRIGONOMÉTRICAS

Caro(a) aluno(a), agora, estudaremos duas leis trigonométricas: a lei do seno e


a lei do cosseno.

pensando juntos

Neste material, foram empregadas AB para representar o segmento da reta e AB para dis-
tância. Todavia existem outras representações para o segmento de reta. Você as conhece?

Lei do seno

Considerando ABC , temos:

BC AC AB
 

sen B AC  
sen ABC  
sen ACB  .
102
Demonstração: considere um triângulo qualquer inscrito em uma circunferên-

UNICESUMAR
cia como a apresentada a seguir:
C

A B

Figura 40 – Triângulo qualquer inscrito em uma circunferência / Fonte: os autores.

Em seguida, traçamos o diâmetro da circunferência BD e o chamamos de d ,


assim como é demonstrado na figura a seguir:

A B

Figura 41 – Construção do diâmetro da circunferência e de um novo triângulo retângulo


Fonte: os autores.

Verificamos que B AC e BDC


 são congruentes. Pelo fato de que:


 
  Cateto Oposto de BDC ,
sen BDC
Hipotenusa

103
Temos:
UNIDADE 3


  CB
sen BDC
d


Entretanto, sabemos que B AC e BDC


 são congruentes. Logo:

sen BDC  
  CB  sen B AC
d
 
 d  sen B  
AC  CB 
CB
d 
 
.
sen B 
AC

Agora, traçamos o diâmetro da circunferência CE e o chamamos de d1 , assim


como é visualizável na figura a seguir:

d1

A B

Figura 42 – Construção do diâmetro da circunferência e de um novo triângulo retângulo


Fonte: os autores.

 e AEC
Verificamos que ABC  são congruentes. Pelo fato de que:


 
  Cateto Oposto a AEC ,
sen AEC
Hipotenusa

104
Temos:

UNICESUMAR

  AC .
sen AEC
d1

 e AEC
No entanto sabemos que ABC  são congruentes. Logo:

 
  AC  sen ABC
sen AEC
d1

 

  AC
 d1  sen ABC 
AC
 d1 
 
.

sen ABC

Finalmente, traçamos o diâmetro da circunferência AF e o chamamos de d2 ,


assim como é visualizável na figura a seguir:

d2

A B

Figura 43 – Construção do diâmetro da circunferência e de um novo triângulo retângulo


Fonte: os autores.

 e AFB
Verificamos que ACB  são congruentes. Pelo fato de que:


 
  Cateto Oposto a AFB ,
sen AFB
Hipotenusa

105
Temos:
UNIDADE 3


  AB .
sen AFB
d2


 e AFB
Contudo como sabemos que ACB  são congruentes, temos:

 
  AB  sen ACB
sen AFB
d2

 

  AB
 d2  sen ACB 
AB
 d2 
 
.

sen ACB

Agora, como d= d=
1 d2 , sabemos que:

BC AC AB
  .

sen B AC  
sen ABC  
sen ACB   

Exemplo 5:

Para determinarmos a medida de AB no triângulo a seguir:

106
C

UNICESUMAR
45°

9cm

60°

A x B

Figura 44 – Triângulo com ângulos de 60º e 45º / Fonte: os autores.

Procedemos, assim, como é exposto a seguir:

9 x

sen 60º sen 45º
9 x
 
3 2
2 2
3  x 9 2
 
2 2
 3  x  9 2
9 2
x
3

9⋅ 2 3
Multiplicando por , temos:
3 3

9 2 3
x 
3 3
9 2  3

3
3 6
107
UNIDADE 3

conecte-se

Caro(a) aluno(a), para melhor compreensão, assista ao vídeo no QRCode, o


qual apresenta a resolução desse problema.

Lei do cosseno

Em relação ao cosseno, considerando o triângulo ABC , verificamos que:

    AC    BC      
2 2 2
a) AB  .
 2  AC  BC  cos ACB

b)  AB    AC    BC   2   AC    BC   cos  ABC 
2 2 2
 .

c)  BC    AB    AC   2   AB    AC   cos  B 
AC  .
2 2 2

Demonstração: para demonstrar as afirmativas (a), (b) e (c), precisamos


analisá-las no triângulo acutângulo (todos os ângulos menores que 90º), no
triângulo retângulo e no triângulo obtusângulo (um dos ângulos maior que
90º), respectivamente:

    AC    BC      
2 2 2
a) AB 
 2  AC  BC  cos ACB é válida para o
triângulo acutângulo. Para tanto, observe o triângulo a seguir:
B

A C
Figura 45 – Triângulo com três ângulos diferentes / Fonte: os autores.
108
Construímos a altura BD em relação à base AC , assim como é demonstrado

UNICESUMAR
na figura a seguir:
B

A C

Figura 46 – Construção da altura BD no triângulo retângulo / Fonte: os autores.

2 2 2
Pelo fato de que h  c1  c2 , verificamos que:

 BC    BD   CD 
2 2 2

  BD    BC    CD 
2 2 2

 AB    BD    AD 
2 2 2

  BD    AB    AD 
2 2 2

Assim, temos:

 AB    AD    BC   CD 
2 2 2 2

  AB    BC    CD    AD  .
2 2 2 2

     
Contudo, como sabemos que AD  AC  CD e

 AD    AC        
2 2 2 2
 2  AC  CD  CD , substituindo AD por

 AC      
2 2
 2  AC  CD  CD , temos:

109
 AB    BC   CD    AC   2   AC   CD   CD 
2 2 2 2 2
UNIDADE 3

  AB    AC    BC   2   AC    CD  .
2 2 2

 BC 
  
Agora, multiplicaremos 2  AC  CD por 
BC
 . Logo, temos:
 

 BC 
 AB    AC    BC    
2 2 2
 2  AC  CD   
 BC 
 CD  .
    BC    
2 2
 AC  2  AC  BC 
 BC 
 CD 
Pelo fato de que  

  cos BCD , temos:
 BC 


 AB    AC    BC      
2 2 2

 2  AC  BC  cos BCD

 e ACB
Além disso, como BCD  são congruentes, temos:

 AB    AC    BC      
2 2 2

 2  AC  BC  cos ACB

    AC    BC      
2 2 2
b) AB 
 2  AC  BC  cos ABC é válida para o

triângulo retângulo. Para tanto, observe o triângulo a seguir:

110
B

UNICESUMAR
A C

Figura 47 – Triângulo retângulo / Fonte: os autores.

    AC    BC 
2 2 2
Note que AB . Além disso, pelo fato de que cos 90º = 0 e
 é um ângulo reto, temos cos ACB
ACB  
  0 e, consequentemente,

     0.
2  AC  BC  cos ACB  

    
 ao teorema de Pitágoras, temos:
Assim, somando 2  AC  BC  cos ACB

 AB    AC    BC      
2 2 2
 .
 2  AC  BC  cos ABC

    AB    AC      
2 2 2
c) BC  2  AB  AC  cos B 
AC é válida para o

triângulo obtusângulo. Para tanto, observe o triângulo a seguir:

C B

Figura 48 – Triângulo obtusângulo / Fonte: os autores.

111
Construímos a altura AD em relação à base BC , assim como é demonstrado
UNIDADE 3

na figura a seguir:
A

B C D

Figura 49 – Construção da altura DA do triângulo / Fonte: os autores.

Pelo Teorema de Pitágoras, sabemos que:

 AB    BD    AD 
2 2 2

  AD    AB    BD 
2 2 2

 AC   CD    AD 
2 2 2

  AD    AC    CD 
2 2 2

Assim, temos:

 AB    BD    AC   CD 
2 2 2 2

  AB    AC    CD    BD  .
2 2 2 2

     
No entanto, pelo fato de que BD  BC  CD e

 BD    BC      
2 2 2
 2  BC  CD  CD , fazendo essa substituição, temos:

 AB    AC   CD    BC   2   BC   CD   CD 


2 2 2 2 2

  AC    BC   2   BC    CD  .
2 2

112
UNICESUMAR
AC
  
Multiplicando o termo 2  BC  CD por
AC
, temos:

 AC 
 AB    AC    BC    
2 2 2
 2  BC  CD   
 AC 
 CD 
      
2 2
 AC  BC  2  BC  AC   
 AC 
CD   cos ACD
Agora, como
 AC 
 
 , temos:

 AB    AC    BC      
2 2 2
 .
 2  BC  AC  cos ACD

 e ACD
Entretanto sabemos que ACB  são ângulos suplementares, isto é, a soma

  180º  ACB
desses ângulos é igual a 180º , ACD  e, consequentemente:

 
  cos 180º  ACB
cos ACD 
 
 .
  cos ACB  
Assim:

 AB    AC    BC   2   BC    AC     cos  ACB 
2 2 2

  AC    BC   2   AC    BC   cos  ACB 
2 2

Por (a), (b) e (c), verificamos que:

 AB    AC    BC      
2 2 2

 2  AC  BC  cos ACB

É válido para todos os tipos de triângulos.

113
Exemplo 6:
UNIDADE 3

Para determinarmos a medida de AB da figura a seguir:


C

60°
6cm
4cm

A B

Figura 50 – Triângulo de lados 4 cm e 6 cm / Fonte: os autores.

Procedemos da seguinte maneira:

 AB    AC    BC      
2 2 2
Pelo fato de que 
 2  AC  BC  cos ACB e
1
AC = 4 , BC = 6 e cos 60º = , temos:
2
1
 AB 
2
 4 2  62  2  4  6 
2
 16  36  24
 28
 AB  28
 2  7 cm

explorando Ideias

Dois triângulos são semelhantes, quando:


Dois ângulos são congruentes (AA): pelo fato de que a soma dos ângulos internos de um
triângulo é igual a 180º, se dois ângulos forem congruentes, o terceiro ângulo também será
congruente. Assim, neste material, foram considerados três ângulos congruentes (AAA).

114
Três lados correspondentes forem proporcionais (LLL): se três lados forem proporcionais,

UNICESUMAR
os três ângulos são congruentes.
Dois lados correspondentes forem proporcionais e o ângulo formado por eles for con-
gruente (LAL): neste caso, os dois ângulos restantes também serão congruentes.
Para ver a demonstração, acesse o link: http://cmup.fc.up.pt/cmup/mecs/Geometria/se-
melhanca%20de%20triangulos.pdf.
Fonte: os autores.

CONSIDERAÇÕES FINAIS

Caro(a) aluno(a), chegamos ao final de mais uma unidade de estudo. Aqui, apro-
fundamos a nossa compreensão acerca das principais razões trigonométricas e
iniciamos o estudo com a caracterização do triângulo retângulo. Após a caracteri-
zação, avançamos para a demonstração do Teorema de Pitágoras, a partir de duas
óticas: a primeira é bastante simples e com certa limitação, enquanto a segunda
é mais elaborada. Além disso, a primeira é indicada para iniciantes, enquanto a
segunda é recomendada para aqueles que cursam licenciatura em Matemática,
assim como você, já que é mais elaborada e válida para todas as situações.
Em seguida, estudamos as razões trigonométricas no triângulo retângulo.
Para tanto, definimos e demonstramos as principais razões trigonométricas e
suas relações. Para essas demonstrações, recorremos à semelhança de triângulos
por AAA (ângulo, ângulo, ângulos) e por LLL (lado, lado, lado). Em outras pa-
lavras, todas essas demonstrações seguem o mesmo raciocínio, o que facilita o
acompanhamento dos raciocínios apresentados.
Mesmo que alguns decorem os valores das razões trigonométricas de ângulos
notáveis, que são os ângulos de 30°, 45° e 60° , precisamos ter a preocupação,
quanto à origem desses valores. Assim, a nossa preocupação é a demonstração,
tendo em vista que nenhum conhecimento matemático surgiu do nada, isto é,
os conhecimentos matemáticos foram construídos, sistematicamente, ao lon-
go do tempo, o que permite a sua reconstituição e demonstração. Em seguida,
avançamos para o estudo dos ciclos trigonométricos. Para isso, recorremos às
semelhanças de triângulos e às reduções ao 1º quadrante.
Finalmente, estudamos as leis do seno e do cosseno, demonstrando-as. Mes-
mo que, aparentemente, essas leis não dependam do triângulo retângulo, elas são
demonstradas por meio da semelhança de triângulos, em especial, de triângulos
retângulos. Esperamos ter contribuído com sua formação acadêmica e profissional.

115
na prática

1. Analise atentamente as relações a seguir e julgue com (V) para as Verdadeiras e (F)
para as Falsas:

( ) sen 2 a  1  cos2 a .

1
( ) cos a = .
sec a
cos a
( ) cotg a = .
sen a
A sequência correta é:

a) F, F, F.
b) F, V, F.
a) V, F, F.
b) V, F, V.
c) V, V, V.

2. Sabendo que a hipotenusa mede 2⋅ 5 cm e um dos catetos mede o dobro do


outro, determine a medida dos catetos.

3. Observe a figura a seguir e determine a medida de AC :


C

6 cm

45° 30°
A B

4. Um quadrado de 8 cm de lado foi transformado em um losango e um dos ângulos


mede 60º . Determine a área desse losango.

116
na prática

5. Observe a figura a seguir e determine a sua área (considere 3 = 1.732 ):


C

45° 30°
A 21 cm B

 l2  3 
 A  
6. Prove a validade da fórmula da área do triângulo equilátero
 4  por

meio do Teorema de Pitágoras e por meio da razão trigonométrica, sabendo que


bh
área do triângulo, a partir de sua base e altura, é dada por A .
2

7. Analise atentamente as relações a seguir e julgue com (V) para as Verdadeiras e (F)
para as Falsas:

( ) A razão inversa da razão tangente é a razão cotangente.

( ) A razão inversa da razão seno é a razão cosseno.

( ) sec2 a  cossec 2a  1 .
cos a
( ) cotg a = .
sen a

A sequência correta é:

a) F, V, F, F.
b) V, V, F, V.
c) V, F, F, V.
d) V, F, V, V.
e) V, V, V, V.

117
na prática

8. Prove a relação tg 2a  sec2 a  1 .

9. Analise atentamente as afirmativas a seguir:

I - Se sen a < 0 e tg a > 0 , a está localizado no 4º quadrante.

II - Se cos b > 0 e cotg b > 0 , b está localizado no 1º quadrante.

III - Se sec g > 0 e cossec g > 0 , g está localizado no 2º quadrante.

É correto o que se afirma em:

a) I.
b) II.
c) III.
d) I e II.
e) II e III.

10. Uma pessoa de 3 m de altura, a qual está em pé, vê a janela do último andar de
um prédio ao ângulo de 60º . Ao afastar 50 m do local que estava, vê a mesma janela
ao ângulo de 30º . Sabendo que o chão é horizontal, qual é a altura aproximada da
janela em relação ao chão?

118
aprimore-se

Para considerar a gênese, devemos discutir qual é o significado que daremos ao


termo Trigonometria. Se o tomarmos como a ciência analítica estudada atualmente,
teremos a origem no século XVII, após o desenvolvimento do simbolismo algébrico.
Porém, se o considerarmos para significar a geometria acoplada à Astronomia, as
origens remontarão aos trabalhos de Hiparco, no século II a.C., embora existam
traços anteriores de seu uso. Se o considerarmos, ainda, para significar literalmente
medidas do triângulo, a origem será no segundo ou terceiro milênio antes de Cristo.
Estudar a história da trigonometria também permite observar o surgimento e o
progresso da Análise e da Álgebra, campos da Matemática nela contidos de forma
embrionária. A trigonometria, mais que qualquer ramo da matemática, desenvol-
veu-se no mundo antigo a partir de necessidades práticas, principalmente ligadas à
Astronomia, Agrimensura e Navegação.
Os primeiros indícios de rudimentos de trigonometria surgiram tanto no Egito
quanto na Babilônia, a partir do cálculo de razões entre números e entre lados de
triângulos semelhantes. No Egito, isso pode ser observado no Papiro Ahmes, co-
nhecido como Papiro Rhind, que data de aproximadamente 1650 a.C., e contém 84
problemas, dos quais quatro fazem menção ao seqt de um ângulo. Ahmes não foi
claro ao expressar o significado dessa palavra, mas, pelo contexto, pensa-se que o
seqt de uma pirâmide regular seja equivalente, hoje, cotangente do ângulo OMV.
Na construção das pirâmides, era essencial manter uma inclinação constante
das faces, o que levou os egípcios a introduzirem o conceito de seqt, que represen-
tava a razão entre afastamento horizontal e a elevação vertical.
Os primeiros vestígios de trigonometria surgiram não só no Egito, mas também
na Babilônia. Os babilônios tinham grande interesse pela Astronomia, tanto por ra-
zões religiosas quanto pelas conexões com o calendário e as épocas de plantio. É
impossível estudar as fases da lua, os pontos cardeais e as estações do ano sem
usar triângulos, um sistema de unidades de medidas e uma escala.
Um importante conceito no desenvolvimento da Trigonometria é o conceito de ângulo
e de como efetuar sua medida, uma vez que ele é fundamental em diversas situações,
como na compreensão das razões trigonométricas em um triângulo retângulo (números
que dependem dos ângulos agudos do triângulo e não da particular medida dos lados).

Fonte: Costa (2003).

119
eu recomendo!

conecte-se

Apresentação: este vídeo mostra como determinar as razões trigonométricas de ân-


gulos notáveis por meio do triângulo retângulo.
https://www.youtube.com/watch?v=77xJJaZsMxA

Apresentação: este vídeo mostra algumas aplicações das leis trigonométricas para
resolução de situações problemas.
https://www.youtube.com/watch?v=m8jWcX2KWlY&list=PL7RjLI0hJPfDqx_Zm0gVbpl-
Dp-1jHB1z1&index=8

Apresentação: este vídeo é destinado aos professores do Ensino Médio e trata sobre
as funções trigonométricas.
https://www.youtube.com/watch?v=NiyX_hTl3tw

Apresentação: um bom material para aqueles que desejam se aprofundar nos estu-
dos da trigonometria de forma elaborada.
http://www.sites.uem.br/profmat/reges_gaieski.pdf

Apresentação: uma boa sugestão para conhecer um pouco da história da trigonome-


tria, soma e diferença de ângulos e suas razões trigonométricas.
http://ecalculo.if.usp.br/historia/historia_trigonometria.htm

120
ANEXO I

UNICESUMAR
TABELAS DE RAZÕES TRIGONOMÉTRICAS
Por meio da calculadora científica, podemos determinar as razões trigonométricas a
seguir, considerando a ordem para o seno, cosseno e tangente do referido ângulo:

a sen a cos a tg a a sen a cos a tg a


1º 0,0174 0,9998 0,0175 46º 0,7193 0,6947 1,0355
2º 0,0349 0,9994 0,0349 47º 0,7314 0,6820 1,0724
3º 0,0523 0,9986 0,0524 48º 0,7431 0,6691 1,1106
4º 0,0698 0,9976 0,0699 49º 0,7547 0,6561 1,1504
5º 0,0872 0,9962 0,0875 50º 0,7660 0,6428 1,1918
6º 0,1045 0,9945 0,1051 51º 0,7771 0,6293 1,2349
7º 0,1219 0,9925 0,1228 52º 0,7880 0,6157 1,2799
8º 0,1392 0,9903 0,1405 53º 0,7986 0,6018 1,3270
9º 0,1564 0,9877 0,1584 54º 0,8090 0,5878 1,3764
10º 0,1736 0,9848 0,1763 55º 0,8192 0,5736 1,4281
11º 0,1908 0,9816 0,1944 56º 0,8290 0,5592 1,4826
12º 0,2079 0,9781 0,2126 57º 0,8387 0,5446 1,5399
13º 0,2250 0,9744 0,2309 58º 0,8480 0,5299 1,6003
14º 0,2419 0,9703 0,2493 59º 0,8572 0,5150 1,6643
15º 0,2588 0,9659 0,2679 60º 0,8660 0,5 1,7320
16º 0,2756 0,9613 0,2867 61º 0,8746 0,4848 1,8040
17º 0,2924 0,9563 0,3057 62º 0,8829 0,4695 1,8807
18º 0,3090 0,9511 0,3249 63º 0,8910 0,4540 1,9626
19º 0,3256 0,9455 0,3443 64º 0,8988 0,4384 2,0503
20º 0,3420 0,9397 0,3640 65º 0,9063 0,4226 2,1445
21º 0,3584 0,9336 0,3839 66º 0,9135 0,4067 2,2460
22º 0,3746 0,9272 0,4040 67º 0,9205 0,3907 2,3558
23º 0,3907 0,9205 0,4245 68º 0,9272 0,3746 2,4751
24º 0,4067 0,9135 0,4452 69º 0,9336 0,3584 2,6051
25º 0,4226 0,9063 0,4663 70º 0,9397 0,3420 2,7475
26º 0,4384 0,8988 0,4877 71º 0,9455 0,3256 2,9042
27º 0,4540 0,8910 0,5095 72º 0,9511 0,3090 3,0777
28º 0,4695 0,8829 0,5317 73º 0,9563 0,2924 3,2708
29º 0,4848 0,8746 0,5543 74º 0,9613 0,2756 3,4874
30º 0,5 0,8660 0,5774 75º 0,9659 0,2588 3,7320
31º 0,5150 0,8572 0,6009 76º 0,9703 0,2419 4,0108
32º 0,5299 0,8480 0,6249 77º 0,9744 0,2250 4,3315
33º 0,5446 0,8387 0,6494 78º 0,9781 0,2079 4,7046
34º 0,5592 0,8290 0,6745 79º 0,9816 0,1908 5,1446
35º 0,5736 0,8192 0,7002 80º 0,9848 0,1736 5,6713
36º 0,5878 0,8090 0,7265 81º 0,9877 0,1564 6,3138
37º 0,6018 0,7986 0,7536 82º 0,9903 0,1392 7,1154
38º 0,6157 0,7880 0,7813 83º 0,9925 0,1219 8,1443
39º 0,6293 0,7771 0,8098 84º 0,9945 0,1045 9,5144
40º 0,6428 0,7660 0,8391 85º 0,9962 0,0872 11,4301
41º 0,6561 0,7547 0,8693 86º 0,9976 0,0698 14,3007
42º 0,6691 0,7431 0,9004 87º 0,9986 0,0523 19,0811
43º 0,6820 0,7314 0,9325 88º 0,9994 0,0349 28,6362
44º 0,6947 0,7193 0,9657 89º 0,9998 0,0174 57,2900
45º 0,7071 0,7071 1
121
4
PROGRESSÃO ARITMÉTICA
E PROGRESSÃO
geométrica

PROFESSORES
Me. Issao Massago
Me. Tiago Peres da Silva Suguiura

PLANO DE ESTUDO
A seguir, apresentam-se as aulas que você estudará nesta unidade: • Sequências numéricas • Progressão
aritmética • Progressão geométrica

OBJETIVOS DE APRENDIZAGEM
• Definir sequências numéricas • Definir progressão aritmética e resolver situações envolvendo-a • Definir
progressão geométrica e resolver situações envolvendo-a.
INTRODUÇÃO

Caro(a) aluno(a), seja bem-vindo(a) à quarta unidade do nosso livro! Nela,


estudaremos as sequências numéricas, em especial, a progressão aritmética
e a progressão geométrica, as quais são conhecidas, por alguns, como PA
e PG, respectivamente.
Pelo fato de que a PA e a PG são apresentadas de forma sistematizada
somente a partir do Ensino Médio, pode ser que você chegue a pensar que
o nosso primeiro contato com elas ocorre somente a partir daquele nível de
ensino. No entanto nosso primeiro contato com a PA ocorre antes mesmo
da escolarização formal, isto é, ao aprender a contar, tendo em vista que
um número natural, a partir do segundo, possui uma unidade a mais em
relação ao número natural anterior. Quanto à PG, nosso primeiro contato
ocorre quando começamos a multiplicar vários fatores iguais, isto é, geral-
mente, antes da formalização da potenciação. Além disso, é perceptível que,
ao iniciar o estudo da PA e da PG propriamente ditas, alguns se preocupam
demasiadamente com a aplicação de fórmulas, esquecendo-se de suas de-
monstrações, o que torna a resolução um processo puramente mecânico.
Mesmo que alguém chegue a pensar que as demonstrações sempre
são difíceis, muitas dessas fórmulas podem ser demonstradas com muita
facilidade, assim como acontece com a fórmula da soma de alguns termos
consecutivos de uma PA, a qual é conhecida como fórmula de Gauss.
Dessa forma, iniciaremos o nosso estudo sobre a progressão aritmética,
definindo-a e analisando alguns exemplos, a fim de facilitar a sua familia-
rização com os conteúdos a serem estudados. Em seguida, iniciaremos as
demonstrações das fórmulas usuais, tendo em vista que a apropriação de
conhecimentos só será concretizada quando compreendermos o meca-
nismo empregado na resolução. Além do mais, como de costume, serão
apresentados alguns cuidados que precisamos ter, ao resolvermos situações
que envolvem a PA. Por último, estudaremos a PG de forma análoga à PA.
Bons estudos!
1
SEQUÊNCIAS
UNIDADE 4

NUMÉRICAS

A sequência numérica é uma sucessão numérica em que seus elementos


são denominados termos, os quais são representados entre parênteses e
separados por vírgula.

Exemplo 1:
1,1, 2, 5, 8
1, 3, 5, 7, 9 
 2, 3, 5, 8,12,17, 25,
 7, 35,165,

Chamamos seus termos de 1º termo, 2º termo, 3º termo, assim sucessivamente, e


os representamos por a1 , a2 , a3 , .

No exemplo 1, 3, 5, 7, 9  , temos:

a1 1;=
= ; a3 5=
a2 3= ; a4 7; a5 = 9 .

Além disso, uma sequência pode ser finita ou infinita, assim como podemos
observar a seguir:
124
Exemplo 2:  2, 3, 5, 8  e 1, 2, 3, , 200  são exemplos de sequências finitas, por

UNICESUMAR
possuírem uma quantidade finita de termos.
Já a sequência 1, 1, 2, 3, 5, 8, 13, é uma sequência infinita, por possuir uma
quantidade infinita de termos.

pensando juntos

Você sempre consegue identificar o primeiro e o último termo de uma sequência?

2
PROGRESSÃO
ARITMÉTICA

A progressão aritmética (PA) é um tipo de sequência bastante comum e, de certa


forma, temos contato com alguns de seus exemplos desde criança. Em outras
palavras, começamos a ter contato com esse tipo de sequência assim que apren-
demos a contar e, em seguida, com a tabuada.
As crianças aprendem a contar 1, 2, 3, , que nada mais é que do que um
exemplo da PA ou de uma tabuada:

125
2 1 2;
UNIDADE 4

2 2 4;
2 3 6;

...
que resulta em PA  2, 4, 6, .

Caro(a) aluno(a), sendo assim, iniciaremos o nosso estudo sobre a PA.

Fórmula geral da PA

Uma sequência numérica é denominada progressão aritmética, ou PA de primei-


ra ordem, quando cada um de seus termos, a partir do segundo, for igual à soma
do termo anterior, com a razão dessa sequência representada por r . Consequen-
temente, cada um dos termos em uma PA, a partir do penúltimo, é igual ao termo
posterior, subtraindo a razão dessa sequência representada por r .

Exemplo 3: na PA 11, 13, 15, 17, 19  , temos a1 = 11 e r = 2 . Assim:

a1  11
a2  a1  r  11  2  13
a3  a2  r  13  2  15
a4  a3  r  15  2  17
a5  a4  r  17  2  19.
O que equivale a:

a1  11
a2  a1  r  11  2  13
a3  a2  r   a1  r   r  a1  2r  11  2  2  15
a4  a3  r   a1  2r   r  a1  3r  11  3  2  17
a5  a4  r   a1  3r   r  a1  4 r  11  4  2  19.

126
Exemplo 4: caso queira, podemos determinar o a20 (vigésimo termo) da

UNICESUMAR
PA 11, 13, 15, 17, 19  , assim como é demonstrado a seguir:

a1 11;
a2 a1 r 11 2 13;
a3 a 2 r 13 2 15;


a19 a18 r 45 2 47;
a20 a19 r 47 2 49.

Exemplo 5: considerando que o vigésimo primeiro termo  a21  de uma PA é


igual a 97 e sua razão é igual a 3 , podemos determinar o décimo oitavo termo,
caso assim preferir, do modo como é exposto a seguir:
a21  97;
a20  a21  r  97  3  94;
a19  a20  r  94  3  91;
a18  a19  r  91  3  88.

Observação: já pensou em determinar o a1000 (milésimo termo) por meio do


procedimento anterior? Quanto tempo levaria para determinar esse termo e qual
é a chance de não haver erros?
Para minimizar o erro e otimizarmos o nosso tempo, recorremos à fórmula
a seguir para determinarmos o último termo procurado de uma PA:

an  am   n  m   r ,

Sendo:

an : último termo em estudo.


am : primeiro termo em estudo.
n : índice do último termo em estudo.
m : índice do primeiro termo em estudo.
r : razão ou diferença.

127
Demonstração: considerando n > m , sabemos que an  an1  r . Além disso,
UNIDADE 4

pelo fato de que an1  an2  r , substituindo an−1 por an2  r , temos:
an  an1  r   an2  r   r  an2  2r

Como an2  an3  r , substituindo an−2 por an3  r , temos:


an3  an2  2r   an3  r   2r  an3  3r .

Repetindo esse processo, temos:


an  ani  i  r , i   e an( nm)  ann m  am .

Assim, verificamos que:


an  an( nm)   n  m   r  am   n  m   r

Observação: por meio dessa fórmula, podemos determinar, além do último


termo em estudo, o primeiro termo, a quantidade de termos em estudo e a razão
da PA em estudo, desde que tenha apenas um elemento desconhecido.

Exemplo 6: podemos determinar o oitavo termo da PA  52, 55, 58, 61,, assim
como é demonstrado a seguir:
a=n a=
8 ?
am= a=
1 52
=n 8=
, m 1, r = 3

Pelo fato de que a fórmula geral da PA é an  am   n  m   r , substituindo os


valores, temos:
a8  52   8  1  3
a8  52  21
a8  73.

Observação: para determinarmos a razão de uma PA de 1ª ordem, basta subtrair


qualquer termo do seu termo sucessor.

Exemplo 7: considerando que o vigésimo termo  a20  de uma PA é igual a 97


e a sua razão é igual a 4 , podemos proceder assim como é exposto a seguir, para
determinarmos o décimo segundo termo  a12  dessa PA:
128
Identificando os termos, temos:

UNICESUMAR
a=n a=20 97
a=
m a=
12 ?
=n 20
=, m 12, r = 4

Pelo fato de que a fórmula geral da PA é an  am   n  m   r , substituindo


os valores, temos:
97  a12   20  12   4
97  a12  8  4
97  a12  32
 a12  65

Exemplo 8: podemos determinar a1 de uma PA cujo a10 = 204 e a32 = 6 ?


Para isso, precisamos determinar r antes da aplicação da fórmula geral da
PA: assim, identificando os termos, temos:
a=n a=32 6;
a=
m a=
10 204;
n = 10;
r =?
Pelo fato de que a fórmula geral da PA é an  am   n  m   r , substituindo
os valores, temos:
6  204   32  10   r
6  204  22  r
 22r  198

 1 
Multiplicando ambos os lados por   , temos:
 22 
 1   1 
 22  r     198   
 22   22 
 r  9

Para determinarmos a1 , temos:


129
an  a32  6;
UNIDADE 4

am  a1  ?;
n  32, m  1, r  9.

Substituindo os valores na fórmula geral da PA:


6  a1   32  1   9 
6  a1  31   9 
6  a1  279
 a1  285

conecte-se

Caro(a) aluno(a), para melhor compreensão, assista ao vídeo no QRCode, o


qual apresenta a resolução desse problema.

Soma dos termos consecutivos da PA

Já parou para pensar em quanto tempo levaria para efetuar a seguinte adição:
1 + 2 + 3 + 4 + 5 +  + 100 ? Se pararmos e analisarmos o problema, talvez não
demore tanto tempo.
Conta a história que a turma de Gauss, na escola, era bastante inquieta e,
certa vez, seu professor decidiu dar uma atividade que deveria envolvê-los por
algum tempo. O professor pediu aos seus alunos que fizessem a soma de todos os
números naturais, entre 1 e 100 . Surpreendentemente, o menino Gauss conse-
guiu concluir a atividade em poucos minutos. O professor conferiu os cálculos e
verificou que Gauss havia acertado. Pediu-lhe, então, que explicasse como havia
feito as contas de forma tão rápida e Gauss prontamente mostrou a sua ideia.
Ele observou que, ao somarmos o primeiro número da sequência com o último,
obtemos o resultado de 101 , e que, ao somarmos o segundo número com o pe-
núltimo, também obtemos 101 como resultado e assim por diante. Ele também
notou que isso ocorria 50 vezes durante toda a sequência de números. Logo,
Gauss fez a simples conta: 50  101  5050 .
130
A fórmula para determinarmos a soma dos termos consecutivos da PA

UNICESUMAR
é dada por:
a  a 
S  1 n n
2

Sendo:

S : soma dos termos de uma PA.


n : índice do último termo.
1 : índice do primeiro termo.
a1 : primeiro termo em estudo.
an : último termo em estudo .

Demonstração: considere a soma dos termos da PA:


n
S ai a1 a2 a3 . . . an 2 an 1 an .
i 1

Note que podemos escrever essa soma de duas maneiras:


S1 a1 a2 a3 ... an 3 an 2 an 1 an
e
S2 an an 1 an 2 an 3 ... a3 a2 a1

Agora, façamos a soma de S1 e S2 :

S1 S2 a1 an a2 an 1 ... an 2 a3 an 1 a2 an a1

No entanto note que os termos  a1  an  ,  a2  an1  ,  an2  a3  e todos os


demais são equidistantes, ou seja, suas somas são iguais a a1 + an . Logo, podemos
reescrevê-los da seguinte maneira:

S1 S2 a1 an a1 an a1 an ... a1 an a1 an .

Pelo fato de que são n termos, essa soma se repete n vezes. Portanto:

131
S1  S2   a1  an   n
UNIDADE 4

Contudo note que S1 e S2 representam a mesma soma dos termos de uma PA,
logo, são iguais. Assim sendo, temos:
2 S   a1  an   n

S
 a1  an   n
2

Exemplo 9: queremos determinar a soma dos 200 primeiros termos da


PA  5, 8, 11, . Dessa forma, procedemos assim como é demonstrado a seguir:
Primeiramente, determinamos o a200 . Utilizaremos a fórmula geral da PA
an  am   n  m   r , em que am é o primeiro termo e an é o último. Assim, temos:

a200  5   200  1  r
a200  5  199  3
a200  602

Para determinarmos a soma dos 200 primeiros termos dessa PA, procedemos
assim como pode ser visualizado a seguir:

(a1  an )
S n
2
5  602
S   200 
2
 S   5  602   100
 S  60700

Exemplo 10: para determinarmos a soma de 16  18  20  L  794  796  798,


procedemos assim como é demonstrado a seguir, tendo em vista que as grandezas
que estão sendo somadas formam a PA 16, 18, 20, L , 794, 796, 798 . Portanto, de-
terminamos a quantidade de termos dessa PA e, em seguida, determinamos a soma.
Pelo fato de que a1 = 16 , an = 798 e a razão é r = 2 , sabemos, por meio da
fórmula geral da PA, que:
132
an  am   n  m   r

UNICESUMAR
 798  16   n  1  2
 798  2n  14
 2n  784
 n  392

Assim, para determinarmos a soma dos 392 primeiros termos dessa PA, proce-
demos da seguinte maneira:
(a  a )
S  1 n n
2
(16  798)
S  392
2
814
S  392
2
 S  407  392
 S  1559544.

anotações















133
3
PROGRESSÃO
UNIDADE 4

GEOMÉTRICA

A progressão geométrica (PG) é outro tipo de sequência bastante comum e que,


de certa forma, temos contato com alguns exemplos, ainda, no Ensino Funda-
1 2 3
mental, em forma de potências do tipo 5  5, 5  25, 5  125, ..., o que resulta
na PG 5, 25,125, ... .
Caro(a) aluno(a), sendo assim, iniciaremos o nosso estudo sobre a PG.

Fórmula Geral da PG

Uma sequência numérica é denominada PG de primeira ordem, quando cada


um de seus termos, a partir do segundo, for produto do anterior e da razão  q 
diferente de 0 . Consequentemente, cada um de seus termos em uma PG, a partir
do penúltimo, é igual ao quociente do termo posterior e da razão dessa sequência,
a qual é representada por q .

134
Exemplo 11: na PG  2, 6, 18, 54, 162  , temos a1 = 2 e q = 3 . Assim:

UNICESUMAR
a1  2;
a2  a1  q  2  3  6;
a3  a2  q  6  3  18;
a4  a3  q  18  3  54;
a5  a4  q  54  3  162,

O que equivale a:

a1  2;
a2  a1  q  2  3  6;
a3  a2  q   a1  q   q  a1  q 2  2  32  2  9  18;

 
a4  a3  q   a2  q   q  a1  q2  q  a1  q3  2  33  2  27  54;

a5  a4  q   a3  q   q   a1  q3   q  a1  q 4  2  34  2  81  162.

Exemplo 12: caso queira, podemos determinar o a9 (nono termo) da


PG 3, 6, 12, ... , assim como é demonstrado a seguir:
a1 3;
a2 a1 q 3 2 6;
a3 a2 q 6 2 12;
...

a8 a7 q 192 2 384;
a9 a8 q 384 2 768.

Exemplo 13: considerando que o décimo segundo termo  a12  de uma PG é


igual a 45927 e sua razão é igual a 3 , podemos determinar o sétimo termo, caso
assim preferir, da seguinte maneira:

135
a12 = 45927;
UNIDADE 4

a12 45927
a=
11 = = 15309;
q 3
a11 15309
a=
10 = = 5103;
q 3
a10 5103
a9 == = 1701;
q 3
a9 1701
a=
8 = = 567.
q 3

Já pensou em determinar o a30 (trigésimo termo) por meio do procedimento


anterior? Quanto tempo levaríamos para determinarmos esse termo e qual é a
probabilidade de não haver erros?
Para minimizarmos o erro e otimizarmos o nosso tempo, recorremos à fór-
mula a seguir para determinarmos o último termo procurado de uma PG:

an  am  q nm .

Sendo:

an : último termo em estudo.


am : primeiro termo em estudo.
m : índice do primeiro termo em estudo.
n : índice do último termo em estudo.
q : razão ou quociente .

Demonstração: considere n > m e seja an  an1  q . Pelo fato de que sabemos


que an1  an2  q , substituiremos an−1 por an2  q na fórmula anterior. Dessa
forma, temos:
an  an1  q   an2  q   q  an2  q2 .

De maneira análoga, sabemos que an2  an3  q . Logo, substituindo an−2 por
an3  q , temos:
an  an2  q2   an3  q  q2  an3  q3 .

Repetindo esse processo, temos:


136
an  ani  qi , i   .

UNICESUMAR
Além disso, como o termo  n  m  indica o último termo menos o primeiro, ou
seja, o i - ésimo termo, sabemos que an( nm)  ann m  am .
Portanto:
an  am  q nm

Observação: por meio dessa fórmula, podemos determinar, além do último


termo em estudo, o primeiro termo, a quantidade de termos em estudo e a razão
da PG em estudo, desde que essa sequência apresente termos consecutivos ou a
quantidade de termos em estudo for par.

Observação: para o caso em que m = 1 , ou seja, o primeiro termo a ser analisado


é o primeiro termo da sequência, sabemos que a fórmula geral pode ser reescrita
n 1
da seguinte forma: an  a1  q .

Exemplo 14: podemos proceder da seguinte forma para determinarmos o oitavo


termo de uma PG de razão igual a 3 e o primeiro termo igual a 5. Identificando
os termos, temos:
a= 8 ?
n a=
am= a=
1 5
q=3

nm
Pelo fato de que a fórmula geral da PG é an  am  q , temos:

a8  a1  q81
a8  5  37
a8  5  2187
a8  10935.

Observação: para determinarmos a razão de uma PG de primeira ordem, basta


dividir qualquer termo por um termo imediatamente anterior a ele.

137
Exemplo 15: considerando que o vigésimo termo  a20  de uma PG é igual a
UNIDADE 4

6144 e a sua razão é igual a 2 , podemos proceder como a seguir para determi-
narmos o décimo quarto termo  a14  dessa PG:

Identificando os termos, temos:

a=
n a=
20 6144
a= 14 ?
m a=
q=2

nm
Pelo fato de que a fórmula geral da PG é an  am  q , temos:

6144  a14  22014


6144  a14  26
6144  a14  64
6144
 a14 
64
 a14  96

Soma dos termos consecutivos à PG

Já parou para pensar em quanto tempo levaria para efetuar a seguinte adição:
7  21  63  ...  137781  413343  1240029 ? Se alguém respondeu que a soma re-
sultaria 1860040 em pouco tempo, provavelmente, essa pessoa possui conheci-
mento da regra da soma dos termos consecutivos de uma PG.
A fórmula para determinarmos a soma dos termos consecutivos da PG é
dada por:
(1  q n )
Sn  a1  , q 1
(1  q )

138
Em que:

UNICESUMAR
Sn : soma dos n termos consecutivos.
a1 : primeiro termo.
q : razão da PG.
n : índice do último termo a ser somado.

Demonstração: para realizarmos essa demonstração, consideraremos a PG


 a1 , a2 , a3 , a4 , , an2 , an1 , an , . Seja Sn , a soma dos n primeiros termos
da PG, logo, temos:

Sn  a1  a2  a3    an1  an (I)

Agora, multiplicaremos todos os termos da soma  I  pela razão q . Assim:

Sn  q  a1  q  a2  q  a3  q    an1  q  an  q ( II )

Entretanto sabemos que podemos reescrever cada termo como an  an1  q .


Portanto, faremos esse rearranjo dos termos, começando pelo a2  a1  q :

Sn  q  a1  q  a2  q  a3  q    an1  q  an  q

a2
Sn  q  a 2  a 2  q  a3  q    an1  q  an  q

a3
Sn  q  a2  a 3  a3  q    an1  q  an  q ( III )

Sn  q  a2  a3  a3    a n-1  q  an  q

an
Sn  q  a2  a3  a3    a n  an  q

Agora, subtrairemos as equações ( III ) e ( I ) , isto é, faremos ( I ) − ( III ) :

Sn  Sn  q  a1  an  q
 Sn 1  q   a1  an  q ( IV )

139
Para podermos finalizar, precisamos nos recordar do termo geral de uma PG, o
UNIDADE 4

qual é dado por:


qn
an  a1  q n1  a1 
q

Se multiplicarmos a fórmula por q , teremos:


an  q  a1  q n

Substituindo essa equação em ( IV ) , temos:

Sn 1  q   a1  a n  q
Sn 1  q   a1  a1  q n


Sn 1  q   a1 1  q n 
(1  q n )
 Sn  a1 
(1  q )

Exemplo 16: para determinarmos a soma dos 9 primeiros termos da PG


 5,10, 20, , podemos proceder assim como é demonstrado a seguir:
Identificando os termos, temos:
S9 = ?
a1 = 5;
q = 2.

Logo:
(1  q n )
Sn  a1 
(1  q )

 S9  5
1  29 
1  2 
 S9  5
1  512 
1
 S9  5  511
 S9  2555.
140
UNICESUMAR
pensando juntos

Na soma dos termos consecutivos da PG, temos q ≠ 1. Você já pensou no motivo dessa
condição?

Exemplo 17: podemos proceder assim como é demonstrado a seguir para


determinarmos a soma de 7 + 21 + 63 +  + 137781 + 413343 + 1240029, tendo
em vista que as parcelas a serem somadas formam uma PG. Para isso, precisa-
mos determinar n:

Identificando os termos, temos:

an = 1240029;
a1 = 7;
21
q
= = 3.
7

n 1
Pelo fato de que an  a1  q , temos:

1240029  7  3n1
 1240029  7  3n  31
7  3n
 1240029 
3

3
Multiplicando ambos os membros por , temos:
7

3 3n  7 3
 1240029   
7 3 7
 531441  3n

No entanto, fatorando 531441 , temos:

141
 312  3n
UNIDADE 4

 n  12

Prosseguindo:
S12 = ?
a1 = 7;
q = 3.

Logo:
(1  q n )
Sn  a1 
(1  q )

 S12  7
1  312 
1  3 
 S12  7 
1  531441
2
3720080
 S12 
2
 S12  1860040.

conecte-se

Caro(a) aluno(a), para melhor compreensão, assista ao vídeo no QRCode, o


qual apresenta a resolução desse problema.

Exemplo 18: podemos proceder do modo como é apresentado a seguir para


determinarmos a soma dos 7 primeiros termos da PG  4, 20, 100, 500, :

142
Identificando os termos, temos:

UNICESUMAR
S7  ?
a1  4;
20
q  5
4
Pelo fato de que a fórmula da soma dos termos consecutivos da PG é
(1  q n )
Sn  a1  , substituindo os valores, temos:
(1  q )

S7  4
1  (5)7 
1  (5) 
 4
1  (78125) 
6
78126
 4
6
 S7  52084

explorando Ideias

Mesmo que a maioria da soma dos infinitos termos das sequências seja convergente

para ou , a soma de infinitos termos dessa PG, em que é verificada a condição


1 q 1 , converge a um número, tendo em vista que q n converge a 0 quando q
aproxima de . Assim, quando 1 q 1 , a soma de infinitos termos da PG pode ser
a1
determinada por S .
1 q

Para mais informações, acesse: https://waldexifba.wordpress.com/material-de-apoio/en-


sino-medio/p-g/formula-da-soma-dos-termos-de-uma-pg-infinita/.
Fonte: os autores.

143
CONSIDERAÇÕES FINAIS
UNIDADE 4

Caro(a) aluno(a), chegamos ao final de mais uma unidade. Nela, aprofundamos o


nosso estudo sobre a progressão aritmética (PA) e a progressão geométrica (PG),
ambas de primeira ordem. Iniciamos com a caracterização de cada uma delas,
tendo em vista que a identificação desse tipo de sequências é fundamental para
a escolha do processo de sua resolução.
Após a caracterização, assim como ocorreu nas unidades anteriores, avan-
çamos para a demonstração de fórmulas, seguida de alguns exemplos, a fim de
concretizar a aquisição de conhecimentos. A apresentação das demonstrações
dessas fórmulas possui uma importância muito grande e não se mantém apenas
para essa disciplina: é algo que deve se tornar comum durante todo o curso de
Matemática e toda a sua carreira acadêmica.
Do mesmo modo como fora frisado nos demais conteúdos estudados nas
unidades anteriores, devemos tomar alguns cuidados para não cometermos
erros. Além disso, para resolvermos os exercícios que envolvem esses tipos de
sequências, muitas vezes, recorremos aos conhecimentos já adquiridos, como o
das equações do 1º grau.
Dessa forma, aqueles que não dominam os conteúdos já estudados, inde-
pendentemente do nível de ensino, podem encontrar dificuldades em resolver
algumas situações-problemas apresentadas. Em outras palavras, as sequências de
conteúdos apresentados na matemática foram pensadas para alcançar níveis cada
vez mais elevados de conhecimentos e, pelo fato de que os conteúdos devem ser
assimilados, e não decorados, precisamos nos preocupar com as demonstrações.
Portanto, pensar em outra possibilidade de demonstração de algumas fórmulas,
como a da soma dos termos de uma PA, pode ser uma boa opção para mudar a
sua maneira de estudar a matemática.
Caro(a) aluno(a), esperamos ter contribuído com a sua formação.

144
na prática

1. Observe as sequências a seguir:

I -  33, 16, 1, 18,


1 1 1 1 
II -  , , , , 
2 3 4 5 
III - 141,137,133,129 

IV - 1, 2, 4, 7 
São exemplos de PA de primeira ordem:

a) I e II.
b) I e III.
c) I e IV.
d) II e III.
e) II e IV.

2. Uma pessoa, ao determinar a razão da PA finita de 21 termos cujo primeiro é 37


e o último é 457 , procedeu da seguinte maneira:
457  37
r
21
420
r
21
 r  20
No entanto o resultado obtido está incorreto. Onde está o erro?

3. Verificou-se que a soma de n primeiros termos de uma PA finita é igual a 205, a1 = 7


e r = 3 . Determine o termo an dessa PA.

4. Meu amigo pegou uma madeira e a cortou em 7 pedaços para serem os sete de-
graus de uma escada. Considerando que o comprimento dos degraus forma uma
PA, bem como o fato de que o primeiro degrau mede 80 cm e o último mede 50

145
na prática

cm, qual deve ser o comprimento mínimo de madeira, se a cada corte que fizer na
madeira perde-se 0, 5 cm?

5. Considerando que x + 4, 2 x + 7 e 4x + 6 são os três primeiros termos de uma


PA infinita, escreva essa sequência.

6. Determine a razão de uma PA que possui, como termos consecutivos, os valores


50 x  4, 10 x2  4 e 52 x + 2 , sabendo que essa razão é um número natural.

7. Observe atentamente as sequências a seguir:

I -  256, 128, 64, 32,


1 1 1 1 
II -  , , , , 
2 3 4 5 
III -  6,12, 24, 36 
1 1 1 1 
IV -  , , , 
 3 9 27 81 
São exemplos da PG de 1ª ordem:

a) I e II.
b) I e III.
c) I e IV.
d) II e III.
e) II e IV.

8. Uma pessoa, ao somar os 10 primeiros termos da PG cujo a1 = 5 e cuja q=2,


esqueceu de adicionar a5 . Qual é a soma encontrada?

9. Quantos valores múltiplos de 4 existem de 1 a 85968 ?

10. O terceiro termo de uma progressão geométrica é 10 e o sexto termo é 80 . En-


contre a sua razão.

146
aprimore-se

Para resolver situações-problemas que envolvem PA e PG, a calculadora reduz con-


sideravelmente o tempo de cálculo. No entanto alguns professores ainda resistem
quanto ao uso dessa ferramenta na sala de aula. Assim, surge a necessidade de
analisar o uso da calculadora como recurso didático. Vamos lá:
A calculadora tem como principal finalidade a resolução de operações de forma
rápida e eficiente. Rivaliza com o cálculo escrito, com os algoritmos, os procedimen-
tos de cálculo. Por incômodo que possa parecer, uma simples maquininha de R$
1,99 substitui todo o ensino das continhas, que não raro, ocupa uma grande parte
do tempo de sala de aula nos diversos níveis de instrução. Para que possamos nos
convencer da utilidade das maquininhas de calcular em sala de aula, devemos, an-
tes, repensar a utilidade dos algoritmos.
Antes de iniciarmos a discussão, devemos nos perguntar: o que é um algorit-
mo? De uma forma simplificada podemos afirmar que nada mais é do que uma
sequência de passos para se realizar determinada tarefa. No âmbito da matemáti-
ca utilizamos o termo algoritmo para designar um procedimento que nos permite
resolver uma operação.
Constitui-se crença infundada a afirmação de que com o uso da calculadora os
alunos deixariam de raciocinar. Ora, ao realizar milhares de contas na mão, o aluno
não estará fazendo mais que exercitar um procedimento, não estará raciocinando.
É possível, inclusive, e isso não é raro, que mesmo fazendo contas com perfeição o
aluno não tenha a menor ideia de porquê de tal algoritmo funcionar.
Infelizmente, na escola, enfatiza-se o cálculo escrito como se dele dependesse
o desenvolvimento da habilidade de cálculo do aluno. É importante salientar que a
habilidade de cálculo está sustentada por quatro pilares:

1. Escrito: realizar as quatro operações com compreensão.


2. Estimado: saber fazer estimativas.
3. Na calculadora: saber operar com destreza uma máquina de calcular.
4. Mental: saber realizar contas sem utilização de papel ou máquina.

147
aprimore-se

A calculadora, além de ser um dos pilares, pode ser útil no desenvolvimento dos
outros. Embora possa parecer estranho, sua utilização pode auxiliar o aluno a com-
preender melhor os algoritmos.
Do nosso ponto de vista, é lamentável que muitos professores dediquem um tem-
po enorme para ensinar algo que pode ser substituído por uma máquina, deixando de
abordar problemas reais, apenas porque os alunos “ainda não sabem as continhas”.
É incoerente com qualquer propósito educacional alunos que terminem o en-
sino médio dominando as quatro operações não conseguirem decidir entre duas
ofertas, por exemplo, se é melhor comprar um pacote de fraldas com 40 unidades a
R$ 15,00 ou um pacote com 30 unidades a R$ 13,00. Em classes menos favorecidas,
uma decisão como essa pode significar um pedaço de carne a mais na mesa.
Além desse aspecto utilitário, a calculadora pode ser utilizada para explorar con-
ceitos matemáticos em que seu próprio uso é o efeito problematizador.
Assim como qualquer recurso didático, o uso da calculadora requer planejamen-
to e objetivos claros. Do nosso ponto de vista, existem dois momentos em que ele é
recomendável e necessário: quando tratamos de números reais e nas explorações
de conceitos matemáticos.

Fonte: Rolkouski (2013).

148
eu recomendo!

livro

A Matemática do Ensino Médio


Autor: Elon Lages Lima, Paulo Cezar Pinto Carvalho, Eduardo
Wagner e Augusto César de Oliveira Morgado
Editora: SBM
Sinopse: este livro, escrito para professores do Ensino Médio e
estudantes de licenciatura em Matemática, aborda os principais
assuntos estudados na segunda série do Ensino Médio. Assim,
a obra possui duas partes bem distintas. A primeira é dedicada à Matemática
Discreta, contendo o estudo de Progressões (com aplicações à Matemática Finan-
ceira), entre outros. Já a segunda parte do livro é dedicada à Geometria Espacial e
tem duas preocupações principais.
Comentário: ler o capítulo 1.

conecte-se

Para compreender um pouco sobre termos e soma da progressão aritmética de or-


dem superior:
https://issuu.com/logusmao/docs/pa_de_ordem_n

Conheça a proposta de ensino de progressões aritméticas de ordem superior, no En-


sino Médio, apresentada por Wilhian Alexander Ferreira Lima:
https://sca.profmat-sbm.org.br/sca_v2/get_tcc3.php?id=93375

149
5
NÚMEROS
COMPLEXOS

PROFESSOR
Me. Tiago Peres da Silva Suguiura

PLANO DE ESTUDO
A seguir, apresentam-se as aulas que você estudará nesta unidade: • Tópico do plano 01 • Tópico do
plano 02 • Tópico do plano 03 • Tópico do plano 04.

OBJETIVOS DE APRENDIZAGEM
• Definir unidade imaginária e números complexos • Compreender a representação de um número
complexo e suas propriedades na forma algébrica • Definir números complexos e suas propriedades na
forma geométrica • Identificar a representação de um número complexo em sua forma trigonométrica
ou polar e compreender as suas propriedades.
INTRODUÇÃO

Caro(a) aluno(a), bem-vindo(a) à quinta unidade! Até agora, você sempre


soube que não poderia calcular a raiz quadrada de um número negativo. Ao
encontrar as raízes de uma equação do segundo grau cujo delta era menor
do que zero , dizíamos não haver raízes reais. Isso se dá, pois não havia
números que fossem negativos após elevá-los ao quadrado. Todo número
era positivo após essa operação. Agora, entretanto, você pode calcular a
raiz quadrada de um número negativo, mas esse processo envolve o uso
de um novo número.
Nesta unidade, definiremos e compreenderemos os números complexos
e suas características. É importante que você, caro(a) aluno(a), consiga com-
preender todas as propriedades e aplicações dos números complexos em di-
ferentes contextos. Em muitos aspectos, esse estudo não se difere de qualquer
outro conjunto, mas há muitas particularidades e é preciso estar atento(a).
Iniciaremos a nossa unidade com a definição de um número complexo,
passando por algumas operações e a definição de conjuntos dos números
complexos. Depois, será introduzido o conceito de unidade imaginária,
suas implicações no conjunto e algumas formas de representação de nú-
meros complexos.
Dentro das diferentes formas de representação, iniciaremos com
a sua forma algébrica, passando por sua representação geométrica e fina-
lizando com a sua forma trigonométrica. Lembre-se que, para cada repre-
sentação, existem diversas propriedades e conceitos.
Além disso, será afirmado que os números complexos são formados
por duas partes: a parte real e a parte imaginária. A parte real é formada
por um número dentro do conjunto dos números reais, enquanto a parte
imaginária é um conceito mais abstrato e que nos permite realizar a raiz
quadrada de um número negativo.
O surgimento dos números complexos é importante para as áreas
mais aplicadas da matemática ou até mesmo na álgebra. Eles podem ser
aplicados em muitos aspectos do dia a dia, especialmente nos aparelhos
eletrônicos e no eletromagnetismo.
1
NÚMEROS
UNIDADE 5

COMPLEXOS

Iniciaremos o nosso estudo definindo o que é um número complexo. Para tanto,


ele será denotado por z , que é um par ordenado  x, y  de números reais. A
partir dessa definição, já temos a noção de que o conjunto dos números reais
está contido no conjunto dos números complexos. No entanto qual é o conjunto
dos números complexos?
Chamamos de conjunto dos números complexos (e denotamos por  ), o conjunto
dos pares ordenados de números reais – em símbolos,temos:    z   x, y  | x, y  
– para os quais estão definidas as seguintes propriedades:

I - Igualdade de dois números complexos:

Sejam z1   a, b  e z2   c, d  . Temos:
z1  z2   a, b    c, d   a  c e b = d

II - Adição de números complexos:

Sejam z1   a, b  e z2   c, d  . Temos:
z1  z2   a, b    c, d    a  c, b  d 

152
III - Multiplicação de números complexos:

UNICESUMAR
Sejam z1   a, b  e z2   c, d  . Temos:

z1  z2   a, b    c, d    ac  bd , ad  bc  .

pensando juntos

Como se relaciona o conjunto dos números complexos  em relação aos demais conjun-
tos conhecidos, , , I, , ?

Exemplo 1: sejam z1   5, 3  e z2  1, 1 dois números complexos, determine


a soma z1 + z2 e o produto z1 ⋅ z2 .
Encontraremos, primeiramente, a soma. Assim:

z1  z2   5, 3   1, 1
=  5  1, 3   1 
 z1  z2   6, 2 

Agora, o produto:

z1  z2   5, 3   1, 1
  5  1  3   1 , 5   1  3  1
  5  3,  5  3 
 z1  z2   8, 2 

Observação: note que um número complexo z   x,0  é identificado com o


número real x . Isso significa que o conjunto dos números reais é um subcon-
junto dos números complexos      . E se tivermos, porém, um número
complexo z1   0, x  ? O que difere o conjunto dos números complexos do
conjunto dos números reais? É nesse momento que introduzimos o conceito
de unidade imaginária.
153
Unidade imaginária
UNIDADE 5

Chamamos de unidade imaginária e indicamos pela letra i , o número com-


plexo dado por  0, 1 . Sua propriedade básica é:
i 2  1 .

De fato, temos:

i 2  i  i   0, 1   0, 1   0  0  1  1, 0  1  1  0    1, 0   1 .

Dessa propriedade, surge um fato muito utilizado nos números complexos e que
o diferencia dos números reais: i  1 .

2
Exemplo 2: resolva a equação do segundo grau z  2 z  2  0 .

Encontraremos as raízes dessa equação, utilizando a Fórmula de Bháskara:

b  b2  4  a  c
z
2a
2  4  4  1  2

2
2  4  (1)

2
2  4  i 2

2
2  2  i

2
 z  1  i

Logo, sabemos que as raízes são z1  1  i e z2  1  i .

154
2
FORMA

UNICESUMAR
ALGÉBRICA

Assim como já fora exposto, um número complexo pode ser escrito de algumas
formas distintas. A primeira maneira de se representar um número complexo
será segundo a sua forma algébrica.
Todo número complexo z   x, y  pode ser escrito na sua forma algébrica como:
z = x +yi

tal que i é a unidade imaginária.


Note que, se z = ( x, y ) , temos:
 x, y    x, 0    0, y    x, 0   y,0 × 0,1  x  yi .
Observe que a parte destacada em negrito é dada por:
 y, 0    0,1   y  0  0 1, y 1  0  0    0, y  .
Por outro lado:
 0,1   y, 0    0  y  1  0, 1  y  0  0    0, y  .
Observação: o número real x é chamado de parte real de z e o número real y
é chamado de parte imaginária de z . Em símbolos, temos:
x = Re( z ) e y = Im( z ) .
155
Exemplo 3: para o número complexo z  2 x  5i , sabemos que Re( z ) = 2 e
UNIDADE 5

Im( z )  5 .
Note que, sob a perspectiva da forma algébrica, as propriedades dos números
complexos são mais práticas do que por meio do uso do par ordenado exibido.
De acordo com Dias (2014), os números complexos são objetos coerentes, isto
é, a soma de dois números complexos é um número complexo, assim como o
produto. Veremos como ficam essas propriedades de igualdade, adição e multi-
plicação de números complexos sob a forma algébrica.

I - Igualdade de dois números complexos:


Sejam z1  a  bi e z2  c  di . Temos:

z1  z2  a  bi  c  di  a  c e b = d .

II - Adição de números complexos:


Sejam z1  a  bi e z2  c  di . Temos:

z1  z2   a  bi    c  di    a  c    b  d  i .

III - Multiplicação de números complexos:


Sejam z1  a  bi e z2  c  di . Temos:

z1  z2   a  bi    c  di    ac  bd    ad  bc  i .

Exemplo 4: dados z1  1  i e z2  2  i , calcularemos:

I - z1 + z2
z1  z2  1  i    2  i 
 1  2   1  1 i
 3  2i
II - z1 − z2
z1  z2  1  i    2  i 
 1  2   1  1 i
 1
156
UNICESUMAR
3
III - 2 z1 + z2
5
3 3
2 z1  z2  2 1  i    2  i 
5 5
 3   3
  2 1   2    2 1   i
 5   5
 6  3
 2    2  i
 5  5
16 13
  i
5 5
IV - z1 ⋅ z2
z1  z2  1  i   (2  i )
 1  2  1  1  1  1  1  2  i
 1  3i

Propriedades dos números complexos

A seguir, exibiremos algumas propriedades muito importantes para os números


complexos.

a) Leis comutativas para a adição e a multiplicação:


Sejam z1 , z2 ∈  , há:

· z1  z2  z2  z1

De fato, dados z1  a  bi e z2  c  di , temos:

z1  z2   a  bi    c  di 
  a  c   b  d  i
 c  a  d  b i
  c  di    a  bi 
 z2  z1
· z1  z2  z2  z1
157
Essa prova fica como exercício.
UNIDADE 5

b) Leis associativas para a adição e a multiplicação:


Sejam z1 , z2 , z3 ∈  , há:

· z1   z2  z3    z1  z2   z3

Sejam z1  a  bi, z2  c  di e z3  e  fi , então:

z1   z2  z3    a  bi    c  di    e  fi  
  a  bi    c  e    d  f  i 
  a  c  e  b  d  f  i
  a  c    b  d  i    e  fi 
  z1  z2   z3

· z1   z2  z3    z1  z2   z3

Fica como exercício essa prova.

c) Lei distributiva da multiplicação em relação à adição:


Sejam z1 , z2 , z3 ∈  , há:

· z1   z2  z3   z1  z2  z1  z3

De fato, sejam z1  a  bi, z2  c  di e z3  e  fi , então:

Conjugado de um número complexo

Já sabemos que um número complexo é formado pela parte real e pela parte
imaginária. Denomina-se conjugado de um número complexo z  a  bi , o
número complexo z  a  bi . Note que a mudança ocorre apenas no sinal da
parte imaginária. Após alguns exemplos, apresentaremos uma aplicação para o
conjugado de um número complexo.
158
Exemplo 5: determine o conjugado de cada um dos números complexos a seguir:

UNICESUMAR
a) z  2  7i
Se z  2  7i , sabemos que z  2  7i .

b) z  2  4i
Se z  2  4i , sabemos que z  2  4i .

c) z  5  3i
Se z  5  3i , sabemos que z  5  3i .

d) z  7  i
Se z  7  i , sabemos que z  7  i .

Observação: note que, se multiplicarmos um número complexo pelo seu conju-


gado, obteremos somente a parte real do número complexo. Em outras palavras,
se z  a  bi e z  a  bi , temos:
z  z   a  bi    a  bi 
  a  a  b   b     a   b   b  a  i

 
 a2  b2   ab  ba  i

Como a, b ∈  , sabemos que ab = ba . Portanto:

 z  z  a 2  b2 .

Com essa informação,podemos introduzir o conceito de divisão entre números complexos.

Divisão entre números complexos

z1
Para calcularmos , z2 ≠ 0 , basta multiplicarmos o numerador e o denominar
z2
pelo conjugado do denominador. Isso transformará essa divisão em uma divisão

por um número real.


159
Exemplo 6: dados z1  a  bi e z2  c  di  0 , temos:
UNIDADE 5

z1 z1 z2 (a  bi ) (c  di )   ac  bd    bc  ad  i 
     .
z2 z2 z2 (c  di ) (c  di ) c2  d 2

Exemplo 7: sejam z1  3  2i e z2  1  i , calcularemos a sua divisão:

z1 3  2i

z2 1  i
3  2i 1  i
 
1 i 1 i
(3  2)  (3  2)i

(1  1)  (1  1)
5i

2
5 1
  i
2 2

Potências de i
2
Assim como já fora exposto, i  1 , o que mostra um comportamento interes-
sante, caso aumentemos o valor da potência. Vejamos a seguir:

■ i0 = 1
■ i1 = i
■ i 2  1
■ i 3  i 2  i  1  i  i
■ i 4  i 3  i   i   i  i 2    1  1
■ i5  i 4  i  1  i  i
■ i 6  i 5  i  i  i  i 2  1
■ i 7  i 6  i  1  i  i
■ i 8  i 7  i  i  i  i 2    1  1

160
Podemos notar que os resultados se repetem em ciclos de 4 .

UNICESUMAR
Observação: pelo fato de que os resultados se repetem em ciclos de 4 , pode-
n 4qr
mos escrever esse resultado como i  i  i r , r  4 , em que r é a divisão
de n por 4 .

Exemplo 8: calcule:

23 3
a) i  i  i

Primeiramente, faça a divisão de 23 por 4 . O resto dessa divisão é 3 , isto é,


23  4  5  3 . Logo, basta utilizar o valor do resto da divisão.

100 0
b) i = i = 1

Note que 100 dividido por 4 dá resto 0 , isto é, 100  4  25 0 .

conecte-se

Caro(a) aluno(a), para melhor compreensão, assista ao vídeo no QRCode, o


qual apresenta a resolução desse problema.

161
3
REPRESENTAÇÃO
UNIDADE 5

GEOMÉTRICA

A primeira maneira exibida de se ilustrar um número complexo é por meio


de um par ordenado z   a  b  , tal que a, b ∈  . Isto é,  a, b       
2

, que é o plano cartesiano.


Então, considere um sistema de coordenadas cartesianas ortogonais xOy e
um ponto p de coordenadas  a, b  . Podemos notar que há uma forte relação
entre os pontos do plano e os números complexos.
Dessa forma, a representação geométrica de um número complexo
z   a, b   a  bi é feita em um plano cartesiano xOy de coordenadas or-
togonais (ou retangulares), denominado Plano Complexo (ou Plano de
Argand-Gauss).

162
UNICESUMAR
Eixo imaginário P
b z = (a, b) = a + bi

ρ = |z|
b = ρ  sen(θ)

θ Eixo real
Ο a = ρ  cos(θ) a

Figura 1 – Exemplo de plano complexo / Fonte: o autor.

De acordo com o plano complexo, alguns elementos são definidos. Entre eles, estão:

■ O eixo Ox é o eixo com a parte real.


■ O eixo Oy é o eixo imaginário.

Módulo de um número complexo

O módulo de um número complexo z é a distância entre a origem e o ponto


P   a, b  . Além disso, é representado por r = z , tal que r  a2  b2 , resul-
tado que segue, diretamente, do Teorema de Pitágoras.

Exemplo 9: determine o módulo de z  3  4i .

r  32  42  9  16  25  5 .

163
Argumento de um número complexo
UNIDADE 5

Chama-se argumento de um número complexo z  a  bi, z   0, 0  , o ângu-


lo q em radianos  0  θ  2π  , o qual é formado pelo vetor entre o ponto de
origem e o ponto  a, b  , com o semieixo real positivo tal que:

a a
cos θ   ;
ρ 2
a b 2

b b
senθ   .
ρ a 2  b2

Exemplo 10: determine o argumento do número complexo z  1  3i .

 3
2 2
De fato, temos a = 1 e b = 3 e r  1   1 3  4  2 .

Logo:

a 1 
cos θ  
ρ 2  π
  θ  rad  60”
b 3 3
senθ  
ρ 2 

π
Portanto, arg z= θ= rad = 60º .
3

164
4
REPRESENTAÇÃO TRIGONOMÉTRICA

UNICESUMAR
OU POLAR

A terceira forma de representação de um número complexo que será apresen-


tada é sob a forma trigonométrica ou polar. Para definirmos essa representação,
partiremos da forma geométrica já apresentada.
Dado um número complexo z  a  bi não nulo, com argumento q , temos:

a
cos θ   a  ρ  cos θ
ρ
e

b
senθ   b  ρ  senθ .
ρ

Portanto, z  a  bi  a  ib  ρ  cos θ  iρ  senθ . Isto é:

z  ρ  cos θ  i  senθ  .

Essa é a forma trigonométrica ou polar de representação de um número complexo z .

165
Observação: substituindo q na forma trigonométrica por  θ  2k π  , tal que
UNIDADE 5

k ∈  , o número complexo z não se altera, pois os argumentos q e  θ  2k π 


são congruentes. Em outras palavras,  θ  2k π  representa um número inteiro
de voltas completas, tanto no sentido positivo quanto no sentido negativo, no
ciclo trigonométrico.
Assim, é comum usar a forma trigonométrica de um número complexo de
uma maneira geral, a qual é dada por:

z  ρ   cos  θ  2k π   i  sen  θ  2k π   .

Para esse caso, os valores de  θ  2k π  são os argumentos de z e q é dito argu-


mento principal de z .

Exemplo 11: obtenha a forma trigonométrica do número complexo z  3i e,


na sequência, faça a sua representação geométrica.

2 2
De fato, temos: a = 0 e b  3 . Logo, r  0  3  9  3 .

Portanto:
a 0 
cos θ  0 
ρ 3  3π
θ  rad  270” .
b 3 2
senθ    1
ρ 3 

Assim, temos:

 3p 3p 
z  3   cos  i  sen   3   cos 270º i  sen270º  .
 2 2 

Podemos ver a sua representação gráfica na figura a seguir:

166
UNICESUMAR
y

x

θ= rad = 270°
2
ρ=3

z = –3i

Figura 2 – Representação gráfica / Fonte: o autor.

Multiplicação de números complexos

Agora, veremos que existe uma interpretação geométrica para a multiplicação


de números complexos. Para isso, consideraremos dois números complexos
z1 , z2 tais que:

z1  ρ1   cos θ1  i  senθ1 

z2  ρ2   cos θ2  i  senθ2  .

Fazendo z1 ⋅ z2 , obtemos:

167
z1  z2  ρ   cos θ1  i  senθ1   ρ2   cos θ2  i  senθ2 
UNIDADE 5

 ρ1  ρ2   cos θ1  cos θ2  cos θ1  i  senθ2    i  senθ1  cos θ2  i  senθ1  i  senθ2  


 ρ1  ρ2   cos θ1  cos θ2  cos θ1  i  senθ2   i  senθ1  cos θ2  i 2  senθ1  senθ2 
  
 ρ1  ρ2   cos θ1  cos θ2  cos θ1  i  senθ2  i  sen‚ 1  cos θ2  senθ1  senθ2 
 ρ1  ρ2   cos θ1  cos θ2  senθ1  senθ2   i   cos θ1  senθ2  senθ1  cos θ2  

Todavia, sabemos que:

cos  q1  q2   cos q1  cos q2  senq1  senq2

sen  q1  q2   cos q1  senq2  senq1  cos q2 .

Portanto:

z1  z2  ρ1  ρ2  cos  θ1  θ2   i  sen  θ1  θ2   .

Observação: note que arg  z1  z2   arg  z1  z2  .


Geometricamente, podemos representar o módulo do número complexo
z1  z2  r1  r2 por:

168
y z1z2

UNICESUMAR
z2

θ1
z1
θ1 θ2
O x
Figura 3 – Representação geométrica do módulo do número complexo z1  z2 / Fonte: o autor.

Observação: em particular, quando um número complexo z é multiplicado por


i , significa realizar, no ponto que representa o número complexo z , uma rotação
p
positiva de rad = 90º .
2

Divisão de números complexos

Da mesma forma que podemos fazer a multiplicação de números complexos sob


a ótica trigonométrica, o mesmo processo pode ser feito para a divisão.
Sejam z1  ρ1   cos θ1  i  senθ1  e z2  ρ2   cos θ2  i  senθ2  com ri ≠ 0 .
Fazendo a divisão, obtemos:

z1 ρ1
w   cos  θ1  θ2   i  sen  θ1  θ2   .
z2 ρ2 

De fato, para demonstrarmos esse resultado, devemos mostrar que, ao mul-


tiplicar w por z2 , obtém-se z1 :

169
ρ1
w z2 cos θ1 θ2 i sen θ1 θ2 z2 ρ2 cos θ2 i senθ2
UNIDADE 5

ρ2

Já aprendemos como é feita a multiplicação entre dois números complexos. Logo, temos:

ρ1
w  z2   ρ2  cos  θ1  θ2  θ2   i  sen  θ1  θ2  θ2  
ρ2
 ρ1   cos θ1  i  senθ1 
 z1

Portanto:

z1 ρ1
  cos  θ1  θ2   i  sen  θ1  θ2   .
z2 ρ2 

z 
Observação: note que arg  1   arg z1  arg z2 .
 z2 

Potenciação de números complexos (De Moivre)

Dado o número complexo z  p   cos q  i  senq  não nulo e um número inteiro


n , temos:
z n  ρ n  cos  n  θ   i  sen  n  θ   .

De fato, provaremos em três etapas:

1. Para n = 0 ou n = 1 . A fórmula segue de maneira intuitiva:

z 0  ρ 0   cos  0  θ   i  sen  0  θ  

■  1   cos 0  i  sen0 
 1 1
1

170
UNICESUMAR
z1  ρ1   cos 1  θ   i  sen 1  θ  
■  ρ   cos θ  i  senθ 
z

2. Para n inteiro maior do que 1 . Decorre, diretamente, sobre a multiplica-


ção de números complexos exibida:

    
z n  
z z  z
  ρ  ρ  ρ   cos  θ  θ  θ   i  sen  θ  θ   
θ
          
 

n fatores n fatores   n fatores   n fatores  
 ρ n cos  n  θ   i  sen  n  θ  

*
3. Para n negativo. Vejamos: seja n   m , com m    :

n
z n  ρ   cos θ  i  senθ  
m
 ρ   cos θ  i  senθ  
1
 m
ρ   cos θ  i  senθ  
1

zm
1

ρ m   cos  m  θ   i  sen  m  θ   
 

Sabemos que  cos 0  i  sen0   1 . Logo:

171
 cos 0  i  sen0 
UNIDADE 5


ρ   cos  m  θ   i  sen  m  θ   
m
 
1
 m
 cos 0  i  sen0 
ρ  cos  m  θ   i  sen  m  θ  
1
  cos  0  m  θ   i  sen  0  m  θ  
ρm
 ρ  m  cos  m  θ   i  sen  m  θ  
 ρ n  cos  n  θ   i  sen  n  θ  

4
Exemplo 12: dado o número complexo z  3  i , calcule z .
2
=
Sabemos que a 3, b i e r 
= 3  11  4  2 . Assim:

a 3
cos θ   
ρ 2  π
  θ  rad  30”
b 1  6
senθ  
ρ 2  

Logo, temos:
z 4  24  cos  4  30º   i  sen  4  30º  
 16  cos 120º   i  sen 120º  
 1 3
 16     i  
 2 2 
 8  8 3i

Radiciação de números complexos

Seja z  ρ   cos θ  i  senθ  , z  0 , queremos calcular:

n
z  n ρ   cos θ  i  senθ  ,
172
Em que n ∈  tal que n ≥ 2 .

UNICESUMAR
O problema de calcular a raiz n-ésima de um número complexo z se dá na
resolução da equação:

wn  z  n z  w .

Fazendo w  ω   cos ϕ  i  senϕ  , temos:


n
ω   cos ϕ  i  senϕ    ρ   cos θ  i  senθ  .

Utilizando a fórmula de De Moivre:

wn  ω n  cos  n  ϕ   i  sen  n  ϕ   
 
 ρ   cos θ  i  senθ 

Pelo fato de que números complexos iguais possuem módulos iguais e argumen-
n
tos congruentes, sabe-se que: ω = ρ e n  ϕ  θ  2k π, k   .

Então:
θ  2k π θ 2k π
ω=nρ e ϕ   .
n n n

Logo:
w  ω   cos ϕ  i  senϕ 
  θ  2k π   θ  2k π  
 n ρ  cos    i  sen   .
  n   n 
Portanto:
w n z
  θ  2k π   θ  2k π  
 n ρ  cos    i  sen   .
  n   n 

173
Assim, é possível calcular n raízes distintas para z . Para tanto, basta fazer
UNIDADE 5

k  0, 1, 2, 3,  ,  n  1 , pois, a partir disso, elas se repetem. Vejamos:

0
■ k  0  j  , ( j é argumento de w );
n

θ 2π
■ k  1  ϕ  
n n

θ 4π
■ k  2  ϕ  
n n

θ 2(n  1)π
■ k  n  1  ϕ   .
n n

Note que esses valores de j não são congruentes, pois estão todos no interva-
lo 0, 2p  . Isso quer dizer que dão origem a n valores distintos para w .

Exemplo 13: calcule z , quando z  1 .

2
De fato, n  2, k  0.1, a  1 e b = 0 . Então, sabemos que r  (1)  0  1 .

Logo:
a 1 
cos θ   1
ρ 1 
  θ  πrad  180”.
b 0
senθ    0 
ρ 1 

Ou seja:
  p  2k p   p  2k p  
w  1  cos    i  sen  
  2   2 

Portanto, temos:

174
Para

UNICESUMAR
 p  p 
k  0  w0  1  cos    i  sen     1  0  i  1  i .
 2  2 

Para
 p  p 
k  1  w1  1  cos   p   i  sen   p  
 2  2 
  3p   3p  
 1  cos    i  sen   
  2   2 
 1  0  i   1 
 i

Assim, as raízes quadradas de z  1 são i e −i .

conecte-se

Caro(a) aluno(a), para melhor compreensão, assista ao vídeo no QRCode, o


qual apresenta a resolução desse problema.

explorando Ideias

Você conhece a fórmula de Euler? Ela é, talvez, uma das fórmulas matemáticas mais co-
nhecidas e é considerada a mais bela por muitos. A Fórmula de Euler, ou Identidade de
Euler, reúne os possíveis cinco números mais importantes da matemática: 0, 1, i, e, π.
Isso tudo em uma simples igualdade:

eiπ +1 = 0

A demonstração dessa fórmula pode ser acessada no link a seguir: https://www.obaricen-


trodamente.com/2010/07/demonstracao-da-identidade-de-euler.html.
Fonte: o autor.

175
CONSIDERAÇÕES FINAIS
UNIDADE 5

Caro(a) aluno(a), nesta unidade, tratamos um pouco sobre os números com-


plexos. Entretanto por que devemos estudar tais números e passar esse conhe-
cimento adiante?
Por mais de 2500 anos, matemáticos foram obcecados em obter as raízes
de equações polinomiais. Essa luta para encontrar as raízes de equações cada
vez mais complicadas é uma das grandes vitórias do pensamento humano.
Durante os primeiros anos dessa “batalha”, se matemáticos, no processo de re-
solução de uma equação, chegassem ao ponto que envolvesse a raiz quadrada
de um número negativo, tal pessoa simplesmente pararia o seu cálculo, pois tal
expressão não tinha sentido.
Após anos de muito estudo, as definições e as propriedades necessárias foram
completadas e, agora, temos a solução para as equações que produzem raízes
quadradas de números negativos, além de várias outras aplicações para os núme-
ros complexos, principalmente, no eletromagnetismo. Sendo assim, definimos o
conceito de número complexo no nosso livro, por meio da apresentação de suas
características principais e das suas várias formas de representação.
Começamos a unidade, identificando um número complexo sob a forma de
um par ordenado. Posteriormente, apresentou-se a sua forma mais usual, a forma
algébrica, por meio da exposição da característica fundamental dos números
complexos: a unidade imaginária. A partir desse momento, apresentaram-se ope-
rações entre números complexos e a ideia de conjugado de um número complexo
e as propriedades da potenciação da unidade imaginária.
Ao apresentar a representação geométrica dos números complexos no plano
complexo ou Argang-Gauss, introduziu-se o conceito de módulo e argumen-
to, definições importantes para as propriedades vindouras. Novamente, foram
expostas as operações entre números complexos sob a forma geométrica. Para
finalizar a unidade, ensinou-se a representação trigonométrica dos números
complexos, introduzindo as propriedades de potenciação (De Moivre) e a radi-
ciação de números complexos.
Embora esse assunto seja tratado com “medo” pelos professores ao transpô-lo
aos seus alunos, é importante perceber que suas propriedades são de fácil com-
preensão e que, às vezes, é melhor tratá-lo com mais esmero, a fim de captar a
atenção dos discentes.

176
na prática

1. Encontre as raízes da equação 2 z 2  8 z  16  0, z   .

2. Exiba o número complexo z  1  3i na forma de par ordenado, na forma geo-


métrica e na forma trigonométrica.

3. Calcule as seguintes potências de i:

a) i 33 .

b) i 999 .

4. Determine o conjugado dos seguintes números complexos:

1− i
a) .
i

1  2i
b) .
2i
 7a 
5. Determine o valor real de a para que o número complexo z    12   2i
 31 
seja um imaginário puro.

b 
6. Determine o valor real de b para que o número complexo z  5    3i seja
3 
um número real.

177
na prática

7. Seja z1  2i e z2  2  3i , calcule z1 + z2 e z1 ⋅ z2 .

8. Determine i 0 + i1 + i 2 + i 3 +  + i 2015 .

9. Para todo z ∈  , mostre que:

a) z  z  2 Re( z ) .

b) z z  .

10. Calcule as seguintes potências, utilizando a regra de De Moivre:

a) 1  i 4 .

 1  3i  .
3
b)

178
aprimore-se

Quando um professor entra na sala de aula e diz que iniciará o estudo dos números
complexos, os alunos pensam que são números, no mínimo, muito complicados. Ao
saber que também existem números chamados de imaginários, os alunos dirão que
tais números, por serem imaginários, não existem. Portanto, para que estudá-los?
Resolver equações sempre foi um assunto que fascinou matemáticos ao longo da
história. Os matemáticos antigos da Babilônia já conseguiam resolver algumas equa-
ções do 2º grau baseados no que, hoje, chamamos de “completamento de quadrado”.
Os matemáticos gregos, que desempenharam importante papel no desenvolvi-
mento da matemática, resolviam alguns tipos de equações do 2º grau com régua e
compasso. A conquista da Grécia por Roma praticamente acabou com o domínio da
matemática grega. Com o fim do Império Romano e a ascensão do cristianismo, a
Europa entrou na Idade das Trevas e o desenvolvimento da matemática ficou nas
mãos dos árabes e dos hindus.
Os matemáticos hindus avançaram nas pesquisas em álgebra e Bháskara é o
nome que imediatamente vem à nossa memória quando falamos de equações do
2º grau. Dependendo da equação, poderia acontecer que o número ∆ = b²−4ac fosse
negativo. Entretanto, isso não perturbava muito os matemáticos da época. Nesse
caso, eles simplesmente diziam que o problema não tinha solução.
A Aritmética e a Geometria tiveram origens independentes, mas, com o tempo,
foram sendo descobertas relações entre números e formas. A ideia de empregar sis-
temas de coordenadas para definir posições de pontos no plano e no espaço já havia
sido utilizada no século III a.C. por Apolônio, em seus trabalhos sobre secções cônicas.
No entanto, foi na primeira metade do século XVII que os geniais matemáticos fran-
ceses, Pierre de Fermata e René Descartes, inventaram, independentemente e quase
simultaneamente, o que, hoje, conhecemos por Geometria Analítica. Fermat não se
preocupou em publicar suas ideias, ao contrário de Descartes, que, no apêndice de
seu mais famoso livro Discurso Sobre o Método de Bem Utilizar a Razão e de Encontrar a
Verdade nas Ciências, publicado em 1637, escreveu um trabalho denominado La Geo-
metrie, que é considerado a pedra fundamental da Geometria Analítica.

179
aprimore-se

Com o domínio da Geometria Analítica, Descartes estudou, entre outras coisas,


as equações algébricas. Em uma passagem do Discurso do Método, Descartes es-
creveu a seguinte frase: “nem sempre as raízes verdadeiras (positivas) ou falsas (ne-
gativas) de uma equação são reais. Às vezes, elas são imaginárias”.
Por esse motivo, até hoje o número é chamado de número imaginário, ter-
mo que se consagrou juntamente com a expressão “número complexo”. Infelizmen-
te, são designações um tanto inadequadas e subjetivas para objetos matemáticos.
Depois de Bombelli, em 1530, outros personagens importantes da história da
matemática deram contribuições ao desenvolvimento da teoria dos números com-
plexos, dentre os quais estão o matemático francês Abraham De Moivre, amigo de
Isaac Newton, e os irmãos Jacques e Jean Bernoulli. Todavia, quem fez o trabalho
mais importante e decisivo sobre o assunto foi Euler.

Fonte: Cerri e Monteiro (2001).

180
eu recomendo!

livro

Trigonometria – Números Complexos


Autor: Manfredo Perdigão do Carmo, Augusto César Morgado e
Eduardo Wagner
Editora: SBM
Sinopse: este livro é um texto de apoio utilizado nos cursos de
aperfeiçoamento para professores de Matemática do segundo
grau, um programa organizado pelo Instituto de Matemática
Pura e Aplicada (IMPA), com patrocínio de VITAE, Apoio à Cultura, Educação e Pro-
moção Social. Assim, a obra apresenta elementos de trigonometria e de números
complexos, dando destaque às relações naturais entre esses tópicos.

181
181
conclusão
conclusãogeral
geral

conclusão
conclusão
geral
geral

Caro(a) aluno(a), neste material, você teve contato com alguns dos conteúdos já
vistos na Educação Básica. No entanto a percepção desses assuntos, em muitos
casos, diferiu-se daquele nível de ensino, tendo em vista que este material foi pre-
parado, especialmente, para você que está cursando licenciatura em Matemática.
Para isso, deu-se atenção redobrada nas demonstrações das fórmulas e das pro-
priedades que contribuem para o resgate da origem dos conteúdos e possibilitam
a sua compreensão plena.
Outra preocupação foi com o uso de linguagem de fácil compreensão, para fa-
cilitar o estudo daqueles que buscam novos conhecimentos. Esse fato não isenta
a responsabilidade dos autores, quanto à escolha dos conteúdos e da forma de
apresentá-los, tendo em vista que o uso da linguagem de fácil compreensão não
pode prejudicar a qualidade de ensino, mas deve contribuir para melhor assimi-
lação dos conteúdos, inclusive, por aqueles que, ainda, não estão familiarizados
com termos específicos da disciplina de Matemática. Assim, mesmo com o olhar
um pouco diferente do tradicional, isto é, do tecnicista, assegurou-se a qualidade
de ensino que devemos oferecer para aqueles que sacrificam seu precioso tempo
em busca de novos horizontes.
Além disso, várias observações feitas, ao longo do desenvolvimento deste mate-
rial, poderão ser levadas, inclusive, aos alunos da Educação Básica. Assim, é impor-
tante compreender a validade de algumas fórmulas e as suas propriedades, para
que possa mostra-las aos alunos, pois você, se ainda não é, será professor(a) de
Matemática. Outra sugestão é a construção de sua linha de raciocínio que ajudará
em sua formação acadêmica e em sua preparação profissional.
Esperamos ter correspondido a sua expectativa e ter contribuído para lhe apro-
ximar da concretização do sonho em adquirir conhecimentos suficientes para assu-
mir uma sala de aula e em conquistar o título de licenciado(a) em Matemática pela
conceituada Unicesumar.

182
182
referências

CARMO, M. P. do; MORGADO, A. C.; WAGNER, E. Trigonometria: números complexos. 5. ed.


Rio de Janeiro: SBM, 2001.

CERRI, C.; MONTEIRO, M. S. História dos Números Complexos. São Paulo: Instituto de Mate-
mática e Estatística da USP, 2001. E-book (13 p.). Disponível em: ime.usp.br/~martha/caem/
complexos.pdf. Acesso em: 1 abr. 2020.

DIAS, N. L. Pequena introdução aos números. Curitiba: Intersaberes: 2014.

IEZZI, G. Fundamentos de matemática elementar: trigonometria. 9. ed. Rio de Janeiro: Atual,


2013.

LIMA, E. L. et al. A Matemática do Ensino Médio. 10. ed. Rio de Janeiro: SBM, 2012.

LIMA, E. L. Números e funções reais. Rio de Janeiro: SBM, 2013.

COSTA, N. M. L. da. A História da Trigonometria. Educação Matemática em Revista, São Paulo,


p. 60-69, 1 mar. 2003.

MORGADO, A. C.; WAGNER, E.; ZANI, S. C. Progressões e Matemática Financeira. 3. ed. Rio de
Janeiro: SBM, 2005.

OLIVEIRA, D. P. A.; VIANA, M. da C. V.; ROSA, M. Um pouco da história das funções: algumas
sugestões de atividades práticas para a sala de aula. Bolema, Rio Claro, v. 27, n. 46, p. 513-
529, ago. 2013.

ROLKOUSKI, E. Tecnologias no ensino de matemática. Curitiba: Intersaberes, 2013.

VIOTTO, V. M. Nem só álgebra, nem só aritmética. Revista Professor de Matemática, Dois


Córregos, v. 16, 1990. Disponível em: http://rpm.org.br/cdrpm/16/8.htm. Acesso em: 25 mar.
2020.

ZANELLA, I. A.; ZANELLA, M. S. Introdução ao Cálculo. Maringá: Unicesumar, 2016.

REFERÊNCIAS ON-LINE:

¹Em: mundoeducacao.bol.uol.com.br/matematica/historia-das-equacoes.htm. Acesso em: 24


mar. 2020.

183
gabarito

UNIDADE 1

1. Alternativa D.

I. Falso. Tome, por exemplo, p ( x)  x3  x e q ( x)   x3  x2 , que possuem grau

 
3 . Assim, temos p ( x)  q ( x)  x3  x   x3  x2   x2  x , que possui grau 2 .
II. Falso. Pode-se utilizar o mesmo exemplo do item anterior.

III. Verdadeiro.

IV. Verdadeiro.

2. Alternativa B.

I. Verdadeiro.

II. Falso. Podem-se obter soluções com nenhuma raiz real.

III. Verdadeiro.

IV. Verdadeiro.

3. Alternativa D.

4. Pelo fato de que, anteriormente, os lados mediam 3 e 5 metros, e ambos foram


aumentados igualmente, podemos dizer que o aumento mediu x metros. Em outras
palavras, ou novos lados medem 3+ x e 5 + x , respectivamente.
Como sabemos que a nova área é igual a 35 , obtemos a seguinte equação:
(3  x)  (5  x)  35
 15  3 x  5 x  x2  35
 x2  8 x  20  0
Encaremos as raízes dessa equação, utilizando a Fórmula de Bháskara. Para isso, temos:

184
gabarito

8  82  4  1   20 
x
2
8  64  80

2
8  144

2
8  12

2
 8  12 4
 x1  2  2  2

 x  8  12  20  10
 2 2 2
No entanto, como tratamos de medidas, não podemos utilizar a raiz x2  10 . Por-
tanto, temos x = 2 , ou seja, os lados aumentaram em 2 metros.

5. A taxa inicial é o nosso termo independente e o valor a cada quilômetro é multiplica-


do pela variável, que mede exatamente a quilometragem percorrida. Logo, a equação
é dada por: 4  0, 75 x  50
6. Vamos denotar a quantidade de ingressos inteiros por x e de meias-entradas por y .
Assim, sabemos que o total arrecadado foi de R$5985, 00 , isto é:
18 x  9 y  5985

Além disso, sabemos que, no total, foram vendidos 380 ingressos. Em outras pala-
vras, todos os ingressos de entrada inteira somados com todos os ingressos de meia-
-entrada são 380 , isto é:
x  y  380 .

Assim, obtemos o sistema:


18 x  9 y  5985

 x  y  380

185
gabarito

Isolando a variável x na segunda equação e substituindo na primeira, temos y = 95 .


Assim, x = 285 . Portanto, foram vendidas 285 entradas inteiras e 95 meias-entradas.

7. Seja x o comprimento da cerca, sabemos que o perímetro de um retângulo é dado


por: 2 ⋅ largura + 2 ⋅ Comprimento . Se a cerca tem 20 m de largura e sabemos que o
perímetro deve ter, pelo menos, 180 m, temos:

2  x  2   20   180
 2  x  40  180
 2  x  40  40  180  40
 2 x  140
1 1
 2 x     140   
2 2
 x  70
Portanto, a cerca deve ter comprimento maior ou igual a 70 m.

8. Queremos saber quando (ou seja, o tempo t ) a velocidade do objeto estará entre 32
e 64 metros por segundo, ou seja, quando a velocidade será maior do que 32 e
menor do que 64 . Como temos a fórmula da velocidade, temos:
32  80  32t  64
 32  80  32t  64  80
 1 
 48  32t  16   
 32 
 1   1   1 
  48       32t      16    
 32   32   32 
 1, 5  t  0, 5

Assim, constatamos que o objeto estará entre 32 e 64 metros por segundo entre 0,5
segundo e 1,5 segundos depois do lançamento.

186
gabarito

9. Vamos denotar de P a quantidade de vacinas contra Pólio dadas pela Secretaria


de Saúde e de S a quantidade de vacinas contra Sarampo. Sabemos que, no total,
foram 60 vacinas, logo, temos a nossa primeira equação: P  S  60 . Além disso,
sabemos que cada vacina de Pólio são 4 doses e cada vacina de Sarampo são 2
doses, e que, no total, foram dadas 184 doses. Isso nos dá a nossa segunda equação
e podemos escrever nosso sistema de equações:

 P  S  60

4  P  2  S  184

Para resolver esse sistema, vamos isolar P na primeira equação e substituí-lo na segunda:

 P  S  60
  P  60  S
4  P  2  S  184
 4   60  S   2 S  184
 240  4 S  2 S  184
 2 S  56
 S  28

Assim, substituindo esse valor na primeira equação, constatamos que foram realiza-
das 28 vacinas contra Sarampo e 32 vacinas contra Pólio nessa Secretaria de Saúde.

10. Como temos um produto entre dois termos menor do que 0, precisamos
analisar os sinais de seus termos, tais que possuam sinais contrários. Isto é:
3 x  6  0 e 5x-7<0 ou 3 x  6  0 e 5x-7<0 .
Analisando o primeiro caso, temos:

1
 3 x  6  0  3 x  6  
3
 1  1
 3 x      6    
 3  3
 x2

187
gabarito

1
5x  7  0  5x  7  
5
1 1
 5x     7   
5 5
7
x
5
Assim, para o primeiro caso, a solução é a interseção desses intervalos. Isto é:

 7  7
 ,    , 2    , 
 5  5
Agora, precisamos analisar o segundo caso. Temos:

 1
3 x  6  0  3 x  6   
 3
 1  1
 3 x      6    
 3  3
 x2
e

1
5x  7  0  5x  7  
5
1 1
 5x     7   
5 5
7
x
5

Assim, para o segundo caso, a solução é a interseção desses intervalos. Isto é:

7 
 ,     2,     2,   .
5 

188
gabarito

Para finalizar, precisamos unir as duas possibilidades. Portanto, o conjunto solução da


inequação é:

 7   7
S   x   | x  ou x  2    ,  2, 
 5   5

UNIDADE 2

1.

10
a. em que x≠0.
2x

x +1
b. em que x≠0.
3x

x
c. em que x  1 .
x +1

2.

a. O denominador deve ser diferente de zero, ou seja:

x 1  0
x 1

b. O denominador deve ser diferente de zero, ou seja:

2x  6  0
2 x  6
6
x
2
x  3

189
gabarito

c. O denominador deve ser diferente de zero, ou seja:

x2  1  0
x2  1
x  1
x  1 e x  1

d. O denominador deve ser diferente de zero, ou seja:

x y 0
x y

3.

x 1  y  1  y
a.  .
x 1  z  1  z

b. A fração é irredutível, portanto, não é possível simplificar.

2 y  y  5
c.  2y .
y 5

 a  b   a  b 2

 a  b a  b
2

a b
d. .
a  a2  ab  b2 
2 a
a a  b

4. Sabemos que a região retangular é dada da forma:

2/y

190
x/y
gabarito

Assim, o perímetro pode ser representado da forma:

x 2 x 2 x  2  x  2 2x  4
     .
y y y y y y
Por fim, a área pode ser apresentada da forma:

x 2 2x
  .
y y y2

5.

60 8 x 15 x
 
20 x 20 x 20 x
60  8 x  15 x
a. 8 x  15 x  60
7 x  60
60 4 4
x ou x  8  ou x  8
7 7 7

2 1

 x  2  x  2  x  2
2   x  2

 x  2  x  2  x  2  x  2
b. 2  x  2
x  22
x0

191
gabarito

1 2 3
 
 x  3  x  3  x  3 x  3
 x  3  2

3  x  3
 x  3  x  3  x  3  x  3  x  3  x  3
x  3  2  3x  9
c. x  3x  2  9  3
2 x  10
10
x
2
x5

6.

1 1 1
 
9 x 4
4x 36 9x
 
36 x 36 x 36 x
4 x  36  9 x
4 x  9 x  36
5 x  36
36 1
x  h ou x  7 h ou x  7h e 12 min
5 5

UNIDADE 3

1. Alternativa E.

( V ) Utilizando a Relação Fundamental da Trigonometria, temos:

sen 2 a  cos2 a  1
 sen 2 a  cos2 a  cos 2±  1  cos 2±
 sen 2 a  1  cos2 a
192
gabarito

1
sec a 
cos a
(V)  sec a  cos a  1
1
 cos a 
sec a

1 1 1 cos a cos a
(V) cotg a      .
tg a sen a 1 sen a sen a
cos a

2. Sabemos que h  2  5 cm e que c1  2  c2 . Logo, pelo Teorema de Pitágoras, temos:

h2  c12  c22

 
2 2
 2 5   2  c2   c22

  4  5   4  c22  c22
 20  5  c22
20
 c22 
5
 c2  4
 c2  2 cm

Portanto, c1  2  c2  c1  4 cm.

3. Sabemos que, pela Lei dos Senos, temos:

AC BC
 .

sen ABC 
sen B AC  
Assim:

193
gabarito

AC 6

sen 30º sen 45º
AC 6
 
1 2
2 2
1
6
 AC  2
2
2
3
 AC 
2
2
2 2
 AC  3  
2 2
 AC  3  2 cm

4. A transformação de quadrado para losango ocorre da seguinte forma:

D D

D C
8 cm 60° 8 cm 30°

8 cm A C A C
E

A B

B B

194
gabarito

d1  d2
Sabemos que a área de um losango é dada por AL  , em que
2
d1 d
d1 = AC , d2 = BD x
são suas diagonais. Então, se= AE
= = 2
y DE
e= ,
2 2
fazendo os procedimentos mostrados na figura, temos:

x 1 x
sen 30º     x  4.
8 2 8
e

y 3 y
cos 30º     y  4 3 .
8 2 8

Portanto, sabemos que d1  2  x  d1  8 e d2  2  y  d2  8  3 . Assim, temos:

88 3
AL 
2
 32  3 cm2

5. Primeiramente, construiremos a altura CD em relação à base AB , assim como


demonstra a figura a seguir:

45° x cm (21-x) cm 30°


A 21 cm B

195
gabarito

Cateto Oposto
Sabemos que tg a = , logo, temos:
Cateto Adjacente

h h
I. tg 45º  1  x  h;
x x

h 3 h
tg 30º   
21  x 3 21  x
3h
  21  x
II. 3
3h 3
   21  x
3 3
 x  21  3  h

Dessa forma, como h=x e x  21  3  h , temos:

h  21  3  h
 h  3  h  21
 h  1, 731  h  21
 2, 731  h  21
 h  7, 69 cm

Logo, resta-nos calcular a área, a qual é dada por:

Base  h 21  7, 69
A   80, 74 cm2 .
2 2

196
gabarito

6.

1º. Por meio do Teorema de Pitágoras, considere o triângulo equilátero a seguir:

l h

A l D B
2

Pelo Teorema de Pitágoras, temos:

2
2 l2
l  h  
2
l2
 h2  l 2 
4
3l 2

4
3l 2
h
4
l 3
 .
2
Logo, a área do triângulo é dada por:

l 3
l 2
l h 2  l  3.
A 
2 2 4
197
gabarito

2º. Por meio da razão trigonométrica, considere o triângulo a seguir:

l h

60°
A l D B
2

Cateto Oposto
Sabemos que sen a = . Logo, temos:
Hipotenusa

h 3 h l 3
sen 60º    h .
l 2 l 2
Portanto, a área do triângulo é dada por:

l 3
l 2
l h 2  l  3.
A 
2 2 4
7. Alternativa C.

1 1 1 cos a
(V)     cotg a
tg a sen a 1 sen a
cos a

198
gabarito

1
(F) = cossec a
sen a

( F ) Pela Relação Fundamental da Trigonometria, temos:

sen 2 a  cos2 a  1

1
Multiplicando todos os termos por , temos:
sen 2 a

sen 2 a cos2 a 1
2
 2

sen a sen a sen 2 a
 1  cotg 2a  cossec 2a

Por outro lado, novamente, partindo da Relação Fundamental da Trigonometria, ago-

1
ra, multiplicando todos os termos por , temos:
cos2 a

sen 2 a cos2 a 1
 
cos2 a cos2 a cos2 a
 tg 2a  1  sec 2a

Portanto:

sec2 a  cossec 2a  cotg 2a  tg 2a  2.

1 1 1 cos a cos a
(V) cotg a      .
tg a sen a 1 sen a sen a
cos a

199
gabarito

1
8. Pelo fato de que sen 2 a  cos2 a  1 , ao multiplicar todos os termos por ,
cos2 a
temos:

sen 2 a cos2 a 1
2
 2

cos a cos a cos2 a
 tg 2a  1  sec 2a
 tg 2a  sec2 a  1

9. Alternativa B.

I. Errado:

sen a  0  3º ou 4º quadrantes.

tg a  0  3º quadrante.

II. Correto:

cos b  0  1º ou 4º quadrantes.

cotg b  0  1º ou 3º quadrantes.

III. Errado:

sec g  0  1º ou 4º quadrantes.

cossec g  0  1º ou 2º quadrantes.

10. Visualizando o esquema, temos o seguinte triângulo retângulo:

200
gabarito

30° 60°
50 m x

Cateto Oposto
Sabemos que tg a = . Logo:
Cateto Adjacente

h
tg30º 
50  x
3 h
 
3 50  x
3
   50  x   h
3
50 3 3x
  h
3 3
3x 50  3
  h
3 3
 50  3  3
 x   h  
 3  3
3h  50 3
x
3
Por outro lado, temos:

201
gabarito

h
tg 60º 
x
h
 3
x
h
x
3
A partir disso:

h 3h  50 3

3 3
 h  3h  50 3
 h  25 3

Agora, precisamos lembrar que a pessoa tem 3 metros de altura e, por isso, preci-
samos somar esse valor à altura encontrada. Portanto, a altura do prédio é:

H  25 3  3  26 3 m.

UNIDADE 4

1.

I.  33, 16,1 , 18, .

Fazendo 16   33   17, 1   16   17, 18  1  17 . Portanto, trata-se de

uma PA de razão r = 17 .

202
gabarito

1 1 1 1  1 1 23 1 1 1 34 1
II.  , , , ,  . Note que    e     . As-
2 3 4 5  3 2 6 6 4 3 12 12

sim, podemos afirmar que não se trata de uma PA.

III. 141,137,133,129  .

Fazendo 137  141  4, 133  137  4, 129  133  4 . Portanto, trata-
-se de uma PA de razão r  4 .

IV. 1, 2, 4, 7  . Note que 2  1  1 , mas 4  2  2 . Logo, não se trata de uma PA.

2. Se uma PA possui 21 termos, existem 20 termos após o primeiro, ou seja, a razão é


somada ao primeiro termo 20 vezes, e não 21 vezes assim como foi feito.

3. Sabemos que an  a1  (n  1)  r . Assim, temos:


an  7  (n  1)  3
 7  3n  3
 an  3n  4

Agora, também sabemos que a soma dos n primeiros termos de uma PA é dada por:

(a1  an )
S n .
2

Substituiremos o valor de na fórmula da soma dos termos:

203
gabarito

Sn 
 a1   3n  4    n
2

 205 
 7  3n  4   n
2
 410  3n2  11n
 3n2  111n  410  0

Resolveremos essa equação do segundo grau por meio da Bháskara:

b  b2  4 ac
n
2a
11  121  4  3   410 

23
11  50441

6
11  71
n
6
 60
n1  6  10;

n   82
 2 6
Contudo, como não podemos ter uma quantidade de termos negativos, sabemos

que n = 10 . Portanto, an  3n  4  an  3  10  4  a n = 34 .

=
4. Sabemos que n , a1 80, a7 = 50 . A partir disso, constatamos que a soma dos
7=
7 primeiros termos da PA é dada por:

204
gabarito

S7 
 a1  a7   7
2


 80  50   7
2
 455.

Agora, se são 7 pedaços, são necessários 6 cortes e como se perde 0, 5 cm em cada


corte, no total, são perdidos 6  0, 5  3 cm. Portanto, o comprimento mínimo da ma-
neira deve ser 455  3  458 cm.

5. De fato, sejam a1  x  4, a2  2 x  7 e a3  4 x  6 . Agora, para que esses valo-


res formem uma PA, sua razão deve ser:

r1  a2  a1  2 x  7  ( x  4)  x  3
r2  a3  a2  4 x  6  (22 x  7)  2 x  1

Mas sabemos que r1 = r2 .


x  3  2x 1
Logo, temos:
x4

Portanto:

a1 = 4 + 4 = 8;
a2 = 2 × 4 + 7 = 15;
a3 = 4 × 4 + 6 = 22.

Logo, trata-se de uma sequência com primeiro termo igual a 8 e razão r =7.

205
gabarito

6. De fato, sejam an1  50 x  4, an  10 x2  4 e an1  52 x  2 . Agora, para que


esses valores formem uma PA, sua razão deve ser:

r1  an  an1  10 x2  4  (50 x  4)  10 x2  50 x  8
r2  an1  an  52 x  2  (10 x2  4)  10 x2  52 x  2.

Todavia, sabemos que r1 = r2 . Logo, temos:

10 x2  50 x  8  10 x2  52 x  2
1
 20 x2  102 x  10  0  
2
 10 x2  51x  5  0

Resolveremos essa equação do segundo grau por meio da Bháskara:

b  b2  4 ac
x
2a
51  2601  4  10  (5)

2  10
51  2401

20
51  49
x
20
 100
 x1  20  5;

x  2  1 .
 2 20 10

1
Logo, obtivemos as raízes x1 = 5 e x2 =. Substituiremos esses valores nos an ' s
10
para encontrar a sua razão. Assim, substituindo primeiramente x1 , temos:

206
gabarito

r1  an  an1  10 x2  50 x  8  r1  10  52  50  5  8  8
Agora, para x2 , temos:

r2  an  an1  10 x2  50 x  8
2
1 1
 r 2  10     50     8
 10   10 
1
 r2   5  8
10
1  50  80

10
31

10

31
Note que r2 = não é um número natural, então, não obedece à hipótese do
10
problema. Portanto, a razão é r =8.

7.

I.  256, 128, 64, 32, . Note que:

128 1 64 1 32 1 1
  ;  ;   . Portanto, trata-se de uma PG de razão q   .
256 2 128 2 64 2 2

1 1
1 1 1 1  3  12  2 4  13  3
II.  , , , ,  . Note que
1 , mas
1 . Além
2 3 4 5  3 1 3 4 1 4
2 3
2 3
disso, como ≠ , é sabível que não se trata de uma PG.
3 4

207
gabarito

12 36 3
III.  6,12, 24, 36  . Note que = 2 , porém = . Logo, como existem razões
6 24 2
diferentes, não se trata de uma PG.

1 1 1 1 
IV.  , , ,  . Note que:
 3 9 27 81 

1 1 1
9  13  1; 27  1  9  1 ; 81  1  27  1
. Portanto, trata-se de
1 9 1 3 1 27 1 3 1 81 1 3
3 9 27
1
uma PG de razão q= .
3

8. Primeiramente, descobriremos qual é o termo a5 . Para isso, sabemos que


n 1
an  a1  q . Assim, temos:

a5  5  251
 a5  5  16
 80.
Agora, encontraremos a soma dos 10 primeiros termos da PG:

Sn 

a1  1  q n 
1 q

 S10 

5  1  210 
1 2
 S10  5  1  1024 
 S10  5115

Portanto, S10  a5  5115  80  5035

208
gabarito

9. Note que os números múltiplos de 4 formam a seguinte PA:

 4, 8, , 85964, 85968 


Isto é, podemos notar que a razão é dada por: an  an1  4 . Em outras palavras,
sabemos que o primeiro termo é a1 = 4 , o último termo é an = 85968 e que a ra-

zão é r = 4 . Logo, basta utilizarmos a fórmula do termo geral:

an  a1   n  1  r
 85968  4   n  1  4
 85968  4  4 n  4
859968
n
4
 n  21492

10. Para calcular o termo seguinte de uma PG, basta multiplicar o termo anterior pela
razão da PG. Portanto, para calcular dois termos a frente, basta multiplicar pelo qua-
drado da razão e, para calcular três termos a frente, basta multiplicarmos pelo cubo

da razão. Logo, se a PG é dada por  a1 , a2 ,10, a4 , a5 , 80  , temos:


a6  a3  q3
 80  10  q3
 q3  8
 q  2.

209
gabarito

UNIDADE 5

1. Resolveremos a equação, utilizando a Fórmula de Bháskara. Assim:

b  b2  4 ac
z
2a
8  82  4  2  16

22
8  64  128

4
8  64

4
8  64  (1)

4
8  64  i 2

4
8  8i

4
 8  8i
 z1  4  2  2i

 z  8  8i  2  2i
 2 4

2. Primeiramente, se z  1  3i , sabemos que, escrito na forma de par ordenado,


esse número é dado por:

z  1,  3  .

Na forma geométrica, é dado assim como mostra a imagem a seguir:

210
gabarito

-3 -2 -1 0 1 2 3

-1

-2
z1 = -1 - 1.73i

-3

Para escrevermos na forma trigonométrica, precisamos encontrar o módulo e o argu-


mento. Dessa forma, temos:

 12    
2
r 3  4 2
e

a 1
cos θ -
ρ 2 4π
θ rad 240”
b 3 3
senθ -
ρ 2

z  ρ  cos θ  i  senθ   2   cos  240º   i  sen  240º   .

211
gabarito

3.

a. Primeiramente, precisamos dividir 33 por 4 . Efetuando essa divisão, sabemos


que 33  4  8  1 , isto é, a divisão possui resto 1 . Logo, temos:

i 33= i1= i .

b. Efetuaremos a divisão de 999 por 4 . Assim, obtemos 999  4  249  3 .


Logo, sabemos que a divisão possui resto 3 e, portanto:

i 999  i 3  i 2  i   1  i  i .

4.
1  i i 1  i    i  i  i 2 i  1 i  1
a.   2
 2
   i  1 .
i i i i (1) 1

1  2i (2  i ) (1  2i )  (2  i ) 2  i  4i  2i 2 2  5i  2 5i
b.       i.
2  i (2  i ) (2  i )  (2  i ) 4  i2 4  (1) 5

5. Basta fazermos Re( z ) = 0 . Assim, temos:


7a 7a 372
 12  0   12  7 a  31  12  a  .
31 31 7

6. Basta fazermos Im( z ) = 0 . Desse modo, temos:


b b
3  0   3 b  6 .
2 2

212
gabarito

7. Note que z1  2  i e z2  2  3i . Assim, temos:

z1  z2   2  i    2  3i 
■   2  2    1  3  i
 4  4i

z1  z2   2  i    2  3i 
  2  2  1  3    2  3  2  1 i

  4  3  6  2  i
 1  8i

8. Note que i 0  i1  i 2  i 3  1+ i -1- i = 0 e mais:

      
i 4  i5  i6  i7  i3  i  i2  i3  i3  i3  i2  i2  i3 
  i  i    1  (i )    i  (i )    1  (1)  (i ) 
 1  i  1  i
0

Dessa forma, notamos que, em grupos de 4 termos, a soma é 0 e o último termo

dividido por 4 sempre dá resto 3 . Agora, note que dividindo 2015 por 4 , temos:
2015  4  503  3 . Portanto, sabemos que todos os elementos da soma podem

ser agrupados em grupos de 4 termos, os quais possuirão soma igual a 0. Logo,


0 1 2 2015
i  i  i   i 0.

213
gabarito

9. Seja z  a  bi e z  a  bi . Logo, temos Re( z ) = a e Im( z ) = b . Assim:

a.  a  bi    a  bi    a  a    b  b  i  2  a  2  Re( z )

b.  a  bi    a  bi   a2  abi  abi  b2i2  a2  b2  

10. A fórmula de De Moivre é dada por: z n  ρ n  cos  n  θ   i  sen  n  θ   .

a. Primeiramente, precisamos calcular o módulo de z , isto é:

r  12  12  2

a 1 2 2
cos θ     
ρ 2 2 2  π
  θ  rad  45”
b 1 2 2 4
senθ    
ρ 2 2 2 
Portanto, temos:

1  i 4      cos  4  45º   i  sen  4  45º 


4
2
 4   cos 180º   i  sen 180º  
 4   1  0 
4

214
gabarito

b. r  (1)2  ( 3 )2  4  2

a 1
cos θ   
ρ 2 2π
θ  rad  120”
b 3 3
senθ  
ρ 2 
Dessa forma, temos:

 1  3i 
3
 23   cos  3  120º   i  sen  3  120º  
 8   cos  360º   i  sen  360º  
 8  1  0 
8

215
anotações




































Você também pode gostar